Vous êtes sur la page 1sur 78

THE STRUCTURE OF THE TRIAL

ORDER OF PROCEEDINGS
1. Voir Dire = entails the questioning of jurors in open court, and allows for three judicially sanctioned purposes:
a. Opportunity to determine whether the juror meets the statutory requirements
b. Determines whether a juror may be challenged for cause because he is not able to impartially decide the issues
submitted to the jury solely on the law and evidence
c. Allows attorney to gather information for preemptory challenges (the right to excuse a prospective juror for any
reason, or for no reason)
2. Opening Statements
3. Plaintiffs Case in Chief = Prima Facie Case: facts sufficient to establish each element of the claim asserted or the offense
charged
a. Each witness of the Plaintiff/Prosecution will be first questioned under direct examination, then cross-examined by
opposing council
b. These examinations may be followed by redirect and recross examination
4. Defendants denials and affirmative defenses = introduction of witnesses and documents in support of his case, including
items authenticated during cross-examination of Plaintiffs witnesses as well as evidence in support of any affirmative
defense(s)
a. Each witness of the Plaintiff/prosecution will be first questioned under direct examination, then cross-examined by
opposing counsel
b. These examination may be followed by redirect and recross examination
5. Plaintiffs Rebuttal = confined to testimony directed to refuting the Ds evidence. Not entitled to present witnesses who
merely lend support to the evidence originally presented
a. Proper scope and function is refutation, which involves evidence which denies, explains, counteracts, qualifies,
disproves, repels, or otherwise sheds light on evidence offered by the defense including evidence rehabilitating the
credibility of witnesses
b. At discretion of judge, evidence tending to refute is admissible in rebuttal even if it would also have been admissible
as part of the case in chief
6. Defendants surrebuttal (rejoinder) = if new points are brought up during Plaintiffs rebuttal, then D may meet them by
evidence
a. Defendant may offer evidence to rehabilitate witnesses whose credibility has been attacked in Plaintiffs rebuttal
7. Defendants counterclaim = Ds case in chief with respect to the counterclaim is normally presented together with his
evidence in opposition of the Plaintiffs main case
a. Plaintiff presents evidence in opposition when he presents rebuttal evidence
b. Such evidence is responded to by Defendant on surrebuttal
8. Both sides close = hearing on the facts comes to an end, and closing arguments begin
9. Reopening of the case = Courts discretion extends to reopening a case for the taking of further evidence after a party (or
even both sides) have closed
a. While liberality in favor of reopening is to be encouraged to afford the fullest possible hearing, the additional
evidence should not be allowed absent a showing of diligence and most certainly should not be allowed if
deliberately withheld in an attempt to deprive the opponent of an opportunity to meet it
10. Jury Instruction
TYPES OF EVIDENCE
Viva Voce Testimony: live testimony in front of a jury as to what people actually observed. Testimony is under oath,
subject to cross examination, and demeanor visible to the jury.
Documentary Evidence: pieces of paper entered into the record. Evidence likely needs viva voce testimony to verify the
document
Real Evidence: something from which jury can get a relevant first hand sense or impression, like the gun used, or the car
in the accident
Demonstrative Evidence: Evidence that helps people to understand a witnesss testimony (viva voce). Has no real
probative value, used to assist the jury in trying to understand the case, expert witnesses, can be maps, charts, etc
Experiments: Conducted in or out of court to explain what something is or how something happened
PRELIMINARY QUESTIONS OF ADMISSIBILITY
RULE 104. PRELIMINARY QUESTIONS
a. Generally. Court determines preliminary questions concerning witnesses, the existence of privilege(s), or the admissibility of
evidence. (not bound by evidence rules except regarding privileges)

b.

c.

d.
e.

Relevancy Conditioned on Fact. When the relevancy of evidence depends on the fulfillment of a condition of fact, such as
authentication or personal knowledge, a prima facie showing of their existence is required as a prerequisite to admitting the
evidence. Thus, evidence sufficient to support a finding by a reasonable juror of the fulfillment of the condition must be
introduced, i.e. evidence which when viewed in a light most favorable to the proponent would permit a reasonable juror to
find that the condition is more probably true than not true
a. The court may consider only that evidence which the jury will have before it
b. Where appropriate, the jury may be instructed to determine whether the evidence is relevant be deciding whether the
condition is fulfilled, and to ignore or employ the evidence accordingly.
c. Moreover, if considering all the evidence upon the issue including evidence subsequently introduced by the
opponent the jury could no longer reasonably find that the condition was fulfilled, the court should instruct the jury
to disregard the evidence
d. Where the admissibility of evidence depends upon connecting facts, the order of proof is largely within the
discretion of the trial judge. Evidence admitted upon a promise to connect it up will be excluded on a motion to
strike if the appropriate connection has not been established
Example: D yells to P a warning about Ds car, which he just loaned to P, the judge must determine whether there is
enough evidence to allow a reasonable juror to believe that the warning was heard (the conditioned fact). Thus, in order
for D to introduce evidence of his warning, the judge must find that the evidence is more likely true
Hearing of Jury. Hearings about confessions admittance is conducted outside the jury. Hearings on other matters shall also
be conducted outside the jury when the interests of justice requires, or when an accused is a witness and so requests.
a. Hearings concerning issues under 104(a) and (b): court must weigh the potential for prejudice derived from the
jurys hearing the evidence against such trial concerns as waste of time caused by taking the same testimony twice
and the dislike of juries being excluded
Testimony by Accused. The accused doesnt, but testifying upon a preliminary matter, become subject to cross as to other
issues in the case, but it may be admitted to impeach the Defendant if he testifies
Weight and Credibility. This rule does not limit the right of a party to introduce before the jury evidence relevant to weight or
credibility

Rule 104 Rules of evidence are administered by the trial judge. Submitting unfounded and irrelevant factual issues as those which
are listed under 104(a) to the jury would be hopelessly confusing, as well as beyond the probable capacity of the jurors. Even if jury is
given an instruction to erase it from their minds, would be difficult to do so.
Reasons that issues are decided solely by the court: (a) Some of the rules which exclude relevant evidence rest upon policy
considerations having no intimate connection with weight and credibility. (b) other rules rest upon policy considerations intimately
connected with or overlapping weight and credibility. In the interest of saving time, the practice of presenting the preliminary evidence
to the court and the jury simultaneously seems proper, except in cases in which a ruling against admissibility would allow the jury to
hear prejudicial evidence.
RULE 103. RULINGS ON EVIDENCE
(a) Effect of Erroneous Ruling. Error will only be found when there is an admittance or exclusion of evidence that substantially
affects the right of a party AND
1. Objection. In the case the ruling is one admitting evidence, a timely objection or motion to strike appears of record,
stating the grounds for the objection if the specific ground was not apparent; or
2. Offer of proof. In the case of excluding evidence, the substance of the evidence was made known to the court by
offer or was apparent from the context.
Once the
(b) Record of Offer and Ruling. The court may add any other or further statement which shows the character of the evidence, the
form in which it was offered, the objection made, and the ruling thereon.
(c) Hearing of Jury. In jury cases, proceedings shall be conducted, to the extent practicable, so as to prevent inadmissible
evidence from being suggested to the jury by any means, such as making statements or offers of proof or asking questions in
the hearing of the jury.
1. Examples: objectionable questions propounded to the witness, the making of statements by counsel, and offers of
proof in the hearing of the jury.
(d) Plain Error. Nothing in this rule prevents the taking of notice of plain error that affects a substantial right(s) although they
were not brought to the attention of the court
Policy of 103(a) = designed to afford the court an intelligent basis of decision and to enable the opponent to take possible corrective
steps. Thus, a mere technical violation of a rule properly brought to the courts attention is not a sufficient basis for reversal, and the
appellate court must also find that the admission or exclusion affected a substantial right of a party.

Motion in Limine = seeking a ruling as to the admissibility of evidence in advance of its introduction at trial.
1) May be made pre-trial or simply set up before the introduction of the evidence that will be discussed during the hearing.
2) Not only shields the trier of fact from inadmissible evidence being presented in their presence as well as the unfair prejudice
that may arise from the mere asking of a question and the making of an objection, it also affords a basis for advance planning
of trial strategy
3) Court has considerable discretion to rule or refuse to rule upon the motion prior to the actual offer of the evidence at trial, and
frequently a pretrial ruling would be inappropriate in light of the necessity to determine the admissibility of the evidence in
relation to factors which themselves would surface only at trial.
Once the court makes a definitive ruling on the record admitting or excluding evidence, either at or before trial, a party need not renew
an objection or offer of proof to preserve a claim of error for appeal. Conversely, a ruling made by the court whch is not definite does
not preserve error for appeal; the losing party is required to either renew the objection or make an offer of proof at trial. And nothing
prohibits the court from revisiting its decision when the evidence is actually offered at trial.
Appellate Review of Rulings on Evidence
1. The abuse of discretion standard: to the extent the trial courts determination turns on an interpretation of a rule of evidence,
i.e., a mistake of law, the review is plenary (de novo review)
2. Where the trial court has made a factual finding, the standard of review is clearly erroneous. Thus, a finding of fact will be
reversed only if it is completely devoid of a credible, evidentiary basis or bears no rational relationship to the evidence in
support.
3. Application of evidence to the facts, is reviewed applying the abuse of discretion standard. Reversal will occur only if the
ruling is manifestly erroneous, i.e. the trial court commits a clear error of judgment.
RULE 105. Limited Admissibility = When evidence is admissible as to one party or for one purpose but not admissible as to another
purpose is admitted, the court, upon request, shall restrict the evidence to its proper scope and instruct the jury.
Upon objection, the offering party has the burden of advising the court as to the party or the limited purpose for which the
evidence is admissible, and the opposing party is then entitled to request an instruction to the jury (a limiting or cautionary
instruction), restricting their use of the evidence to its proper scope.
o Example: the admission of past criminal convictions against a defendant is being admitted for character for
truthfulness and only for that purpose, it IS NOT to be used to determining whether or not the defendant committed
the alleged crime.
Counsel frequently refrain from requesting a limiting instruction in order to avoid emphasizing potentially damaging
evidence
Absent plain error, a failure to request a limiting instruction precludes raising the question on appeal
On occasion, despite the availability of a limiting instruction, evidence will be excluded
o Consideration = If, after taking into consideration the probable effectiveness of a limiting instruction, the court
determines that the probative value is substantially outweighed by the dangers of unfair prejudice, confusion of the
issues, or misleading the jury (Rule 403), the evidence will not be admitted
An admission by a non-testifying co-defendant implicating the accused is not admissible.
o BUT, a properly redacted statement of a non-testifying co-defendant, not substantively admissible against him, may
be introduced with a proper limiting instruction, when the confession is redacted to eliminate not only his name but
any reference to his existence
o A properly redacted statement is one that does not facially incriminate the co-defendant
o Been held to satisfy rule with the insertion of we or they or someone when following reference to other
evidence, does not refer to and/or implicate
JUDICIAL DISCRETION
Questions involving the exercise of discretion concerning the admission or exclusion of relevant evidence arise with regard to the
courts overall obligation to administer the conduct of the trial in the interests of the ascertainment of truth. 3 federal rules bear
directly upon the courts overall discretion to ensure the ascertainment of truth.
RULE 102. PURPOSE AND CONSTRUCTION = These rules shall be construed to promote fairness in administration, elimination
of unjustifiable expense and delay, and promotion of development of the law to the end that the truth may be ascertained and
proceedings justly determined.
RULE 403. EXCLUSION OF RELEVANT EVIDENCE ON GROUNDS OF PREJUDICE, CONFUSION, OR WASTE OF
TIME. Relevant evidence may be excluded if its probative value is substantially outweighed by the danger of unfair prejudice,

confusion of issues, or misleading the jury, or by considerations of undue delay, waste of time, or needless presentation of cumulative
evidence.
RULE 611(a). MODE AND ORDER OF INTERROGATION AND PRESENTATION.
(a) Control by court. The court shall exercise reasonable control over the mode and order of interrogating witnesses and presenting
evidence so as to:
(1) Make the interrogation and presentation effective for the ascertainment of truth;
(2) Avoid needless consumption of time;
(3) Protect witnesses from harassment or undue embarrassment

RELEVANCE Article 4
Evidence, which is relevant, Rule 401, is admissible except as otherwise provided by law, Rule 402. Evidence which
is not relevant is not admissible, Rule 402. Relevant evidence may be excluded under Rule 403 if its probative value
is substantially outweighed by the danger of unfair prejudice, confusion of the issues, or misleading the jury, or by
considerations of undue delay, waste of time, or needless presentation of cumulative evidence. The admissibility of
evidence and the determination of logical relevancy merges with the concerns comprising grounds of exclusion
under Rule 403.
RULE 401. RELEVANT EVIDENCE = evidence having any tendency to make the existence of any facts that are of
consequence to the determination of the case more probable or less probable than it would be without the evidence.
Fact of consequence used in place of materiality, which serves to clarify that the breadth of admissibility of relevant
evidence extends to facts not in dispute and also evidence which is essentially background in nature offered as an aid to
understanding
o Example: biographical information concerning a witness, charts, photos, etc.

Any tendency: means more than zero, so this makes almost anything relevant; must make the factual proposition it is being
used to assert more true than not true
o Not necessary that the item of evidence alone convince the trier of fact or be sufficient to convince the trier of fact of
the truth of the proposition for which it is offered. All that is required is that the item offered, taken alone or in
conjunction with other evidence in the case, might suggest the inference proposed to a reasonable man

fact that is of consequence to the determination of the case: this means the facts the evidence tends to establish must have a
link to the elements of the cause of action or defense. (Probative Value_.
o Example: in a medical malpractice action the Plaintiff wishes to introduce evidence that the doctor was drunk the
night before the operation.
o If the Plaintiff is asserting that the doctor failed to perform the procedure properly, such evidence is relevant
o If the plaintiff is only asserting that the procedure selected two weeks before the operation was not suitable, then
evidence of drunkenness is irrelevant to establish that the selected procedure was not suitable.
o NOTE: replaces the term material proposition (which meant ultimate fact)

Three General Categories Encompassing Facts of Consequence to a Determination of the Action


(1) Facts which comprise direct evidence of an element of a claim or defense. (referred to at c/l as material propositions)
(2) Facts from whose establishment may be inferred facts amounting to elements of claims or defenses
a. Intermediate facts which are circumstantial evidence of elements of claims or defenses
(3) Facts bearing circumstantially upon the evaluation of the probative value to be given to other evidence in the case, including
demonstrative evidence and the credibility of witnesses, including demeanor, impeachment, rehabilitation, and background
information.
*Questions of logical relevancy arise solely with respect to the latter two categories dealing with circumstantial evidence

Relevant: describes either direct or circumstantial evidence which has a tendency to prove or disprove a fact of consequence
o Under Rule 401: the objection irrelevant raises both concepts, i.e. the evidence does not tend to make more or less
probable the fact it is offered to establish OR the fact for which evidence is offered to establish is not itself the
consequence of the litigation

1.

Direct Evidence: Evidence whose sole inference directly establishes an element of the cause of action, aka a fact of
consequence.
a. Example: testimony such as I saw X shoot B, is direct evidence as to a fact of consequence, that A shot B.
b. NEVER IRRELEVANT

2.

Circumstantial Evidence: Evidence offered to establish a fact of consequence where an inference in addition to the truth of
the matter asserted need be made (need additional reasoning).
a. Example: the police stop a man fleeing from the scene of a stabbing and find a bloody knife on him. The police did
not see the stabbing, so the jury must make an inference that finding the man with a bloody knife near the scene, at
the time of the incident, makes it more likely than not that he was the stabber.
b. RELEVANCY CONDITIONED ON PROBATIVE VALUE

401.2 DEMONSTRATIVE AND REAL EVIDENCE


If its appearance or other physical characteristics render a fact of consequence more probable, the demonstrative or real
evidence is relevant.
DEMONSTRATIVE EVIDENCE = a way of illustrating or explaining the testimony of a witness. It has no probative value; only
helps to clarify and explain the testimony to the jury.
All evidence from which the trier of fact may derive a relevant firsthand sense impression in contradistinction to the
conventional presentation of oral testimony and the introduction of documentary exhibits. Includes such items as a model,
map, chart, photograph, or other tangible item.
1.

2.

3.

4.

Maps, Models, Drawings, Charts, and Tangible Items: Unless the illustration is essential to an understanding of the
testimony, it is largely cumulative in effect, and admission or exclusion rests within the discretion of the trial judge under
Rule 403. In practice, the use is now almost universally permitted.
The line of demarcation falls between explanation of testimony, which is entirely proper, and the reduction of testimony to
graphic form in such manner as to create the possibility of undue emphasis.
o Example: When an exhibit actually presents in graphic form the testimony of a witness concerning a particular
matter, it may be argued that the memorialization is likely to be given undue weight by the trier of fact, and thus
excludable on the grounds of misleading the jury, under Rule 403. BUT, exercise of discretion in favor of exclusion
should be employed sparingly.
Photographs, Videotapes, Animations: once properly authenticated, admissible if helpful to the understanding of a fact of
consequence in the litigation. Relevant photos may be excluded on the basis of any of the grounds specified in Rule 403,
particularly unfair prejudice and misleading the jury.
Inadmissible: Where either the probative value is slight, OR the fact of consequence sought to be proved is either itself
relatively unimportant in the litigation or undisputed, the likelihood of the photograph being excluded under Rule 403
increases.
Admissible: When the overall importance of the photos in understanding the matter, taken in light of their ability to withstand
any inclination to punish without regard to culpability simply because of the gruesomeness shown in the photos, makes their
admission the norm.
Still Photos: a sufficient foundation is laid by testimony of any person with personal knowledge at a time relevant to the
issues of the subject matter sufficient to support a finding that the photo is a fair and accurate representation of the subject
matter depicted. Photographer is not required and lapse of time between the taking of the photo and the time of consequence
is irrelevant if the conditions have not changed.
Admissible: Changed conditions alone DO NOT render the photo inadmissible, if after the changes are explained since the
photo will not be misleading
Inadmissible: Since the photo makes difficult any separation of illustration from assertion, a change may make the picture so
confusing or misleading as to require exclusion under Rule 403.
Personal Injuries: Photographic evidence should not be excluded merely because it might arouse feelings of horror or
indication in the jry.
May be admitted over objection that the evidence is cumulative in light of other evidence of the injury, such as expert
testimony or displays, or that the injury is not in dispute
All photographs will be admitted where they help the jury to better understand the medical testimony, and thus the
admission of photos showing treatment, such as tractions, is within the discretion of the judge

If the gruesome nature of the photograph is caused by autopsy or emergency room procedure, the photographs may be
excluded by reason of their relatively slight probative value under Rule 403

5.

Criminal Prosecutions: Despite their gruesomeness, and thus arguably prejudicial effect on the jury, generally will be
admitted in the courts discretion where they tend to prove such things as the existence of a crime, cause of death, the number
and location of the wounds, the manner in which they were inflicted, the amount of force used, the willfulness of the act in
question, a persons identity, or to corroborate evidence concerning an unusual cause of death.
However, if the gruesome nature of the photograph(s) is caused by autopsy or emergency room procedures, the
photographs may be excluded by reason of their relatively slight probative value under Rule 403
Even where a defendant stipulates to each of the facts as to which the photograph is relevant, the photograph, including
an autopsy photo, may be admitted on the theory either that prosecution has the right to present every element of its case
OR for the moral weight of the evidence

6.

View by Trier of Fact: Take jury to scene where they can get a relevant first hand impression.
Available in any case, but the general rule is that it is at the discretion of the court to permit a witness to testify at a view
since a court reporter and adequate audio devices would be required in order for the testimony to be admissible.
Judges do not like views, so will have to show why the view would be better than a picture
o NOTE: Jury cannot say anything at all during he view, and thus this carries a risk of a mistrial along with
cost, so judge will likely direct you to show photographs, etc. instead
o Goetz Case: guy shot 4 black guys in subway Judge allowed jury to view subway car in order to determine
the Ds reasonable belief to use deadly force in those circumstances

7.

Experiments: Conducted to determine what happened, and thus an attempt to duplicate what actually happened to prove that
what happened in experiment, is exactly what happened at the scene, so have to show that the experiment was done under the
same circumstances
Generally conducted out of court because you dont want jury to see if your experiment fails; and courtrooms are not
laboratories
Admissible: Need to lay a foundation with a witness who testifies to exactly what happened and an expert to testify as to
the science behind the experiment.
o You can conduct the experiments as many times as you want in order to get the results you want.
o Opposing party does not have to be present when you conduct the experiment.
Similarity Rules: must abide by the similarity rules when replicating an accident, in order to prevent undue prejudice to
the jury

Drug Testing: does not involve any substantial similarity, so not subject to any similarity rules. Different from something that is
illustrative of itself (which is a demonstration), because drug testing is merely evidence to prove that the drug is actually what it is.
Substantial similar conditions are needed
Objections due to unfair prejudice: OC will claim that it is unfairly prejudicial (misleading the jury; unable to contradict) because
the lawyer cannot cross experiment due to his absence during the experiment
Suggestions that OC should be allowed to be present have been explicitly rejected
But then why is the OC often allowed to be present?
o Because we function under the presumption that 97% of the cases settle
NOTE: Have a tactical question here, its not just if the evidence is admissible under the rules, but the issue is how do you use evidence
as a litigation strategy, thereby taking advantage of the evidence.
REAL EVIDENCE = Relevant, first hand evidence in the form of a tangible object, such as the production of an object which
usually, but not always, had a direct or indirect part in the incident, such as a murder weapon, piece of exploding bottle, or article of
clothing and also the exhibition of injured parts of the body.
It provides the trier of fact with an opportunity to draw a relevant first hand sense impression and used to show conformity
with eyewitness descriptions. If seeing is believing, the jury should be permitted to see.
o Direct Evidence = may be direct as when the condition of the object itself is at issue, OR
o Circumstantial Evidence = evidence which affords inferences as to facts of consequence
Admissible: if it is capable of having been used in the event. If it can be tied to the D OR victim. As overall policy, if the fact
of consequence for which the evidence is offered is important in the litigation and if the probative value of the tangible
evidence is high, exclusion should occur only in the most unusual circumstances.

Inadmissible: Relevant real evidence may be excluded on the grounds of unfair prejudice or needless presentation of
cumulative evidence in accordance with Rule 403.

1) Tangible Objects: those involved in an incident of almost every kind have been admitted in recognition of the explanative
value of the sense impressions derived from the object. The fact of consequence need not be in dispute
Must be proved to be genuinely what it purports to be. If probative value depends upon similarity of condition in a
particular respect, that condition must be proved to be unchanged. Changed conditions do not require exclusion where
after the changes are explained, the trier of fact will not be mislead
2) Instrumentalities: If an object is shown to be connected with D, admissibility does not require a showing that such an object
was actually used in the offense charged so long as it is shown that the object is at least suitable for the commission of the
offense.
o Example: bloody knife found at the scene of a murder may be admitted for a purpose other than to show the
defendant committed the crime
Connection with the crime may be circumstantial, such as establishment of the defendants possession of masks,
burglary tools, or quantities of weapons
Connection with the crime may be direct, such as possession of bait money from the robbed bank or positive eyewitness
identification of items employed in committing the crime
3) Physical Injury: most probative evidence available as to a fact of consequence and thus, exhibition is permitted even in the
absence of controversy as to the existence of the injury or its nature and extent.
Even if the injury is particularly shocking, displays is considered proper
Under Rule 403, the jury should be deprived of such evidence only under the most unusual of circumstances
o NOTE: caution should be exercised in permitting the injured person to perform certain acts or be manipulated by a
doctor because of problems of difficulty of cross-examination, and the inducement of an undue emotional response
by the manifestations of pain
RULE 402. RELEVANT EVIDENCE GENERALLY ADMISSIBLE; IRRELEVANT EVIDENCE INADMISSIBLE
All relevant evidence is admissible, except as provided by the Constitution, by Act of Congress, by these rules, or rules prescribed by
the Supreme Court. Evidence which is not relevant is not admissible. If its appearance or other physical characteristics render a fact of
consequence more or less probable, the demonstrative or real evidence is relevant.
Demonstrative evidence: ok when promotes understanding of other evidence
Direct Real Evidence: ok when it is genuinely what it purports itself to be and, if essential to probative value, that its
condition is unchanged.
401.4: Real Evidence: Tangible objects; instrumentalities of crime: relevant real evidence may be excluded on grounds of
unfair prejudice or needless presentation of cumulative evidence in accordance with 403. But, the exclusion of an object of
undoubted relevance and probative value on either ground is unlikely
404, 405, and 407 412 call for exclusion of relevant evidence in various specific situations because of the risk of
misleading; confusion; prejudice is so great that truth would probably be obscured; necessary to encourage socially desirable
conduct.
Rule 104(a) = court must exercise broad discretion in drawing on its own experience in the affairs of mankind in evaluating the
probabilities upon which relevancy depends.
RULE 403. EXCLUSION OF RELEVANT EVIDENCE ON GROUNDS OF PREJUDICE, CONFUSION, OR WASTE OF
TIME
Rule favors the admissibility of relevant evidence. Relevant evidence may be excluded only if its probative value is substantially
outweighed (must be drastic) by the danger of unfair prejudice, confusion of the issues, or misleading the jury, or by
considerations of undue delay, waste of time, or needless presentation of cumulative evidence.

Probative Value: how important the evidence is to establishing an element of the crime or defense it is offered to prove. If
the fact that the evidence is trying to prove is not important then the evidence will be likely excluded.
o Opposing Party has to argue the probative value is substantially outweighed by other factors.
o Exclusion is only used sparingly as an extraordinary remedy.
o Where relevant evidence is subject to exclusion, the trial court should advise counsel of the specific ground for
exclusion so that counsel, if possible, can obviate the objection

Balancing Test Court should consider the importance of the fact of consequence for which the evidence is offered in the context of
the litigation, the strength and length of the chain of inferences necessary to establish the fact of consequence, i.e. the evidences
probative value, the availability of alternative means of proof, whether the fact of consequence for which the evidence is offered is
being disputed, and where appropriate, the potential effectiveness of a limiting instruction (Rule 105). Judge must assume that the
evidence will be believed by the trier of fact.
Applies to evidence being admitted against less than all opposing parties or for a limited purpose such as for its effect upon
the credibility of a witness
Trial Concerns (Disadvantages)
1. Unfair Prejudice: using the evidence that establishes a fact of consequence beyond the necessary standard of proof OR using
evidence that has an undue tendency to suggest decision on an improper basis, commonly either an emotional one, such as
bias, sympathy, hatred, contempt, retribution or horror.
An improper basis also includes consideration of the evidence by the jury for a purpose for which it has not been
admitted in spite of a limiting instruction not to do so. Where a danger of unfair prejudice is perceived, the degree of
likely prejudice must also be considered
o Gruesome Photographs: unaltered photos of the victim at the scene or immediately after the crime will be
allowed no matter how grotesque, however photos of the body after an autopsy will not be allowed. Photos
of accident injuries at the time of the accident are necessary to establish damages and will be allowed as
long as they are not altered.
o Previous Convictions: evidence of previous convictions leads to the Bad Man Inference, which means
that the jury will infer that if he committed that crime he is likely to do it again. Therefore, except in certain
circumstances, previous convictions or evidence of previous crimes are excluded.
o NOTE: prior bad acts can be admitted for such things as impeachment of credibility
2. Confusion: Distracting the jury introduce so much evidence on a marginal issue or a non-issue that the jury will begin to
confuse the issues
3.

Misleading: refers primarily to the possibility of the jury incorrectly evaluating the probative value of a particular item of
evidence, usually be overvaluing, for any reason other than the emotion reaction associated with unfair prejudice.
Evidence of the results of a lie detector test, even where an attempt is made to explain fully the significance of the
results, is likely to be overvalued
Demonstrative evidence such as charts, tests, maps, models, etc. which varies substantially from the fact of
consequence
Sheer amount of time spent upon a question into believing the issue to be of major importance and accordingly into
attaching too much significance to the issue

4.

Waste of Time; Undue Delay; Cumulative Evidence: exclusion of evidence where its probative value is substantially
outweighed by considerations of undue delay, waste or time, or needless presentation of cumulative evidence.
Excluded because an unsuitable amount of time would be consumed in presenting and clarifying the situation;
and/or
Where the evidence being offered is relevant to a fact as to which substantial other evidence has already been
introduced, including evidence bearing on the question of credibility; and/or
Where the evidence itself possesses only minimal probative value, such as evidence admitted as background; and/or
Where evidence is thought by the court to be collateral

*Rule 403 does not enumerate surprise as a ground for exclusion of relevant evidence. While claims of unfair surprise may still be
justifiable despite procedural requirements, enforcement of discovery sanctions or the granting of a continuance are more appropriate
remedies than excluding the evidence on the basis of balancing the harm caused against the incremental probative value*
Offer to Stipulate: In evaluating the incremental probative value of the proffered evidence, the fact that the opponent has offered to
stipulate or is not disputing the proposition for which the evidence is being offered should be considered, but the court will typically
allow the evidence to still be admitted due to concept that the proponent of the evidence need not accept a stipulation, but rather is
entitled to present otherwise admissible evidence.
Example: in case of murder, even if D will stipulate that the victim was shot in the head, the prosecution can still admit the
pictures of the body
Reasoning: the jury wants to hear and see the whole story, not be told by the judge. AND, reasoning lies in the need for
evidence in all its particularity to satisfy the jurors expectations about what the proper proof should be.

Parr: reason for the rule is to permit a party to present to the jury a picture a naked admission might have the effect
to rob the evidence of much of its fair and legitimate weight.

RULE The prosecution or other litigant is entitled to prove its case free from any defendants option to stipulate any evidence away
Old Chief: a criminal defendant may not stipulate or admit his way out of the full evidentiary force of the case as the
government chooses to present it
Multiple purposes: where there are multiple relevant purposes to admit a piece of evidence and stipulating to it doesnt
cover all of these purposes, the evidence wont be excluded
Judges Discretion: The judgment of the trial court in regards to the above listed variables as well as others, to the extent in which
they outweigh or are outweighed by the proper aspects of the evidence, should be disturbed only if the exercise of discretion was
abused.
Trial courts have appeared to use Rule 403 sparingly to exclude relevant evidence and reversal is rare
A finding on the record by the trial court of the result of conducting the balancing of Rule 403 is encouraged but NOT
required
Relevancy Objections
A. Irrelevant Does the evidence actually have the tendency to establish the fact of consequence to the determination of the
action?
B. Immaterial Is the proposition that it actually established a part of the case? What element of the case does the evidence go
toward proving?
A fact might establish what you intend for it to establish, but might not be part of the case.
i. Example: evidence that a minor was a prostitute this is relevant established that she consented. But, this
proposition is immaterial consent is not a defense
RULE 404. CHARACTER EVIDENCE IS NOT ADMISSIBLE TO PROVE CONDUCT
(a) Character Evidence Generally. Evidence of a persons character or a trait of character is not admissible for the purpose of
proving action in conformity therewith on a particular occasion EXCEPT:
1. Character of Accused. Allows an accused to offer evidence of a pertinent trait of his character and allows the
prosecution to rebut such evidence once the accused has done so.
2. Character of the alleged victim. Allows an accused to offer evidence of a pertinent trait of his character and allows
the prosecution to rebut such evidence once the accused has done so OR evidence of a character trait of peacefulness
of the alleged victim offered by the prosecution in a homicidal case to rebut evidence that the alleged victim was the
first aggressor.
a. If evidence of a trait of character of the alleged victim of the crimes is offered by an accused under 404(a)
(2), evidence of the same trait of character of the accused may be offered by the prosecution.
3. Character of Witness. Evidence of the character of a witness, as effecting the witnesss credibility as provided in
Rules 607, 608, and 609.
*Subject to exceptions provided in Rules 413-415 that relate to the character of the defendant when he is charged with offenses of
sexual assault or child molestation or when a civil case is brought against the defendant and to the provisions of Rule 412 relating to
the character of the victim in sex offense cases*
(b) Other Crimes Wrongs, or Acts. Evidence of other crimes, wrongs, or acts is not admissible to prove the character of a person in
order to show action with conformity therewith.
It may, however, be admissible for other purposes, such as proof of motive, opportunity, intent, preparation, plan, knowledge,
identity, or absence of mistake or accident, provided that upon request by the accused, the prosecution in a criminal case shall
provide reasonable notice in advance of trial or during trial if the court excuses pretrial notice on good cause shown, of the
general nature of any such evidence it intends to introduce at trial.
Character = nature of a person, his disposition generally, or his disposition in respect to a particular trait such as peacefulness or
truthfulness.
Reputation = the community estimate of him
Reputation may enter as a method of proving character under Rules 405 and 608.
PP Character evidence is of slight probative value and may be very prejudicial.

It tends to distract the trier of fact from the main question of what actually happened on the particular occasion. It subtly
permits the trier of fact to reward the good man and to punish the bad man because of their respective characters despite what
evidence in the case shows actually happened.

10

COMPETENCY OF LAY WITNESSES


Testimonial risks are associated with evaluating credibility and cross-examination.
TESTIMONIAL RISKS
o Recordation
o Recollection
o Narration or ambiguity
o Sincerity or fabrication
When determining competency, view question most favorably in the light of the witness claiming competency
(1) GENERAL RULE OF COMPETENCY
RULE 601. GENERAL RULE OF COMPETENCY = Every person is competent to be a witness except as otherwise provided in
these rules. However, in civil actions and proceedings, with respect to an element of a claim or defense as to which State law supplies
the rule of decision, the competency of a witness shall be determined by state law.
Eliminates all grounds of witness in-competency with respect to a claim or defense as to which federal law provides the rule of
decision except those specifically recognized. Included in the grounds for incompetency not recognized are age, religious belief,
mental incapacity, color of skin, moral incapacity, conviction of a crime, marital relationship, and connection with the litigation as a
party, attorney, or interested person.
Such matters survive in most instances as avenues of impeachment of the witness
Requirements
(1) Rule 601: Physical and Mental Capacity = the witness had the capacity to accurately perceive, record, and recollect
impressions of facts at the time of the event
(2) Rule 602: Personal Knowledge = The witness in fact did perceive, record, and recollect impressions having any tendency to
establish a fact of consequence in the litigation,
(3) Rule 603: Oath or Affirmation = the witness declares that he will tell the truth, understands the duty to tell the truth, as well
as understands the difference between the truth and a lie or fantasy, and
(4) Rule 604: Narration = the witness possesses the capacity to comprehend questions and express himself understandably,
where necessary with the aid of an interpreter
No other personal qualifications of a witness are required no mental qualification is specified
Reasoning: standards of mental capacity have proven elusive, few witness were actually disqualified, and moreover that a
witness is wholly without mental capacity is difficult to imagine
RULE 601. CAPACITY = Witness has to have the capacity to accurately perceive, record, and recollect impressions of facts at the
time of the event AND the witness did in fact perceive, record, and recollect impressions of facts having any tendency to establish a
fact of consequence in the litigation.
Thus, its not only can he see but was he looking? (Doesnt matter if a witness wears glasses and he didnt have them on
he might not see well but he still has the capacity to see.)
Rule 104(a): competency of a witness is a matter to be decided by the court.
o Courts routinely conduct hearings to determine competency; expert witness testimony may also be received and
psychiatric examination
o Minimum credibility should be evaluated in light of the need for the testimony as well as the other factors in Rule
403.
Reasoning: Though the tribunal is unskilled, and the testimony difficult to weigh, on balance it is still better
to let the evidence come in for what it is worth with cautionary instructions (also subject to crossexamination and extrinsic evidence allowed to be introduced)
TEST OF CAPACITY whether a reasonable juror must believe that the witness is so bereft of his powers of observation,
recordation, recollection, and narration as to be so untrustworthy as a witness as to make his testimony lack relevancy.
Tendency is to resolve doubts as to minimum credibility of the witness in favor of permitting the jury to hear the testimony
and judge the credibility of the witness for itself. Thus, functions as an effect on the weight to be given to testimony,
rather than precluding admissibility.

11

Testimony of a witness passing the test of minimum credibility may nevertheless be excluded under Rule 403; the more
deficient the witnesss capacities the more likely the probative value of the witnesss testimony will be outweighed by trial
concerns enumerated in Rule 403.
Otherwise competent witnesses may be barred from testifying as to a particular matter by privilege

(1) Mental incapacity or immaturity: Testimony of a witness whose mental capacity has been seriously questioned may still be
excluded on the grounds that no reasonable juror could possibly believe that the witness in fact possesses personal knowledge or
understands the difference between the truth and a lie or fantasy and the duty to tell the truth
Neither age nor sanity is a test for competency = if someone is sane or extremely young, but in the judges opinion
understands the oath and has relevant first hand knowledge they will be allowed to testify.
o An adjudication of feeblemindedness does not render a witness incompetent, nor does having spent time in a mental
institution
o The mere fact that a witness has been found incompetent to stand trial does not preclude the witness from testifying
18 USCA 3509(c) = DETERMINATION OF THE COMPETENCY OF A CHILD WITNESS
(1) Effect of Federal Rules of Evidence. Nothing shall be construed to abrogate Rule 601
(2) Presumption. A child is presumed to be competent
(3) Requirement of a Written Motion. A competency exam regarding a child witness may be conducted by the
court only upon written motion and offer of proof of incompetency by a party
(4) Requirement of Compelling Reasons. A competency exam regarding a child may be conducted by the court
only if the court determines, on the record, that compelling reasons exist. A childs age alone IS NOT a
compelling reason
(5) Persons Permitted to be Present. The only person who may be permitted to be present are
(A) The judge;
(B) The attorney for the government;
(C) The attorney for the Defendant
(D) A court reporter; and
(E) Persons whose presence, in the opinion of the court, is necessary to the welfare and well-being
of the kid, including the kids attorney, guardian ad litem, or attendant.
(6) Not Before a Jury. Competency exam regarding a child witness shall be conducted out of the sight and
hearing of a jury
(7) Direct Exam. Normally conducted by the court on the basis of questions submitted by both sides. The court
may permit an attorney, but not a party acting as an attorney pro se to examine a kid directly on
competency if the court is satisfied that the kid will not suffer emotional trauma as a result of the exam
(8) Appropriate Questions. Questions shall be appropriate to the age and development level of the kid, shall not
be related to issues at trial, and shall focus on determining the kids ability to understand and answer simple
questions
(9) Psychological and Psychiatric Examinations. Shall not be ordered without a showing of compelling need.
Generally, a kid is 1-2 is out, kids aged 5+ are competent. Kids who are 3 or 4, judge will determine competency based on a hearing
without the defendant where the judge will ask if the child understands what happens when you lie.
Questions to determine if kid can distinguish truth from lie: If I told you my robe was white, would that be a lie or the truth? and Is
Superman real or just make believe?
(2) Alcohol or Drug Use as Affecting Competency: Habitual use of intoxicants or drug addiction does not by itself make a witness
incompetent to testify.
Even if the witness at the time of the event was intoxicated or under the influence of drugs will still be allowed to testify if
they understand the oath and have relevant first hand knowledge since competency is determined by an assessment of
minimum credibility
(3) Dead Mans Statutes. Proposed, but not accepted by Congress the abolition of incompetency of witness by virtue of state Dead
Mans Statutes in all actions tried in a federal court
Generally provide for the exclusion of an interested witnesss testimony concerning a conversation and/or event which took
place in the decedents or incompetents presence when offered against the latters representative.
o Example: in a car accident case in which the plaintiff died, the defendant driver may not be permitted to testify as to
the facts constituting the accident and/or statements admitting fault allegedly made by the deceased plaintiff prior to
his death.

12

Rationale: where death or incompetency has closed the mouth of one party, the law seeks to effect equality by closing the
mouth of the other and reflect the view that a party will lie when he cannot be directly contradicted and the unrealistic
assumption that jurors will believe everything they hear.
Although motivation behind abolition was due in part that this disqualification leads to more miscarriages of justice than it
prevents, Congress felt that since statutes represent state policy, they should not be overturned in the absence of a compelling
federal interest.
NOTE: NO federal Dead Man Statute. Thus, ONLY applies in either state court which has such statute OR a diversity case
and the federal court is applying a States laws, which has such a statute.
o Florida does have such a statute

(4) Competency is for the Judge and Jury to decide: For the judge, its just a possibility standard can a reasonable juror give ANY
weight to the testimony? If yes, then its an ultimate issue for the jury to determine.
If a ground of general disqualification is known, objection must be made before any testimony is given
If the disqualification is only as to certain matters, an objection need not be raised until the witness is asked about the barred
event or conversation. In practice, such objections are resolved in advance of the testimony by means of a motion in limine
To the extent practicable, proceedings must be conducted without the knowledge of the jury
o A motion to strike will not lie if the ground of incompetency was known before the testimony was given
Where the ground is based upon lack of minimum credibility, the objection will normally be made after the proponent of the
witness has attempted to law a sufficient foundation for personal knowledge. (sometimes a voir dire exam of the witness is
conducted prior to the court ruling)
(2) THE REQUIREMENT OF PERSONAL KNOWLEDGE
RULE 602. LACK OF PERSONAL KNOWLEDGE. A witness may not testify to a matter unless evidence is introduced sufficient
to support a finding that the witness has personal knowledge of the matter
Admissible testimony is limited to matters of which the witness has acquired personal knowledge through any of his own
senses
o Evidence to prove personal knowledge may, but need not, consist of the testimony of the witnesss own testimony.
o Can also include corroborating evidence
o Absolute certainty either of observation or of recollection is not required to establish personal knowledge. All that is
required is an opportunity to observe AND a belief that what is related depicts the perception.
Thus, phrases such as I think, I believe, or my best recollection is NOT per se inadmissible
o Courts generally will allow cautious answers such as to the best of my memory, even if it may mean that his
perception was careless or inaccurate or that his once reasonable clear picture has faded
Rule 104(b): Requirement is one of conditional relevancy, and thus the proponent of the evidence must introduce the sufficient
evidence for the judge to support a finding by a reasonable jury that it is more probably true than not true that the witness had the
capacity and opportunity to observe through his senses and record a relevant sense impression, did in fact observe, record, and can
now recollect the relevant sense impression, and can comprehend questions and narrate the relevant sense impression of the matter
related.
The court may not refuse to permit a witness to testify because the court believes the witness to be obviously mistake or
falsifying.Test is not improbability, but impossibility.
BOOT-STRAPPING With respect to establishing personal knowledge of a hearsay declarant, the content of the statement offered
and objected to, may itself be considered in ruling on admissibility, though not yet admitted into evidence. Thus, court able to
consider the content of the statement to be admitted when determining whether sufficient evidence to support a finding of personal
knowledge
Allows the court to consider the content of the statement when it makes sense to do so, while still preserving the overall issue
as one of conditional relevancy
RELATION TO HEARSAY = A witness testifying to an extrajudicial statement, which is defined as not hearsay or is admissible
under an exception to the hearsay rule
Rule 801(d), 803, 804, 807: need not have personal knowledge of the matter related in the statement, but must possess
personal knowledge as to the fact of the statement itself, as to the matter related, and to the declarant himself.
Personal knowledge of the declarant also not required for statements admissible as an admission of a party opponent

13

RELATION TO EXPERT WITNESS = Rule 703 allows an expert witness to express opinions based on facts of which he does not
have personal knowledge. But, a lay person under Rule 701 is required to have personal knowledge of the facts upon which the
opinion is based.
Objections: (closely tied to Hearsay due to lack of personal knowledge)
Lack of Personal Knowledge = it a witness testifying to an event acknowledges being in a distinct location at the exact time
Hearsay = if a witness states that he was told of the event by another person
(3) OATH OR AFFIRMATION
RULE 603. OATH OR AFFIRMATION: the witness has to declare that he will tell the truth, by oath or affirmation, understands his
duty to tell the truth, and understands the difference between the truth and a lie or fantast
An oath contains a formal calling upon G-d
An affirmation is merely a statement that one will tell the truth
o A witness who will not take an oath or affirmation is incompetent. You cannot make up your own oath
Reasoning: (1) Affect the conscience of the witness to impress his mind with the duty to speak the truth and (2) if the witness willfully
falsifies his testimony, he may then be punished for perjury.
NOTE: NO requirement for moral capacity in the sense of requiring that the witness actually possess a sense of responsibility to speak
the truth and no requirement of a belief of G-d. Policy is to merely add a stimulus to truthfulness wherever such a stimulus is
feasible
Rule 104(a): Whether a witness has sufficiently declared by oath or affirmation is a determination made by the court
(4) INTERPRETERS
RULE 604. INTERPRETERS. Subject to the provisions of these rules relating to qualification as an expert and the administration of
an oath or affirmation to make a true, exact translation.
Court, in its discretion, may
A. appoint an interpreter of its own selection when witness/defendant speaks no English or not well enough to understand
and communicate with counsel and those that use sign language
B. Fix reasonable compensation
a. Criminal: fees paid either out of funds provided by law or by the government
b. Civil: paid out of funds provided by law or paid by one of more of the parties at the discretion of the court or
taxed ultimately as costs
Rule 702: Interpreter must meet the qualifications for an expert witness, which requires in the case of an interpreter that by reason of
knowledge, skill, experience, training, or education, the interpreter is capable of providing a true translation.
(5) COMPETENCY OF JUDGE AS WITNESS
RULE 605. COMPETENCY OF JUDGE AS WITNESS. The judge at the trial MAY NOT testify in that trial as a witness and NO
objection need be made in order to preserve the point.
Can testify in court during a case on appeal
A. C/L Rule = judge competent to testify in a trial over which he was presiding, but could in his discretion decline to do so
B. Previous Rule = Judge was disqualified from testifying to material disputed facts but free to testify with respect to matters
merely formal and undisputed
Reasoning: (1) role of witness is plainly destructive of the courts image of impartiality; (2) create problems concerning jury
overvaluing the testimony; (3) ruling on objections to his testimony; (4) the conduct of cross; (5) virtual impossibility of impeachment,
and (6) continuation of the trial before the same judge
(6) COMPETENCY OF JUROR AS WITNESS
RULE 606. COMPETENCY OF JUROR AS WITNESS

14

(a) At the Trial. A member of the jury may not testify as a witness before that jury in the trial of the case in which the juror is
sitting. If the juror is called so to testify, the opposing party shall be afforded an opportunity to object out of the presence of
the jury
Rule does not prevent the questioning of jurors by the judge or counsel as part of jury selection.
In the unlikely event a juror is called to testify, opposing party must be afforded the opportunity to object out of the
presence of the jury
(b) Competency of Juror to Attack Validity of Verdict or Indictment. Jurors are prohibited from attacking their verdict or
indictment by testifying directly or indirectly by means of affidavits or statements as to any matter or statement occurring
during the course of the jurys deliberations or to the effect of anything upon his or any other jurors mind or emotions as
influencing him to assent to or dissent from the verdict or indictment or concerning his mental processes in connection
therewith.
A juror cannot testify about anything that took place while in the jury room, meaning jury deliberations.
Improper intrinsic influences on a jurys verdict are not competent to impeach the verdict.
Thus, juror may not attack his verdict on the grounds of a quotient verdict, that a fellow juror was
intoxicated, that a jury fell asleep, that jury instructions were misinterpreted, that the jury drew adverse
inferences from the defendants failure to testify, or on the basis of jury speculation as to insurance
Also excluded: internal discussions, arguments, mental and emotional reactions, votes, or statements, even
a statement purporting to relate specific factual information which was incorrect
Does not preclude inquiry to confirm the verdict (polling the jury), but does preclude testimony to prove that the
jurors were operating under a misunderstanding about the consequences of the result that they had agreed upon, or
that the jury misapplied or misunderstood a jury instruction is precluded.
Ex: cannot come back two weeks later and testify that they felt pressure to vote that way
(c) BUT, a juror may attack the verdict by testifying to
1) Whether any extraneous prejudicial information was improperly brought to the jurys attention (a radio newscast or a
newspaper account);
2) Whether any outside influence was improperly brought to bear upon any juror, (a threat to the safety of a member of his
family);
3) Whether there was a mistake in entering the verdict onto the verdict form (either checked wrong box or jury failed to
make a unanimous decision); OR
4) An unauthorized view, experiment, or jury investigation that had transpired
5) A juror can testify about corruption or coercion
6) Dishonest answers by a juror on voir dire
2 levels of analysis of error used to warrant a reversal
A. Harmless Error Analysis = if extraneous information gets in or juror conducted own investigation or experiment, and judge
determines that there was no prejudice on the jury.
B. Reversible Error = Test: if a reasonable possibility that such influence or information affected the verdict, such as a bribe or
juror lied on voir dire requires a new trial
a. NOTE: A preliminary showing of reversible misconduct that would warrant reversal for a hearing will give rise to a
presumption of prejudice placing the burden of proof on the victorious party to show harmlessness. While the juror
may testify describing the extraneous influence or information, the juror may not testify about its impact.
i. TEST = Judge should not decide the question ex parte, but rather should determine the circumstances, the
impact thereof upon the juror, and whether or not it was prejudicial, in a hearing with all interested partied
permitted to participate.
Policy Reasons: (a) discourages harassment of jurors by losing parties eager to have the verdict set aside; (b) encourages free and open
discussion among jurors; (c) reduces incentives for jury tampering; (d) promotes verdict finality; (e) maintains the viability of the jury
as a judicial decision-making body
(G) COMPETENCY OF LAWYER FOR A PARTY AS WITNESS AT TRIAL
Rule 601 confirms that a lawyer for a party is not as such incompetent to testify. But, judge has wide discretion to refuse to permit a
lawyer from testifying in favor of his client
i.e. where the lawyers testimony forms a link in the chain of custody of an exhibit or the lawyers
testimony is intrinsic evidence of a proper inconsistent statement.
Also, prevent such testimony where other sources of evidence as to the fact of consequence are available or
where the necessity for testimony by the lawyer could have been avoided

15

Lawyers can testify as witnesses in trials they are involved in. If it is a minor matter then the lawyer will be allowed to testify. If it is a
major matter then the lawyer may be compelled to resign and testify. A lawyer can be compelled to testify about evidence in his
possession and what happened to it. But, this isnt an evidence rule guided by the ABA rules
ABA Rule 3.7: Lawyer as a Witness
(A) A lawyer cant take a case where he knows he has to testify unless
1. Testimony relates to an uncontested issue;
2. Relates to legal services rendered; or
3. Where disqualifying him as the lawyer would have a substantial hardship on the client
(B) A lawyer may act as an advocate in a trial in which another lawyer in his firm is likely to be called as a witness unless
precluded by a conflict of interest
CONFLICT/IMPROPER REPRESENTATION: there is likely to be a substantial conflict between the testimony of the client and
that of the lawyer or a member of the firm
Primarily the responsibility of the lawyer to determine if there is a conflict in the representation
Problem can arise whether called as a witness on behalf of the client or by opposing counsel
A lawyer may act as advocate in a trial in which another lawyer in the lawyers firm is likely to be called as a witness
unless precluded from doing so by Rule 1.7 (Conflict of Interest, Generally) or Rule 1.9 (Conflict of Interest, Former
Client).
Two situations where they may happen
1. Taking possession of physical items that will be offered at trial
Do not want to have to testify as to the condition of the item or chain of custody
2. Taking statements if you take a statement you may be required to testify if its non-collateral and no one
else was present. Make sure someone else is there. Someone else from your firm can testify.
May have to testify to an issue such as inconsistent statement
APA Rule 1.7. Conflict of Interest General Rule
(a) A lawyer shall not represent a client if the representation of that client will be directly adverse to another client unless
1. The lawyer reasonably believes that there will be no adverse effect; AND
2. Each client consents
(b) A lawyer shall not represent a client if the representation may be materially limited by his responsibilities to another client or
to a third party, or by his own interests, unless:
1. He reasonably believes the representation will not be adversely affected; AND
2. Clients consented
APA Rule 1.9. Conflict of Interest: Former Client
A lawyer who has formerly represented a client in a matter shall not thereafter:
(a) Represent another person in the same or substantially related matter in which that persons interests are amterially
adverse to the interests of the former client unless the former client consents after consultation; OR
(b) Use information relating to the representation to the disadvantage of the former client except as Rule 1.6 would permit
with respect to a client or when the information has become generally known
Direct Examination of a Witness: When a lawyer calls a witness, that lawyers questioning of that witness is called direct
examination.
RULE 611. THE MODE AND ORDER OF INTERROGATION
(a) Court controls the mode and order of interrogation to:
(1) Make it effective to ascertain the truth;
(2) Avoid waste of time;
(3) Protect witnesses from harassment and embarrassment
(b) Scope of Cross-Examination. Cross-Examination should be limited to the subject matter of the direct examination and
matters affecting the credibility of the witness. The court may, in the exercise of discretion, permit inquiry into additional
matters as if on direct examination.
(c) Leading Questions. Generally leading questions are not allowed on Direct Examination, but are permissible on crossexamination.
Non-Leading Questions: Questions that do not suggest the answer, but keep the witness within the bounds of
relevancy and the exclusionary rules of evidence and serve to afford the opponent an opportunity to object to

16

inadmissible evidence before it is given. Questions can be general or specific, and should use both to elicit testimony
and allow the witness to tell her story.
NOTE: using the term whether or not or other form of alternative does not of itself keep a question from being
leading
o Leading questions suggest the answer and are most likely answered yes or no. The vice of such a question
lies in substituting the suggestions of counsel for the actual testimony of a witness
o ISSUE: whether an ordinary man would get the impression that the questioner desired one answer rather
than another, which can be determined by the extent of the particularity of the question.
Leading: When the question describes an incident in detail and asks if the incident happened, the natural inference is
that the questioner wants an affirmative answer.
Non-Leading: When a question is neutral or sufficiently balanced, such as at what time did this occur?

EXCEPTIONS: when leading questions are allowed on direct exam


A. When necessary to develop the witnesss testimony; AND
B. When directed to a witness regarded as hostile as a matter of law
Necessary to Develop Situations
1. When the matter is preliminary or inconsequential, which means facts are not in dispute.
2. Hostile or unwilling witness=
a. Hostile in Fact:
A witness who demonstrates defiance by giving damaging testimony, or by answering reluctantly,
obstinately, or belligerently. NOTE: Mere giving of damaging answers in the absence of defiance
does not permit counsel to lead.
Also, witness who is unwilling, i.e. a witness shows himself by demeanor or testimony to be
reticent, deceptive, hesitant, evasive, or uncooperative
b. Hostile in Law: Witnesses, such as an adverse party, who are automatically considered hostile so that
leading questions are permitted without a prior in-court showing that leading questions are necessary for
the orderly development of the testimony.
Definition: a hostile witness, an adverse party or a witness identified with an adverse party, which
is determined at the time of his examination.
EX: mother of defendant
3. A child or adult witness with communication problems, mental or physical
4. A witness whose recollection is exhausted
a. Forgetful witness, when the witness answers I cannot recall or I cannot remember to a non critical
issue you are allowed to follow up with a leading question. However, if the witness gives you the wrong
answer the court has discretion to allow you to follow with a leading question.
5. When the witness freezes because he is frightened or upset and you need to suggest the answer for him to testify at
all (frequently a child victim of sexual assault)
6. To impeach your own witness as provided in Rule 607.
7. When a witness is unresponsive or shows a lack of understanding
Refreshing Recollection: under appropriate circumstances, witness may be refreshed by means of either a leading question of the
showing of a writing.
Circumstances = Only if the witness testifies that his recollection is exhausted and that he cannot recall the matter forming
the subject of the inquiry
Must be conducted as provided in Rule 103(c) so as to prevent inadmissible evidence from being suggested to the jury by
any means, such as asking question in the hearing of the jury.
o Whether the matter under consideration is of significance in the litigation, such as, when the government seeks to
refresh the recollection of a witness it has called as to a statement bearing directly on the guilt of the defendant, it is
similarly improper to employ a leading question incorporating the content of the alleged statement before the jury.
Proper procedure: employ a currently existing writing or other item or reduce the oral statement to writing
and proceed as set out below
Only if the witness states that his recollection has in fact been refreshed may the initial non-leading
question be repeated before the jury.
o Where the witness cannot recall a preliminary or other relatively unimportant matter, a leading question in open
court incorporating the answer may be employed. Similarly, leading questions, which suggest that something else
occurred without directly stating what the even was, are also proper to refresh recollection

17

RULE 612. WRITING USED TO REFRESH MEMORY


When a witness responds to a question with I dont know and it is either a critical issue or it is not a critical issue and the follow up
leading question does not refresh their recollection, you have to refresh with a written document. The witness cannot read the
document out loud, he can only read it to himself then put it down and cannot testify as to what was written in the document.
The writing itself need not be admissible in evidence; all that is required is for the witness to state the facts from his own
present recollection after inspecting it.
Original Writing Rule [Rule 1002] DOES NOT apply = the document can be anything, however you must show it to the
opposing attorney who may use it for cross examination, and opposing counsel can enter the document into evidence for the
sake of credibility if he wishes.
You can use a document to refresh a witnesss recollection before testifying, but the adverse party can request the document,
inspect it, to cross-examine the witness thereon, and to introduce in evidence those portions which relate to the testimony of
the witness.
o If a writing is not produced, court shall make any order justice requires, except that in criminal cases when the
prosecution elects not to comply, the order shall be one striking the testimony or, if the court determines that the
interests of justice so require, declare a mistrial.
If you use a privileged document you might be waiving the privilege so it is better not to show them the document, but
instead prep orally.
If it is claimed that the writing contains matters not related to the subject matter of the testimony, the court shall examine the
writing in camera, excise any portions not so related, and order delivery of the remainder of the party entitled thereto.
RULE 106. COMPLETENESS & FAIRNESS
NOTE: Judge does not need to intervene, unless the opponent objects. No objection = waiver.
A. Remainder of or Related Writings and Recorded Statements Employed at TIME OF INTRODUCTION. When a writing or
recorded statement or part thereof is introduced by a party, an adverse party may require the introduction at that time of any other part
or any other writing or recorded statement which ought in fairness to be considered contemporaneously with it.
Recognition is thus given to be a principle of Completeness: a writing or recorded statement may be so related to another that
in fairness both should be considered together without regard to whether the related writing or recorded statement is or is not
contained in the same document.
PP: (1) avoids the danger of mistaken or misleading first impression when matters are taken out of context; (2) avoids the
inadequate remedy or requiring the adverse party to wait until a later point in the trial to repair his case
Apply with equal force to writings and recorded statements introduced for a limited purpose such as impeachment
Additional part of the writing or recorded statement must relate to the same subject matter and tend to deny, explain, modify,
qualify, counteract, repel, disapprove or otherwise shed light on the writing or recorded statement already received
o Such writing may be admitted only if otherwise admissible
Whether otherwise inadmissible evidence should be admitted depends upon whether its probative value in
such regard is substantially outweighed by dangers of unfair prejudice, confusion of the issues, misleading
the jury, or waste of time
NOTE: ONLY applies to written or recorded statements; NOT applicable to oral statements
B. Remainder of Oral Statements, Writing or Recorded Statement Employed on Next Examination
Counsel may eschew Rule 106 and develop the matter on cross-examination or as part of his own case.
The remainder of oral statements and related oral statements may be introduced by an opposing party on his next examination
of the same witness, whether cross or redirect.
Principle of Completeness does not give an adverse party an unqualified right to introduce an omitted part of a conversation
or related conversation otherwise inadmissible merely on the ground that the opponent has opened the door. This is where
the arguments on this Rule occur; what does this mean?
o Under unusual circumstances, the court may require the proponent to introduce contemporaneously other parts of
oral conversations pursuant to the general authority of the court to control the mode and order of interrogating
witnesses and presentation of evidence.
RULE 615. EXCLUSION OF WITNESSES. THE RULE. At the request of a party, the court shall order witnesses excluded
so that they cannot hear the testimony of other witnesses, and it may make the order of its own motion. But, does not authorize
the court to exclude
(1) Natural person who is a party; or

18

(2) A non-natural partys legal representative such as a business or the government


(3) A person whose presence is shown by a party to be essential to the presentation of a partys cause such as support
people, i.e. the mother of a child witness who is also testifying and expert witnesses
(4) Persons authorized by statute to be present, such as family members of the victim who are going to testify at the
sentencing hearing.
Amended to include victims of a crime, unless the court determines that the testimony by the victim would
be materially affected if the victim heard other testimony
Also, exclusion and separation does not extend to rebuttal witnesses or witnesses called to impeach
credibility.
-

The purpose of this rule is to keep a witness from tailoring his testimony to fill in gaps in the testimony of some other witnesses
In some courts, the rule means automatic sequestration and segregation of witnesses while in others you need a separate order,
but a lawyer can still discuss with a future witness the testimony of prior witnesses or even read the transcript to the witness
himself as long as the witness does not read it

Remedies for violation of the rule:


(1) Declare a mistrial,
(2) Permit cross examination concerning the violation
(3) Refuse to allow the witness to testify,
(4) Instruct the jury to weigh the credibility of the witness in light of the witnesss presence in court or discussions with another
witness, or
(5) Hold the witness in contempt
a. NOTE: thrust of judicial opinion absent compelling circumstances is against either declaring a mistrial or disqualifying
the witness
Separation & Sequestration = the court may take further measures of separation designed to prevent communication between
witnesses, such as ordering them to remain physically apart, ordering them not to discuss the case with one another, and ordering the
witnesses not to read a transcript of the trial testimony of another witness
If the judge means exclusion and separation, can still get the transcript and discuss it with your witness, they just cant read it
themselves.

19

HEARSAY
Overview Rule 802 = Hearsay is not permitted unless its one of the exceptions. In-court statement means in court, in presence of
the trier of fact, and under oath. Allows observation of demeanor by jury, under oath, and subject to cross-examination
- Absent an exception to the rule against hearsay, a hearsay statement is excluded on the ground that the trier of fact is not in a
position to assess the accuracy of the statement, and with it the weight to be assigned to the matter asserted as true in the outof-court statement: the hearsay statement was not made by the witness under oath, in the presence of the trier of fact
observing the witnesss demeanor, subject to contemporaneous cross-examination
- Can testify to what animals do or say parrot or drug-sniffing dog b/c animals arent people animals do not lie. This is
the risk we want to limit risk of sincerity
- Out-of-court statements that either not hearsay pursuant to Rules 801(a)-(c) or are exempt from the bar of the rule against
hearsay through definition as not hearsay, pursuant to Rules 801(d)(1) and (2), or fall within a hearsay exception, pursuant to
Rules 803, 904, and 807, to be admitted into evidence, must still meet other requirements for admissibility, such as relevance,
authenticity, and when the contents of a document are sought, the Original Writing Rule, Rule 1002.
- Rule 103(a) = While hearsay is not admissible, it is nevertheless incumbent upon the party opposing the introduction of an
inadmissible hearsay statement to properly object. In the absence of such an objection, the jury may consider hearsay for
whatever value it may have; such evidence is to be given its natural probative effect as if it were in law admissible.
Three possible solutions to hearsay besides exclusion:
(1) Abolish the rule against hearsay and admit all hearsay
Effect would not be automatically to abolish the giving of testimony under ideal conditions; if the declarant were
available, compliance with the ideal conditions would be optional with either party.
Thus, the proponent could call the declarant as a witness as a form of presentation more impressive than his
hearsay statement or the opponent could call the declarant to be cross-examined upon his statement.
If the declarant were unavailable, a rule of free admissibility would make no distinctions in terms of degrees of
noncompliance with the ideal conditions and would exact no quid pro quo in the form of assurances of
trustworthiness.
(2) Admit hearsay possessing sufficient probative force, but with procedural safeguards
Abandonment of a class exceptions in favor of individual treatment has been impressively advocated for
Admissibility would be determined by weighing the probative force of the evidence against the possibility of
prejudice, waste of time, and the availability of more satisfactory evidence.
Procedural Safeguards would consist of notice of intention to use hearsay, free comment by the judge on the weight
of the evidence, and a greater measure of authority in both trial and appellate judges to deal with evidence on the
basis of weight
(3) Revise the present system of class exceptions
Approach currently is that of the c/l, i.e. a general rule excluding hearsay, with exceptions under which evidence is
not required to be excluded.
Plan submitted as calculated to encourage growth and development in this area of law, while conserving the values
and experience of the past as a guide to the future.
ALTERNATIVE DEFINITION OF HEARSAY = Hearsay is a statement offered in evidence, other than one made by the declarant
while testifying at the trial or hearing, to the extent relevance depends upon (1) the truth of the matter asserted or (2) the declarants
belief in the truth or falsity of the matter asserted.
Rule 801. DEFINITION OF HEARSAY
(a) Statement =
1. An oral or written assertion (either directly expressed or matters implicitly being asserted) OR
2. Nonverbal conduct, if it is intended to be an assertion (shaking head, pointing, etc.).
Regardless of the form, the question to ask is what was the person actually intending to convey.
(b) Declarant = the person who made the statement.
(c) Hearsay = out of court statement not subject to oath, demeanor, or cross examination which is offered to prove the truth of
the matter asserted.
Its only hearsay when we believe the out of court declarant believes what he said OR what the out of
court declarant said has to be true in order for the testimony to be relevant.
Documentary evidence as well as oral assertions of a witness, whether or not recorded, may fall within the
Hearsay Definition.

20

When a statement offered to prove the truth of the matter asserted is made other than by the declarant while testifying at the trial
or hearing, the statement is hearsay without regard to whether or not the out of court decalarant is available to testify or actually
testifies at the trial or hearing at which the out of court statement is offered. Thus, the definition of hearsay in Rule 801(a)-(c) applies
to all statements not made at the trial or hearing, and thus not made subject to cross examination before the trier of fact.
-

If a statement is relevant not to prove the fact of the matter asserted, but to prove that the statement was actually said, its
NOT hearsay
o NOTE: This is the basic distinction between what is hearsay and what is NOT hearsay

4 Hearsay Risks Qualities to be Considered in Evaluating Witnesss Testimony


1. Perception witness has capacity to observe the events and did in fact observe the events through one of the senses
2. Recordation and Recollection witness remembers the event; risk of memory
3. Narration witness expresses what happened; risk of ambiguity
4. Sincerity witness isnt making it up or lying; risk of fabrication
Each of the 4 risks associated with evaluating the accuracy of a witness in-court testimony are present as well when an out-of-court
statement is offered to prove the truth of the matter asserted. Thus, the trier of fact must evaluate the risks when determining the
trustworthiness of the hearsay statement. When the statement offered for the truth of the matter asserted was made out of court, the
trier of fact, where ascertaining inaccuracies, does not have the benefit of having the declarant before it, under oath, and subject to
cross. Hearsay is thus excluded.
In court statements allow for:
A. Oath or affirmation
B. Demeanor of witness can be observed by jury. Traditionally been believed to furnish trier and opponent with valuable clues
and the willingness to falsify may become more difficult in the presence of the person against whom directed.
C. Cross-examination. Today, basis of hearsay rule tends to center on this requirement based on the belief that cross is effective
in exposing imperfections of perception, memory, and narration.
Trier of fact must use his own capacity for accurate observation and his ability to interpret what is happening in his presence and must
determine to what extent he can rely upon the witnesss recollection, sincerity, memory, and perception.
Questions to determine if there is a hearsay risk:
1. Does declarant have the belief?
- Narration: Did declarant understand what he was saying? (doesnt understand English) Declarant intended to say something
other than what he actually said
- Sincerity: is declarant lying?
2. Does the belief reflect reality?
- Recordation & Recollection: Is the declarant correctly remembering the incident?
- Perception: Was the declarant physically capable of perceiving the thing about which he made the statement?
Thus, a statement is hearsay if it requires a yes answer to Question 1 OR Question 2 (offered for the truth of the matter
asserted going into declarants head, and thus subject to 4-hearsay risks)
**If either belief risks or 4 hearsay risks are present, then the statement is hearsay. NO NEED to assess the magnitude of the
hearsay risks.**
- Magnitude of hearsay risks bears solely upon whether the hearsay statement is felt sufficiently trustworthy to be
admitted pursuant to an exception to the rule against hearsay
If the statement is being offered purely to prove that the statement was actually said, or the statement has legal significance
regardless of the declarants belief or reality of the situation, it is not hearsay
Example 1: A-witness hears declarant say that he was shot while in the hospital. Attorney wants to bring in declarants statement to
prove that he was not unconscious after the accident. This statement isnt being used prove the fact asserted (that he was shot), just
that he was conscious. Therefore, the attorney doesnt have to show that the declarant really believed he was shot or that this belief
reflects reality in order for it to be relevant, and thus the statement is NOT hearsay.
Example 2: Declarant writes a note to his wife that says he would be shot if he didnt help robbers rob a bank. Declarants note is
being offered to prove not that the robbers would have actually shot him, but that he believed he would be shot if he didnt assist.
Therefore, attorney needs a yes answer to Question 1, that declarant believed the utterance to be true, but not Question 2 since it

21

doesnt matter if the robbers actually would have shot him. According to Graham, this WOULD be hearsay see circumstantial
utterances to show state of mind.
A. Prior Statements
When the out of court declarant does in fact also testify at trial, cross or direct and redirect exam, at that time, provides an opportunity
for the party opposing the truth of the out of court statement to explore the statement before the trier of fact. Nevertheless, general
admissibility of prior statements of in-court witnesses is NOT provided for ! Individual Exceptions, which limit this definition of
NOT-Hearsay are made in Rules 801(d)(1), 801(d)(1)(A), (d)(1)(B), and (d)(1)(C).
B. Irrelevance
A) Whether the statement was self-serving or disserving at the time of being either made or offered
B) Whether the statement is being offered as direct or circumstantial evidence
C. Presence or Absence of the party against whom an extrajudicial statement is offered has no bearing upon either its status as
hearsay or its admissibility
- Occasionally, whether the party against whom a statement is offered was present when the statement was made has a bearing
upon whether the statement is hearsay under 801(a)-(c) or NOT hearsay under 801(d)(2)
o An oral statement offered to show notice cannot have been effective as such unless it was made in the presence of
the person sought to be charged with notice.
o The presence of a party is essential if it is claimed that she admitted the truth of an oral statement by failing to deny
it.
D. Objection under Rule 103
- A party who does not make a timely objection cannot complain of the admission of hearsay.
- The question remains, however, of the weight and probative value of hearsay so admitted. The almost infinite variety of
hearsay precludes any answer except that hearsay will be considered and given its natural probative effect
E. Nonverbal Conduct Intended as an Assertion
- Nonverbal conduct may, on occasion, clearly be equivalent of an assertive statement, that is, done for the purpose of
deliberate communication, and thus classified as hearsay under 801(a)(2).
o Example: Nodding yes or no, pointing to identify the picture of the perpetrator in a mug shot book, pointing out
the perpetrator in a lineup, and the sign language of the hearing impaired are plainly assertions as are spoken words.
Also, a videotape of the injured Plaintiff recreating the accident which caused his injuries
F. Nonverbal Conduct NOT Intended as an Assertion
- Nonverbal conduct not intended as an assertion is NOT hearsay under Rule 801(a)(2).
- When a person acts without intending to communicate a belief, his veracity is not involved.
- There is frequently a guarantee of the trustworthiness of the inference to be drawn because the actor has based his actions on
the correctness of his belief.
o Example: people walking around with open umbrellas used to prove it is raining is not hearsay because the people
are not intending any assertion, [but if you ask one of them how is the weather and they answer it is raining then
that is hearsay.]
Reasoning: The intent to assert is absent and thus the danger believed to be inherent in hearsay, with respect
to sincerity, is absent. The objection still remains regarding the accuracy of the actors perception and
recollection are untested by cross as to the possibility of an honest mistake. However, these risks are more
sensible factors to be used in evaluating weight and credibility rather than grounds for exclusion. Practical
necessity also supports treating nonverbal assertive conduct as falling outside the definition of hearsay
o Example: car stopped at a traffic light offered for the inference that the light was red
Reasoning: treatment of such conduct as hearsay would too often exclude highly trustworthy and probative
evidence
Resort to the residual hearsay exception of Rule 807 is an inappropriate response to a frequently
recurring situation
- The Court must be satisfied of the probative value of the proffered proof in light of trial concerns.
o Thus, the inference of belief drawn from the nonassertive conduct and/or the inference drawn from such belief when
offered to prove the truth of that fact impliedly being asserted may be too ambiguous to warrant submitting the
evidence to the jury; its probative value may be so slight in comparison with the possibility of confusing and
misleading the trier of fact that exclusion pursuant to Rule 403 is proper.

22

Example: If that person (see above) opening an umbrella was known to be both exiting a store that sells
umbrellas and superstitious concerning the opening of umbrellas indoors, ambiguity associated with the
offering of such evidence to establish that it was raining at the time would certainly be enhanced.
Accordingly, whether exclusion is warranted under Rule 403 would depend upon examination of all
relevant circumstances
If evidence of conduct is offered on the theory that it is not intended as an assertion, and hence NOT hearsay, the burden of
showing that an assertion was intended is on the party objecting to the evidence. The question of intention to assert is one for
the court, under Rule 104(a).
o Example: Even if the person intended to make an assertion, such as punching someone who insulted your
companion, the persons conduct may itself be relevant to establish the safe fact of consequence in the litigation and,
if so, is NOT hearsay for such purpose.
Nonverbal conduct not intended as an assertion, and thus NOT hearsay under Rule 801(a)(2) is frequently, but incorrectly,
treated as an admission of a party opponent.
o Example: flight, failure to speak or act, and the fabrication, destruction, or suppression of evidence. Such evidence
may be excluded upon application of Rule 403.

G. Oral or Written Conduct NOT Intended as an Assertion


- Considerations present with respect to nonverbal nonassertive conduct support the position taken in Rule 801(a)(1) that oral
or written conduct not intended as an assertion is NOT hearsay.
o Examples: screams of pain, singing, outbursts of laughter, singing a song or uttering or writing an expletive. But this
is very, very unusual since any of this conduct may be intended as an assertion.
H. Statements Offered Other Than to Prove the Truth of the Matter Asserted are NOT Hearsay
- Hearsay does not encompass all extrajudicial statements, but only those offered for the purpose of proving the truth of
matters asserted in the statement according to Rule 801(c). Therefore, when the mere making of the statement is the relevant
fact, i.e. tends to establish a fact of consequence, under Rule 401, hearsay is not involved. Such statements are likely said to
be offered solely for the fact said and not for the truth of the matter asserted, i.e., the truth of their contents.
1.

Verbal Act The statement itself, the verbal act, has independent legal significance or gives rise to legal
consequences, sometimes called an operative act. Statement itself is relevant because of the fact that it was said.
o Examples: Testimony by an agent as to a statement by the principal granting him authority to act as an
agent is NOT hearsay. Also, statements constituting contracts, canceling of an insurance policy, constituting
an anticipatory breach, constituting a crime, directions issued by a cop directing traffic, a stop sign at an
intersection, statements offered as evidence of defamation, evidence offered that a statement has been made
as part of a fraudulent scheme and is false, and statements offered to place in context other statements
otherwise admissible made in a conversation.

2.

Characterizing Act Statements comprising operative acts are assertions, which relate to and characterize a
particular act.
o Examples: Oral statements that accompany and explain an ambiguous legal document
Independent act of giving the bank a check for $100 to deposit. You can characterize the act of
giving a bank a $100 check by telling them which account to deposit the money into and that
statement of which account to deposit the money into is NOT hearsay; the fact that it occurred is
relevant.
Where an instrument designating the executives wife as his beneficiary was unclear as to whether
she was to be the beneficiary of his insurance policy or of a sic months gratuity payment, oral
statements accompanying delivery of the instrument resolving the ambiguity were NOT hearsay

3.

Effect on Listener Statements made by one person, which later become known to another offered as a
circumstance under which the latter acted and as bearing upon his conduct. Also, statement being offered for the
purpose of showing the probable state of mind of the listener include being placed on notice or having knowledge.
o Cop can say he went to a certain location because he received a radio call to do so.
BUT, cop cannot become specific by repeating definite complaints of a particular crime by the
accused, this is so likely to be misused by the jury as evidence of the fact asserted that the content
of the statement, absent special circumstances enhancing probative value, such as the cop shot a
person leaving the bank after being advised that a clerk was shot, should be excluded on the
grounds that the probative value of the statement admitted for a non-hearsay purpose is
substantially outweighed by the danger of unfair prejudice, under Rule 403.

23

In a negligence action to recover damages for personal injury sustained in a fall, a statement to a customer
of a food store by the manager that the floor in aisle 2 is wet is NOT hearsay, and thus admissible, when it
is offered to show the unreasonableness of the customers conduct in skipping down aisle 2. BUT, the same
statement is offered to show the floor was wet is hearsay, since it goes to the truth of the matter asserted,
i.e. that the floor was wet
o Threats made to the defendant bearing on the reasonableness of his apprehension of danger or providing
motive for the action are NOT hearsay when offered for such a purpose
o The victims fearful state of mind is an element in proving extortion. The testimony of victim as to what
others say to them and the testimony of others as to what they said to the victim are admitted, not for the
truth of the information in the statements, but for the fact that the victim heard them from which one can
infer the state of mind of a reasonable person having heard such statements
o Where the defendant alleges duress as a defense, statements threatening her are NOT hearsay when offered
to show her state of mind; for an income tax violation, evidence as to advice received by the defendant is
NOT hearsay when offered to prove intent.
o Instructions to an individual to do something, such as a mom telling her son to wait here, Ill be right
back, is NOT hearsay
Impeachment Prior statements of a witness inconsistent with the witnesss in-court testimony offered solely to
impeach, Rules 607 and 613, are NOT hearsay
o Reasoning: prior inconsistent statements are admissible because they attack credibility and are not being
offered for the truth of the matter asserted, but to show that the statements were in fact said. Also,
inconsistency shows the witness may be lying.
o

4.

NOTE: Evidence relevant for its effect on the listener is subject to exclusion under Rule 403, after taking into consideration the giving
of a limiting instruction = at the time the evidence is being introduced, i.e. testimony is not as to the truth of the matter asserted but as
to explanatory/listening effect.
PROBLEM AREAS IN DEFINING HEARSAY
1. Circumstantial Evidence Whether evidence is direct or circumstantial is irrelevant in determining whether evidence is hearsay.
But, judges will sometimes characterize otherwise inadmissible hearsay evidence as circumstantial in order to allow circumstantial
evidence to be admissible. To stop this, enacted the residual hearsay exception, Rule 807, which allows circumstantial evidence to be
admissible on an ad hoc basis (so it doesnt create precedent) when the evidence is highly reliable
A) Mechanical Traces: the presence of something upon a person or premises may constitute circumstantial evidence that gives rise to
an inference that a person did an act with which these circumstances are associated. Frequently relevant as circumstantial evidence
looking backwards to show that some act was or was not done, and hearsay questions only arise when the relevancy of the
circumstantial evidence derives solely from the truth of the mechanical trace.
- (a) the presence upon a person or premises of articles, fragments, stains, or tools, (b) brands on animals or times, or (c) tags,
signs, and numbers on cars, railroad cars, or other vehicles or premises and (d) postmarks, fingerprints, and footmarks.
- License Plates: offered to prove owner was involved in crime/accident; postmarks on envelopes: offered to prove D was
acquainted with the sender; receipts from commercial transactions: offered to prove person whose name on receipt stayed at
hotel on night in question
o Hearsay: A tag bearing the name Bill Snow on a briefcase with drugs. Since the relevancy of the tag to identify the
defendant whose name is Bill Snow with the briefcase to which the tag is attached derives from the truth of the
assertion made on the tag, i.e. briefcase belongs to Bill Snow. Thus, the tag is hearsay, since its relevancy derives
solely from the truth of the mechanical trace it names the defendant.
Where the relevancy of the mechanical trace is not solely derived from the device itself, i.e. someone with personal knowledge
testifies = NOT hearsay witness testifies that he knows Bill Snow owns the briefcase, NOT hearsay, since it requires in-court
testimony authenticating the tag
Example: A book of matches bearing the name Red Fox Inn is found on the defendant accused of a murder committed at the Red Fox
Inn
- Hearsay: If authenticated solely as having been taken off the person of the defendant, the matchbook is hearsay
since its relevancy depends on the acceptance of the assertive statement on the matchbook that its origin is the Red
Fox Inn.

24

NOT Hearsay: Owner of the Red Fox Inn testifies that the matchbook found on the defendant is identical to the
matchbooks he places on tables for use by customers. Here, the relevancy of the matchbook is no longer dependent
on the truth of the matter asserted, but is placed upon personal knowledge and the process of comparison.

B) Character of an Establishment: situations where the character of an establishment is sought to be proved by evidence of
statements made in connection with activities taking place on the premises.
- Basically when someone is associated with the establishment, a customer or potential customer, makes a statement that
characterizes the establishment
o Hearsay: If cops answer phone and record calls each answered by cops, stating this is A, place 20 on X, these
statements are hearsay because there is no independently relevant act, apart from the statements placing the bets
themselves, for the statements to characterize,, and thus the statements are irrelevant if offered solely for the fact that
they were said.
o NOT Hearsay: For the out of court statements placing a bet to is relevant only when offered to prove the truth of
the matter necessary implicitly being asserted by the out-of-court declarant, i.e. that the establishment being reached
is in fact a betting parlor. Also, statements fall within the residual hearsay exception under Rule 807
NOT hearsay when established that the declarant believed he was calling a betting parlor have to
establish that the declarant intended to call the number reached; must have believed that the number dialed
was a betting parlor; must have intended to place a bet; must have intended to place a bet instead of playing
a practical joke
o NOT Hearsay: if cops were at the location and overheard an employee taking a bet (or through a wire tap), this
statement of the out-of-court declarant no longer has to be true to be relevant. So, even if the declarant didnt want to
place a bet or didnt believe he was calling a betting parlor, it wouldnt matter at that point, because the officer heard
the employee accept the bet.
o NOT Hearsay: Drug balance sheet found in apartment associated with defendant offered to prove that the
apartment associated with the defendant is used for drugs. Thus, not hearsay since the government is not trying to
prove the truth of the matter asserted, i.e., who owed how much, but the character of an establishment made in
connection with activities taking place on the premises.
C) Personal Knowledge of Independently Established Facts: Statements are introduced to prove that the declarant had personal
knowledge of the truth of the matter asserted when the truth of the matter asserted is firmly established by independent evidence.
- Personal knowledge of the declarant is relevant in such cases to establish the presence of the declarant at a particular location
at a particular time
- Hearsay statements used to establish the declarants personal knowledge of already independently established facts are NOT
hearsay because they are not being offered to prove the truth of the matter asserted, rather they are offered to prove the
declarant had personal knowledge of those facts
o Sharon Schunk: Child makes statements to her mom and police during questioning about the home of a man who
abused her (general statements regarding the appearance of the steps to the porch, the front door, and the room and
articles therein.) Both mom and police can testify to what the child said as circumstantial evidence of personal
knowledge not being offered to prove the truth of the matter asserted because it proves the child had such
knowledge; and that such state of mind on her part was acquired by reason of her having been in that room and
house prior to making such statements. Even though there is evidence in the record of what the defendants home
looked like, the childs recollection isnt being offered to prove what the defendants home looked like (hearsay), but
instead is being offered to prove that the place the child described was in fact the same place as the place of abuse
(NOT hearsay).
Childs statement contain all of the hearsay risks, although risks are small. But, because the risks of
perception, recordation and recollection, narration, and sincerity are present, the statements fall within the
definition of hearsay when offered to prove her personal knowledge of objects in the house to establish that
the assault took place there. However, statements are admissible under the hearsay exception of Rule 807
D) Circumstantial Use of Utterances to Show State of Mind: When declarations indirectly, impliedly, or inferentially indicate the
state of mind of feeling of the declarant, it is NOT hearsay; BUT, a direct declaration of the existence of a state of mind or feeling,
which it is offered to prove, is hearsay
- NOT Hearsay: if A testified that B told him C is the finest of my sons,
- Hearsay: If A testified that B told him I like C better than my other sons, where the evidence was being shown to prove B
had a special fondness for C
- Mental Capacity and hearsay: Person says I am Napoleon and its being offered to bear upon the persons mental state. Its
hearsay since you have to believe that the declarant actually believes hes Napoleon or you cannot draw any inferences as to
the persons mental state.

25

Graham disagrees with this. Even if the declaration only proves declarants belief, not just that the statement is true, its still
hearsay still proving declarant really believed it unless the trier of fact finds that the declarant really believed his statement,
cant draw any inference.
NOTE: with the availability of the residual hearsay exception of Rule 807, trustworthy and necessary hearsay will no longer be
inadmissible simply because it failed to fit neatly into one of the pigeonhole hearsay exceptions. Also, now that Rules 801(a)-(c)
include (1) a statement to the extent relevant only if the declarant believes the matter asserted to be true; and (2) a statement whose
relevance depends upon the matter asserted being true without reference to whether a further inference is then going to be drawn. Any
contrary suggestion with respect to statements forming the basis of a non-asserted inference is incorrect!
PRIOR STATEMENT BY WITNESS
Rule 801(d)(1). When the out-of-court declarant is also an in court witness, the witnesss prior statements, whether consistent
or inconsistent with the witnesss in court testimony, or constituting a prior statement of identification, can be explored on direct and
redirect examination or cross-examination of the witness to the same extent as the witnesss in court testimony on personal knowledge
of the matters stated in such out-of-court statements.
Reasoning: Since witness is under oath throughout her testimony, her demeanor is being observed throughout, and thus
the truth assessing components of the ordinary system of oath, demeanor, and cross are satisfied wherever the out-ofcourt declarant testifies at the trial or hearing and is subject to cross
If the witness admits on the stand that he made the statement and that it was true, he adopts the statement and there is NO hearsay
problem. Problem only arises when the witness on the stand denies having made the statement or admits having made it but denies the
truth
Rule 801(a) Committees Note A statement is NOT hearsay if it is being offered as the basis for inferring something other
than the truth of the matter asserted.
But Graham disagrees = it is still hearsay because there is still a sincerity risk. The truth of the matter asserted must be
assumed in order for the non-asserted inference to be drawn. Risk of sincerity arises from the fact that the matter asserted
must be true for any inference at all to arise.
o Example: Out-of-court statement by witness says Car A was going 80 mph about 5 blocks from the site of the
accident. While this isnt directly asserting Car A was going 80 mph at the time of the accident, its also not really an
inference. This isnt directly asserting Car A was going 80 mph at the time of the accident however you still have to
believe the car was going 80 mph 5 blocks away for it to be true so it is hearsay
Advisory Committee notes that, under Rule 801(d)(1), not all of the in-court witnesses are defined as NOT Hearsay Rule
requires that the declarant actually testify as a witness, and it then enumerates three situations in which the statement is
excepted from the category of hearsay. Compare Rule 63(1), which allows any out-of-court statement of a declarant who is
present at the trial and available for cross.
NOTE: Consideration of trial concerns underlying Rule 403, the danger of unfair prejudice, confusion of the issues, misleading the
jury, undue delay, waste of time, and the needless presentation of cumulative evidence, result in only some prior inconsistent
statements, Rule 801(d)(1)(A), and some prior consistent statements, Rule 801(d)(1)(B), being defined as NOT hearsay. On the other
hand, as interpreted, all prior statements of identification of a person made after perceiving the person again, Rule 801(d)(1)(C) are
defined as NOT hearsay.

26

NOT HEARSAY DEFINITIONS


Rule 801(d). STATEMENTS ARE NOT HEARSAY IF:
(1) PRIOR STATEMENTS BY A WITNESS. The declarant testifies at the trial or hearing and is subject to cross concerning
the statement, and the statement is
(A) INCONSISTENT WITH THE DECLARANTS TESTIMONY, AND WAS GIVEN UNDER OATH SUBJECT TO THE
PENALTY OF PERJURY AT TRIAL, HEARING, OR OTHER PROCEEDING, OR IN A DEPOSITION.
Thus, under Rule 801(d)(1), prior inconsistent statements made under oath at formal proceedings are no substantively admissible
substantive admissibility is allowed only for those statements possessing the highest degree of certainty of making made under
circumstances conducive to truth telling.
The foundation requirement of Rule 613 applies to prior inconsistent statements admitted
Change from c/l
o Reasoning: unlike other situations involving un-sworn or oral statements, there can be no dispute as to whether the
prior statement as made and the context of a formal proceeding and an oath provide firm additional assurances of the
reliability of the prior statement
Prior inconsistent statements not falling within the scope of Rule 801(d)(1)(A) may still be employed for the limited purpose
of impeachment in accordance with Rules 607 and 613
For a prior inconsistent statement by a witness to NOT be hearsay, it must be:
1. Made by declarant who is testifying at trial
2. Subject to cross-examination concerning the statement
3. Prior statement must be inconsistent with current testimony
4. Prior statement must have been given under oath, subject to penalty of perjury
5. At a trial, hearing, or other proceeding, or in a deposition, including Grand Jury testimony
a. Note: grand jury testimony ok; either oral o written statements even if videotaped under oath to cops is not NOT
hearsay, it is hearsay!
(B) CONSISTENT STATEMENTS OFFERED TO REBUT AN EXPRESS OR IMPLIED CHARGE AGAINST THE
DECLARANT OF RECENT FABRICATION OR IMPROPER INFLUENCE OR MOTIVE
Note: Generally, a witness cannot be corroborated on direct or redirect examination or rebuttal by proof of prior statements consistent
with his in-court testimony, because whatever inherent probative value such consistent statements may have is felt to be insufficient
when viewed in light of trial concerns, under Rule 403.
BUT, under certain circumstances the probative value of a prior consistent statement clearly warrants introduction. According to Rule
801(d)(1)(B), to rebut an express or implied charge that the witness is motivated or has been influenced to testify falsely or that his
testimony is a recent fabrication, evidence is admissible that he told the same story before the motive or influence came into existence
or before the time of the alleged recent fabrication.
RULE: The prior consistent statements is exempt from the bar of the rule against hearsay, Rule 801, through definition as
NOT hearsay, Rule 801(d)(1)(B), and thus is admitted as substantive evidence.
A. Where admissible, the prior consistent statement may be testified to by either the witness himself or any other person
with personal knowledge of the statement.
B. Rule 801(d)(1)(B) does not require the declarant testify at the trial and that he be subject to cross, so long as the declarant
is available to be recalled
Requirements for defining consistent testimony as NOT hearsay
1. Declarant testifies in court and is subject to cross; and
2. The statement is consistent with the declarants in-court testimony; and
3. The statement is being offered to rebut a charge of recent fabrication; and
4. The prior consistent statement must have been made before the supposed corruption, which is affecting the truthfulness of the
testimony.
Improper Motives = love, money, marriage, sex, etc. AND
Improper Influence = bribery, intimidation, etc.

27

These are things that would naturally cause a person to testify in one direction even if they dont consider what theyre saying
to be true.

Example: John witnesses an accident involving a car driven by Mary and a truck driven by Bill. The factual issue in dispute is the
color of the traffic light at the intersection facing both parties. At trial, John testifies that the light facing Mary was green. On cross,
Bills attorney brings out that 4 weeks after the accident, John met Mary for the first time at her lawyers office, that they dated
thereafter, and that they are now engaged to be married. On redirect, John may testify that one day after the accident John met Mary
for the first time at her lawyers office, that they dated thereafter, and that they are now engaged to be married. On redirect, John may
testify that one day after the accident, and thus before the alleged improper influence or motive arose, he told his best friend Tim that
the light facing the car driven by this woman was green. However, he may NOT testify that two weeks after John and Mary were
engaged he told his mother that the light facing Mary was green.
NOTE: A prior statement that corroborates the witness may be admitted without reference to Rules 801(d)(1)(B) if it serves to explain
or modify a fragment introduced by the opposite party for purposes of impeachment, whether an inconsistent statement or failure to
speak when natural to do so. This is not substantive evidence, but is admitted for corroborative purposes only, and the jury will be so
instructed.
(C) PRIOR IDENTIFICATION OF A PERSON AFTER PERCEIVING HIM
Rule: Statement is NOT hearsay when the declarant testifies in court and is subject to cross and the statement is one
identifying a person made after perceiving the person again.
Requirements for defining prior identification as NOT hearsay
1. You have to see the person again, after you saw him at the scene of the act or crime.
2. Any person present at the prior identification may testify, such as the cop at the lineup, and statement will be admitted, even
if the declarant at trial states that he now cannot ID the D.
o Reasoning: This is to avoid witness intimidation
Example: If, after your purse is stolen, witness gave a description to the police, the description should not be admitted under this Rule
because she did not perceive the D again.
Therefore, witness must participate in a lineup: either a photograph or a sketch, after the incident to be able to have your prior
identification admitted.
NOTE: There is no requirement that the witness first be impeached based on the theory that courtroom identification is so
unconvincing as practically impeaching itself, thus justifying the corroboration. There is no restrictions upon admissibility other than
having the alleged out-of-court declarant in court on the witness stand subject to cross concerning the statements.
Purpose of the Rule: is to permit the introduction of more meaningful identifications made by a witness when memory was fresher
and there had been less opportunity for influence to be exerted upon him.
Rule 801(d). STATEMENTS ARE NOT HEARSAY IF:
(2) ADMISSION BY A PARTY OPPONENT. The statement is offered against a party and is
Overview: an admission by a party-opponent is admissible as substantive evidence to prove the truth of the matter asserted, and
classified as NOT hearsay based on the theory that their admissibility is the result of the adversary system rather than satisfaction of
the conditions of the Hearsay Rule. Lack of opportunity to cross is deprived of significance by the incongruity of the party objecting to
his own statement on the ground that he was no subject to cross by himself at that time.
NOTE: No requirement for personal knowledge or mental capacity. Admission in the form of an opinion are competent, even if the
opinion is a conclusion of law. Admissibility does not depend upon whether the declarant is unavailable, available, or actually testifies.
But, admissions by a party opponent to may be excluded under Rule 403.
(A) THE PARTYS OWN STATEMENT, IN EITHER AN INDIVIDUAL OR REPRESENTATIVE CAPACITY, OR
Rule 802(d)(2)(A): a partys own statement made in his individual capacity when offered by an opposing party is defined as
NOT hearsay.
The party making the statement does not have to testify (nor be available or unavailable), nor have personal knowledge of the
subject matter of the statement, nor have required mental capacity, nor does it have to be against interest when made or when
offered.

28

1.

The declarant may be an individual or a representative of a non-natural party.


No foundation need be laid before the witness testifies
Plea of Guilty

Rule: A guilty plea is NOT hearsay and may be used as evidence if the plea was made by a party opponent and is being
introduced by the adverse party.
Introduced as proof that the offender admitted facts constituting guilt, not as proof that the essential facts have been
previously found to exist.
The Ds reason for entering plea may also be admitted.
Exclusion for a guilty plea entered to a minor offense (offense punishable by imprisonment for no more than a year).
o Reasoning: motivation to defend at this level is often minimal or nonexistent.
o BUT, majority of decisions do admit the guilty plea in these circumstances with explanation.
Can avoid Rule by pleading nolo contender
Note: may also be admitted under Rule 804(b)(3), as a statement against interest of a non-party
2.

Statements of Party Made in a Representative Capacity

Rule: Admissions of representatives are admissible regardless of whether the statement was made in an individual or
representative capacity. This includes an administrator, executor, trustee, guardian, or agent.
The statement is admissible without reference to whether the individual was acting in a representative capacity in making the
statement; all that is required is that the statement be relevant to representative affairs, since declarant is present and able to
adduce whatever counterproof might exist in order to deny
3.

Persons in Privity of Jointly Interested

Rule: A statement made by a person in probity or with a joint interest (joint tenants) are NOT admissions of a party opponent.
Privity = mutual or successive relationship to the same rights of property (inherit property).
Reasoning: statements neither furnish criteria of credibility nor aid in the evaluation of testimony
BUT, statements will usually qualify as representative admissions, Rule 802(d)(2)(C), or as statements against interest, Rule
804(b)(3), or within another hearsay exception
Example: So if As dad said that the property line only extended out 10 yards and then he dies, leaving the property to A, As dads
statement is not admissible against A if A is being sued regarding the property line.
(B) A STATEMENT OF WHICH THE PARTY HAS MANIFESTED AN ADOPTION OR BELIEF IN ITS TRUTH, OR
Rule 801(d)(2)(B): Statement is NOT hearsay when a person adopts someone elses statements to which a party has manifested
his adoption or belief in their truth.
Person is held to have manifested an adoption or belief in the truth of a statement ONLY if it appears that the person
understood and demonstrably assented to its truth.
Words or Conduct
May either be expressed, such as yeah, you are right, or implied from the partys reliance upon the contents of statements in
related conversations, such as bet we wont get as much money from the next store.
Question of Conditional Relevancy under Rule 204(b) and the burden of proof is on the proponent to show that the adoption
was intended.
o Example: if the contested matter in a civil action for damages is whether a visiting kid tripped over her shoe laces or
was pushed down the stairs, a statement by the homeowner such as I am very sorry. My son said that my dog
jumped on your kid, would be an admission. The statement, my son said he thinks my dog jumped on your kid,
does NOT indicate approval of the reported assertion, and is thus not an admission.
The person assumes the correctness of the 3rd partys statements in his own statement so its an admission. If you get a sense
that the persons incorporating the statement, then its an adoption
Silence
Idea based on the premise that a person would deny a statement if they thought it to be true.

29

Police

Includes criminal and civil cases


o If someone accuses a party of a crime and the party doesnt protest, (unless it involves a police officer), its an
adoption
Even if someone sends you a letter, there are circumstances in which failure to reply to the letter (or email) can be an adopted
admission by silence
When a cop speaks to someone not under arrest and silence ensues, it is an adoption by a party opponent. If a police officer
speaks to someone before they are mirandized and there is silence it can be used for purposes of impeachment only (the
federal system)

(C) A STATEMENT BY A PERSON AUTHORIZED BY THE PARTY TO MAKE A STATEMENT CONCERNING A


SUBJECT; OR
Rule 802(d)(2)(C): statements made by a person authorized to make a statement concerning the subject matter
The president of a corporation, managing partner, attorney, etc.
The authority of the agent to speak on the subject, which may be express or implied, must be established. The contents of the
statement is to be considered, but not sufficient
Authorization may be established by the acts or conduct of the principal or his statements to the agent or a 3 rd party.
Statements by the authorized person to the principal himself are also included accordingly, a partys books or records are
useable against him without regard to intent to disclose to third persons.
NOTE: When you have to establish that the person making the statement was authorized to do so, look to if he could make decisions
about the subject matter they are talking about or whether the subject matter was in the scope of decision-making.
Example: When an employee gets fired and finds out from someone else at the company that they were fired for being over
55 years old; in order for that statement to be admissible, the declarant must be either the decision maker or in the chain of
decision making.
(D) A STATEMENT BY THE PARTYS AGENT OR SERVANT CONCERNING A MATTER WITHIN THE SCOPE OF
THE AGENCY OR EMPLOYMENT, MADE DURING THE EXISTENCE OF THE RELATIONSHIP, OR The contents of
the statement shall be considered, but are not alone sufficient to establish the declarants authority under subdivision (C), the
agency or employment relationship and scope thereof under subdivision (D), or the existence of the conspiracy and the
participation therein of the declarant and the party against whom the statement is offered under (E).
Rule 802(d)(2)(D): statement made by an agent concerning a matter within the scope of his employment is NOT hearsay.
Requirements for statement made by an agent to be considered NOT hearsay:
1. A statement of an agent or servant
2. Concerning a matter within the scope of his employment
3. Made during the existence of the relationship; agent or servant still employed at the time of the statement

In determining whether an agency or employment relationship existed at the time of the statement and scope of such a
relationship, the content of the statement is to be considered but is not alone sufficient.
Authority to speak is no longer a concern all that is required is that the statement be within the scope of the employment and
that the employee still be employed.
In criminal prosecutions, government employees are not considered agents or servants of a party-opponent for the purpose of
admission rules
An attorney may act as an ordinary agent and admissions will be admissible against his principal. Also, attorney has authority
in general to make judicial admissions for the client in all matters

Rule 802(d)(2)(E): A statement of one coconspirator is admissible against the others as an admission of a party-opponent in both civil
and criminal cases, if.
Requirements for statement of one co-conspirator being NOT hearsay:
1. Made during the course;
2. And in furtherance of the;
3. Common objectives of the conspiracy

30

The court applies the more probably true than not burden of proof and only if the court decides that the statement was made
during the course and in furtherance of the common objectives of the conspiracy, is the statement admissible.
Contents of the statement alone are not sufficient to establish the existence of the conspiracy and the participation therein of
the declarant and the party against whom the statement is offered
o In addition to the content of the statement, you need some independent evidence connecting the not present party to
the conspiracy. As long as you have the slightest amount of independent evidence, you can use the statement too.
o Example: hotel receipt showing that co-conspirator was staying in the same hotel as the other conspirators.
In reaching its decision, court takes account of all relevant evidence: statement, any circumstantial evidence, and any
evidence offered on behalf of the accused.
Court should not refer to its preliminary determination of facts leading to the introduction of the statement in the jury
instructions

Question of how these admissions are treated in court:


(A)

Judicial admissions removes the matter from consideration Not evidence, but has the effect of withdrawing a fact from
contention. Its conclusive and cant be contested. Jury will be instructed to take the fact as the truth.

Examples:
1) Pleading D in case pleads in the answer admitting a certain allegation in the complaint. This is a judicial admission. D
admitted it. P doesnt have to do discovery or present evidence with respect to that particular admission. P can still present
other evidence of the admission if he chooses (but cant take took much time do to so under Rule 403). The idea is that you
can eliminate facts of the case that arent contested to focus the case on the real issues
2) Other judicial admissions are formal factual admission in open court by lawyers (not legal arguments);
3) Request to admit (the part of discovery where you take all the things you think arent being contested and you stipulate
certain facts) this can be partial admit to the fact that a document is not hearsay but dont admit to its relevancy. Then the
response of the judge becomes a judicial admission
(B)

Evidentiary admissions Everything that is not a judicial admission is an evidentiary admission and can be contested. This
includes responses to depositions and interrogatories.

31

HEARSAY EXCEPTIONS
Both Rules 803 & 804 specify requirements considered to be sufficient guarantees of trustworthiness to justify introduction absent an
opportunity to conduct contemporaneous cross of the declarant
NOTE: A statement qualifying as an exemption to the hearsay rule must, of course, satisfy other provisions, such as being relevant;
based on personal knowledge when made; be properly authenticated; and meet the requirements of the Original Writing Rule (1002)
where the content of writing, recording, or photo is being proved before it can be admitted into evidence. Therefore, these exceptions
are phrased in terms of non-application of the Hearsay Rule, rather than being phrased in positive terms of admissibility.
Rule 403 still remains applicable = Even though the evidence meets the requirements of an exception, the court may still exclude the
evidence on the grounds that after considering, where applicable, the effectiveness of a limiting instruction, Rule 105, the incremental
probative value of the evidence is substantially outweighed by the danger of unfair prejudice, confusion of the issues, or misleading
the jury, or undue delay, waste of time, or needless presentation of cumulative evidence.
RULE 803. HEARSAY EXCEPTIONS; DECLARANT AVAILABLE AS A WITNESS:
These are statements/documents that are hearsay. But, theyre admissible as exceptions to the hearsay rule.
Each of these exceptions apply even where the declarant is available to give in-court testimony, and thus doesnt matter if the
witness testifies at all or if the witness has a different or the same story as declarant.
Admissible whether or not self-serving when made or offered.
Opinion Rule of 701, 702, 704 (used to elicit more concrete informative answers) as it relates to the form of the testimony is
not applicable since out of court statements are not made where alternative forms of expressions may be secured.
RULE 803(1): PRESENT SENSE IMPRESSION: statement describing or explaining an event or condition, or immediately
thereafter.
Declarant describes exactly what he is looking at without knowing how/why it happened.
o Reliability = slight chance of recording failure, and less time to make it up.
Does not require personal knowledge of the event by the in-court witness (to whom the statement was made) but only
personal knowledge as to the making of the statement by the declarant.
o Evidence to prove personal knowledge as to the making of the statement may consist solely of the statement itself
considered in light of surrounding circumstances, i.e. corroboration is not required.
o Example: guy working under the car while the accident happens and the other guy watching the accident sees it and
makes a statement. There need not be corroboration.
Exception only applies to statements the explain or describe the current event like a freeze frame; only descrive what
happened at that moment.
o Example: the mechanics say the light has been out all day, will not be admitted under this exception
Immediately thereafter: this is an ambiguous term, usually this should mean seconds, but some jurisdictions extend it to
minutes.
Rule 803(2). EXCITED UTTERANCE: A statement relating to a startling event or condition made while the declarant was
under the stress of excitement caused by the event or condition. (Oh, my G-d statements)
An expression of excitement that shows you are responding instinctively to an event or condition without the capacity for
reflective thought thus producing statements free of conscious fabrication
Based upon a reliability determination that lessens sincerity risk.
Recognized in spite of the fact a starting event may also impair accurate observation
The statement may be made by a bystander; it need not be made by a participant as one may be startled by an event as to
which one is not an actor
It does not matter that the statement contains an opinion, provides details of the event, accuses someone of a crime, or is selfserving.
Admissible even if witness denies making the statement at trial.
Where the only evidence of the existence of the startling event or condition is in fact the content of the statement itself, the
prevailing practice is to permit introduction AND personal knowledge of the declarant may also be established by reference
to the content of statement
o Example: If the statement that light has been out all day is made at the time of the startling incident then it is
admissible because it relates to the startling incident.
Requirement for Admissibility

32

1.
2.
3.

The occurrence of an even or condition sufficiently startling to produce a spontaneous and unreflecting statement;
Absence of time to fabricate, i.e., the statement must be made while still under the influence of the startling event or
condition; and
A statement relating to the startling event or condition.

Factors to be considered an Excited Utterance Exception: Totality of the Circumstances


A) Amount of Time = standard measurement is the duration of the state of excitement
B) Nature of the condition or event
C) Age and condition of declarant
D) Presence or absence of self-interest; and
E) Whether the statement was volunteered or in response to a question = the fact that someone says what happened does not
exclude it as an excited utterance, however, if there are a series of questions preceding the utterance then it is no longer an
excited utterance.
Example: If someone drugged, passed out in a trunk, wakes up, sees the cops, then blurts out what happened to him
it is still considered an excited utterance
Example: Guy calls police saying, help me, Harry is coming at me with a knife the cops show up and Harry is dead on the floor with
knife stabs. Does statement get admitted? Yes, it is not the job of the judges to make any determination about the bad faith on the part
of the declarant, that job is for the jury. The jury understands that this may be a self-serving statement.
Sexual Battery Cases
Problems: A lady attacked in a car is left in a field after being assaulted, has to run home past the police station, her aunts
house, etc, she gets home and goes upstairs and her mom follows her and asks what happened? Even though there were
other opportunities to make the statement somewhere else, this will get admitted because of the stress of the victim.
Child sex abuse cases = liberal standard is applied; broader scope of factor (time: usually 2-3 days)
Unidentified Declarant: at the scene of an accident the declarant makes a statement and then is not seen or heard from again the
statement is still admissible as an excited utterance and it is up to the jury to decide its value
Where the statement by an unidentified bystander lacks trustworthiness for any reason, including uncertainty as to whether it
was ever made, courts are prone to conclude that a sufficient showing as to personal knowledge of the declarant (602), has
not been made.
NOT excludable under 403 = evaluation of whether an in-court declarant is telling the truth is not one of the functions
undertaken by the court in evaluating probative value vs. trial concerns; after first assuming the jury finds the statement to be
made, trial judge is required to view probative value most favorable to the proponent of the evidence.
Res gestae (things done): common law term, carefully avoided by the Federal Rules; encompassed in defining hearsay, and
hearsay exceptions
NOTE: In both 803(1) & 803(2), issues of bad faith of in-court witnesses is not applicable it is a jury determination as to the
weight given to the statement, and not considered an abuse of discretion upon an appeal
RULE 803(3). THEN EXISTING MENTAL, EMOTIONAL, OR PHYSICAL CONDITION; INTENT AS PROOF OF DOING
ACT INTENDED; STATEMENT AS PROOF OF FACT REMEMBERED OR BELIEVED; WILL CASES.
Rule: A statement of the declarants then existing state of mind, emotion, sensation, or physical condition (such as intent, plan,
motive, design, mental feeling, pain, and bodily health), but not including a statement of memory or belief to prove the fact
remembered or believed unless it relates to the execution, revocation, identification, or terms of declarants will.

Statement has to be forward looking, and NOT backwards (retrospective declaration)


o But, the then existing mental state, emotion, sensation, or condition may be inferred to exist into the future and to
have existed in the past
o Example: Dr. Shepards wifes statement, Dr. Shepard poisoned me is inadmissible because it is not a statement of
present mental, emotional, or physical condition, but only backwards looking
Exception: Will cases: can introduce a backwards looking statement to prove the truth of the matter
asserted, (he has made a will, has made one for a particular purpose, or he has not revoked a will), i.e.
evidence that declarants will was destroyed and a new one was created
Duration/Continuity of Inference: discretion lies within the court, and varies with the particular attitudes or feelings at issue
and with the cause, and may require some reasonable indication the state of mind was the same at the material time.

33

If statement contains an assertion as to particular facts, limiting instruction is required


Rule 403, probative value applies
Mutual Life v. Hillmon
o (1) upheld rule allowing evidence of intention as tending to prove the doing of the act intended; AND
o (2) Approved introduction of a statement of intent of a declarant to infer not only the declarants future act but also
the future act of another. (Graham disagrees)

Examples:
A) Current state of emotions: Admits I love you.
B) Declarants intent tending to prove the doing of the act intended: I am going to Boston on a business trip. The plane crashed
en route, is this statement admissible? Yes, it is a current statement of your intent and is admissible
C) Imbedded Statements of intent of another party: I am going to get a pond of grass from Harry at the Publix Parking lot.
This statement is admissible against Harry even though it is not a statement by him about his then existing mental condition
because the SCOTUS says so.
D) Example of Hearsay not included under 803(3): I believe I saw John throw the ball through the window, when offered to
prove that John threw the ball through the window.
Rule 803(4). STATEMENTS FOR PURPOSES OF MEDICAL DIAGNOSIS OR TREATMENT: statements made for purposes
of medical diagnosis or treatment and describing medical history, or past or present symptoms, pain, or sensations, or the
inceptions or general character of the cause or external source thereof as reasonably pertinent to diagnosis or treatment.

It doesnt matter who the statement is made to (doctor, nurse, family member) as long as it is made for purpose of diagnosis
or medical treatment.
o Telling the truth to enable correct medical treatment
No long a distinction (as was at c/l) as to a treating doctor and an examining doctor as long as the statement is made for
diagnosis or treatment.
o Examining Doctor: will be able to testify even if the only purpose of the exam was to enable him to testify
Statement can be made by patient or person with interest in victims well being
Statements to lay witnesses are not admissible
General Statements of Causation = limited to those of inception or general character of the cause or external source of the injury
insofar as reasonably pertinent to medical diagnosis or treatment. Thus, must be relevant as to why you are hurt and must have
concurrence to what you say and what the doctor thinks
Part of a statement can be included within a part that is inadmissible; I was hit by a car (admissible), that ran a red light
(inadmissible)
Child Sexual Abuse = statements of the alleged child-victim to a physician identifying the D as the perpetrator, when a member of the
childs family or household, are admissible as pertinent to diagnosis or treatment of emotion and psychological injuries and pertinent
to preventing a recurrence of the injury.
Familial or custodial setting statements are admissible due to the idea that prevention is part of treatment.
Example: I was hit by a car and thrown 30 feet and hurt my leg. All of this is admissible because it will tell the doctor information to
help him treat you. However, information like I was in the crosswalk; the light was red; etc.. is not relevant for purpose of medical
treatment.
Rule 803(5). RECORDED RECOLLECTION: A memorandum or record concerning a matter about which a witness once had
knowledge, but now has insufficient recollection to enable the witness to testify fully and accurately, shown to have been made
or adopted by the witness when the matter was fresh in the witnesss memory and to reflect that knowledge correctly.
Requirements to a record being admissible as an exception if
1. Proponent can show that the witness once had personal knowledge of the matter;
2. That the record or memo was prepared or adopted by him when it fresh in the memory;
3. That it accurately reflected his knowledge; and
4. That the witness currently has insufficient recollection to enable him to testify fully and accurately

Lawyer can read the memo or record, but it has to be evaluated as live testimony, cannot go into jury courtroom as is
evidence entered into the record

34

Any foundation that can establish that the witness knew it at the time will work, BUT has to be something that the witness
saw before being on the witness stand.
o Example: Your inventory list of items stolen after a robbery.
o NOTE: Witness can testify that he doesnt remember making the document (thus, not seeing it before), but that the
writing is his as well as the signature

Witnesss can testify


A. Either that he remembers making an accurate recording of the event in question, which he now no longer sufficiently
remembers,
B. That he routinely makes accurate records of this kind, OR
C. If the witness has entirely forgotten the exact situation in which the recording was made, that he is confident from the
circumstances that he would not have written or adopted such description of the facts unless that description truly described
his observations at the time.
Rule 803(6): Records of Regularly Conducted Activity (Business Record Rule): Any document of acts, events, conditions,
opinion, or diagnoses, made at or near the time of occurrence created by or from info transmitted by a person with personal
knowledge is a hearsay exception when the business regularly records such events or acts and these events or acts regularly
occur within the business unless the source of information or the method or circumstances of preparation indicate a lack of
trustworthiness.
Requirements to meet 803(3) Exception:
1. In the regular course of business to make this record
a. Cops: prosecutorial vs. Ambulance drivers: medical treatment
2. Record kept in the regular course of business
3. Record made at or near the time of the matter recorded
4. Record made by a person within the business with personal knowledge of, or made from information transmitted by a person
within the business with personal knowledge of, the acts events, conditions, opinions, or diagnosis appearing in it
5. Whoever created the report or offering information that was put into the report must have a duty to do so
Example: Cop writes down what a witness tells him in the police report this is information being conveyed by a person with first
hand knowledge (witness), but that person is not in the business, so it doesnt fall under this rule.
Verifiability: doesnt have to be something that can be verified ex: cop at scene of crime writes down his own perceptions
cant verify this after the fact but its still a requirement of his job.
Foundation must be laid by the records custodian (person in charge of the records), other qualified witness, or by
certification by records custodian. Custodian need not have personal knowledge as long as he knows that these records are
kept there, such as hospital records custodian ca testify to hospital records.
Exception covers any business except public agency.
Anticipation of litigation: have to show the document was created in the act of regularly doing business, and thus, if it was just created
in anticipation of litigation, its not admissible.
But, sometimes a document is created in regular course of business but it relates to future litigation such as a dept that
investigates employee accidents this will possibly be let in, but it will be given weight as to reflect that it was prepared in
anticipation of litigation (here, would need a witness with first hand knowledge)
Multiple Level Hearsay: Different hearsay rules can mix together under this rule.
Example: At the scene of an accident, an ambulance driver is told by an occurrence witness, The boy was hit by that car. It went
through the red light.
Assuming the witnesss statement meets the requirements of an excited utterance, only that part of the statement relating to
the car striking the boy is admissible under 803(2), 803(6), and 805. Since the ambulance drivers duty to record does not
extend to statements not pertinent to medical diagnosis or treatment, the part of the statement relating to the car going
through a red light is inadmissible even though the ambulance driver included that segment of the statement in the report.
If the same statement is made to a cop, the entire excited utterance would be admissible under Rule 803(2), 803(8) and 805 in
a civil case by virtue of the fact that the cop was under a business duty to record all statements relating to the cause and
circumstances of the accident being investigated.
Lack of Trustworthiness
1) Belief that the requirements of the Exception have not been satisfied, i.e. record was not kept the way it was supposed to be;
2) Record itself has motivations for falsification

35

Rule 803(7). ABSENCE OF ENTRY IN RECORDS OF REGULARLY CONDUCTED ACTIVITY. Failure to record or
include a matter, which would ordinarily be included in a record of a regularly conducted business activity, offered to prove
the non-occurrence or non-existence of the matter
Testimony of the custodian or other qualified witness needed to lay proper foundation
Either the record from which the matter is absent must be introduced or someone with personal knowledge must testify that a
diligent search failed to disclose the matter in the memo, report, record, or data compilation
Example: Mechanic does complete check up on car and records what he does, but doesnt have record having checked the brakes. The
absence of this record can be admitted to prove the brakes were not checked.
Note: Exception can also be used to prove lack of authority no student ID so youre not a student here
Rule 803(8). PUBLIC RECORDS AND REPORTS. Public records are hearsay exceptions if the concern:
(A) Activities of the agency o Employment records, statistical material, budgets, maintenance records, etc. Records as to what the agency has to do
in order to run as an agency, not the records of the job they have to perform.
Example: FAA sends an investigator after a plane crash and he documents his investigation. You can admit thus
under (A) to show that an investigation was performed, but NOT the results of the investigation (possible can use
C).
(B) Matters observed by a duty imposed by law as to which matters there was a duty to report, excluding in criminal
cases, matters observed by cops as against private Ds (due to Confrontation Clause), OR
o Can use against government in criminal case
o NOTE: if cops are not investigating a crime (such as calibrating breathalyzers, filling out forms, etc.) this isnt a
confrontation clause issue and thus record is admissible even against a criminal D
(C) Factual findings resulting from an investigation made pursuant to legal authority against both civil and criminal
actions against the government
o Covers investigations of things like train or plain accidents.
o Includes determinations of what happened, how and why it happened and who caused it to happen
o

Records must have been made by a person with personal knowledge and under an official duty to do so
Copies of Reports are admissible (only need authentication with stamp of custodian)
Documents filed by private parties with government, such as tax returns, are not admissible under 804(8)

Lab reports = cant get the report in alone as a public record against a criminal D have to produce the person who conducted the test
o Allow retesting if the original tester is unavailable or even calling the original testers boss.
Factors of Factual Finding Admissibility under Rule 803(8)(C)
(1) The timeliness of the investigation;
(2) The special skill or experience of the official;
(3) Whether a hearing was held and the level at which it was conducted;
(4) Possible motivation problems;
(5) Whether it is reliable whether the facts, data, or opinions taken as a whole are of a type reasonably relied upon by experts
in the particular field in forming opinions or inference upon the subject.
Rule 803(9). RECORDS OF VITAL STATISTICS: Records in any form of births, fetal deaths, deaths, or marriages if made to
a public office pursuant to law are NOT excluded by the hearsay rule.
Example: If you are married by a rabbi, or give birth in a private hospital these documents are not issued by the government, but if the
law requires to be filed with a public office then they are admissible and do not have to be recorded by a public official.
Note: If issue of trustworthiness or reliability, record is usually admissible, but goes to weight of credibility.
Rule 803(10). ABSENCE OF PUBLIC RECORD OR ENTRY: same as 803(7); can prove the absence of a public record is
indicative of the fact that the activity did not occur where the normal procedure is to keep a record of the event.
Examples

36

(A) Record of fixing the brakes on a school bus cannot be found. This can be admitted to show the brakes were not fixed.
(B) The absence of a license is admissible to prove driving without a license, and other license activity.
Rule 803(16). STATEMENTS IN ANCIENT DOCUMENTS: statements in a document 20 years old or older are admitted
under this rule if they are found in a place where they are likely to be found and the authenticity of which is established.
Requirements:
(a) its condition creates no suspicion concerning its authenticity;
(b) Was in a place where it could be likely to be if authentic; AND
(c) Been in existence for 20 years or more
Examples: old newspapers in a library or old letters in grandmas attic
Rule 803(17). MARKET REPORTS, COMMERCIAL PUBLICATIONS. Market quotes, tabulations, lists, directories, or
other published compilations used and relied upon by the public or persons in certain occupations.
o
o
o

Has to be something used by the public generally


Must bring in hard copy, even if found on the internet doesnt have to be a published form
However, anything that is not numbers, statistics, or objective is not under this rule such as a credit report. (but it could be
used to show a person relied upon it and get in under effect on listener, not hearsay definition)

Examples: (a) catalogues of auto parts used by mechanics for price, stock quotes, life expectancy tables (b) weather reports (c) reports
the public has to rely on
Rule 803(18). LEARNED TREATISES: published periodicals or pamphlets on a particular subject that are considered an
established reliable authority to which is called to the attention of on cross or relied upon on direct.
If admitted, statements may be read into evidence, but may not be received as exhibits
If another expert refutes the experts use of the learned treatise through a different learned treatises, both are admissible and
up to the jury to balance both of them
Requirements for Authentication of actual article (not merely the entire publication)
(1) Have the expert witness testify it is a reliable authority;
(2) Judicial Notice judge can determine if reliable;
(3) Call another expert to testify it is reliable
Note: if you are cross examining an expert and you want to bring in other articles to contradict his opinion the way to do this by telling
the judge we have our own expert who will authenticate our article and ask to proceed subject to connection, then you just need to call
an expert to authenticate.
Rule 803(22). JUDGMENT OF PREVIOUS CONVICTION. Evidence of final judgment, entered after a trial or upon a plea of
guilty (but not upon a plea of nolo contender), adjudicating a person guilty of a crime punishable by death or imprisonment in
excess of one year, to prove any fact essential to sustain the judgment, but not including, when offered by the Government in a
criminal prosecution for purposes other than impeachment, judgments against persons other than the accused. The pendency
of an appeal may be shown but does not affect admissibility.
Note: Conviction can be used in a civil case only if the criminal case and the civil case arise out of the same transaction or occurrence.
Example: In a case of possession of stolen goods, cant use the conviction of the man who stole the property in the case against the
guy who bought the property. Conviction would be helpful to prove the goods were stolen, but since the D in the present case wasnt
represented in the first case this is not admissible against him.

Rule 804(a). HEARSAY EXCEPTIONS; DECLARANT UNAVAILABLE AS A WITNESS


For these exceptions, you need the declarant to be unavailable as defined under the rule. In all the previous exceptions, it did
not matter whether or not the declarant was available.
DEFINITION OF UNAVAILABILITY: A witness is unavailable if he fall into one of the following categories

37

1.
2.
3.
4.
5.

Witness unavailable because of an assertion of some sort of privilege from revealing the info
a. 5th amdt privilege or priest-penitent privilege; or
Persists in refusing to testify concerning the subject matter of the declarants statement despite an order of the
court to do so; or
Testifies to a lack of memory of the subject matter of the declarants statement; or
Is unable to be present or to testify at hearing because of death or then existing physical or mental illness or
infirmity; or
Is absent from the hearing and the proponent of a statement has been unable to procure the declarants
attendance (or in the case of a hearsay exception, under (b)(2), (b)(3), or (b)(4), the declarants attendance or
testimony) by process or other reasonable means.

A declarant is NOT unavailable as a witness if exemption, refusal, claim of lack of memory, inability, or absence is due to the
procurement or wrongdoing of the proponent of a statement for the purpose of preventing the witness from attending or
testifying.
Cannot prevent someone from testifying (through threats, etc), and then say they are unavailable.
Cannot use former testimony, unless witness/declarant is considered unavailable
Civil Case
Cannot require appearance of a non-local witness (out of state.)
Must exercise reasonable means to get the witness to testify (pay hotel bill, travel, possibly lost wages, etc.)
But, if the person refuses to come or wants more money, theyre unavailable
Criminal Cases
Can issue a subpoena anywhere in the US to require them to come in order to first procure statement
Must make a reasonable effort to locate the person though before they are declared unavailable under the rule.
NOTE: Unavailability is NOT limited to those 5 factors also can include: child who is too young to testify or too frightened of the
D, defense counsel, or courtroom, or someone who has been found to be incompetent is similarly considered unavailable. A kid in a
sex abuse prosecution is also unavailable where testifying in the presence of the accused creates the substantial likelihood of the kid
suffering emotional trauma.
Rule 804(b)(1). FORMER TESTIMONY. Testimony given as a witness at another hearing of the same or a different
proceeding, or in a deposition taken in compliance with law in the course of the same or another proceeding, is admissible as a
hearsay exception if:
1) the witness is unavailable as defined in 804(a); AND
2) the party against whom the testimony is now offered, or, in a civil action/proceeding, a predecessor in interest, had
an opportunity and similar motive to develop the testimony by direct, cross, or redirect.
Criminal: The witness has appeared someplace in which the D has had an opportunity for cross, and then the witness has become
unavailable at current trial. The previous testimony can be read in court, but cannot have the document admitted into evidence.
Requirements in order to read into the record Former Testimony when declarant is unavailable:
1. Party that the testimony is being offered against had the opportunity and similar motive to develop the witness testimony
Opportunity = full and fair chance to develop, not that the opportunity was utilized
o Thus, if the defense did not take advantage of it, still admissible because of the fact that the
opportunity was given
Similar motive = Identical issues
2. Testimony given at previous trial, hearing, deposition, or suppression hearing must be of the same subject matter that will
be testified to at trial.
Example: at a preliminary hearing for a motion in limine for suppression of evidence due to a 4th Amendment
violation the testimony given at that hearing wont be the same as what the witness will testify to at trial. So this
is not admissible.
D is not a party to a Grand Jury hearing so you can never admit Grand Jury testimony against a D, but it can be
admitted against the government.
Situations where witness may become unavailable

38

A. You can depose your own witness and give the other party the opportunity to cross if you are afraid the witness will not
be available at trial
B. At a preliminary hearing, if you are concerned your witness will not be available at trial, you must convince the judge a
full opportunity for cross is warranted to have his testimony subsequently admissible if the witness is not available
a. Example: at a probable cause hearing, the judge might determine that enough testimony was given to establish
probable cause and cut off the opportunity for cross.
Predecessor in Interest in a Civil Case: former testimony of an unavailable declarant can be admissible against a new P or D if he is
a predecessor in interest to the prior plaintiff or defendant.
A predecessor of interest is defined as someone who
o Is not a party to the prior lawsuit but is litigating the same issue; or
o Has a sufficient community of interest with the prior party such as a spouse, business partner, fiances, but note that
distant relatives probably do not qualify.
NOTE: In criminal cases, this provisions will never apply, you need the same D at all times
Offense against a child: childs testimony may be taken in a room outside the courtroom and be televised by 2-way closed circuit
television or may be taken by deposition recorded and preserved on videotape provided that the court
o Finds that the kid is unable to testify in open court in the presence of the D, because of fear;
o There is a substantial likelihood, established by expert testimony, that the kid would suffer emotional trauma from
testifying because of the presence of the D, or
o Kid suffers a mental or other infirmity, or
o Conduct by D or defense counsel causes the kid to be unable to continue testifying
Rule 804(b)(2). STATEMENT UNDER BELIEF OF IMPENDING DEATH. In a prosecution for homicide or in a civil
proceeding, a statement made by a declarant who believes his death is imminent is admissible if it concerns the cause or
circumstances of impending death.
Requirements of when statement admissible under belief of impending death
1. A settled hopeless expectation of death = declarant has to really believe they are dying;
2. Lost all hope of surviving;
3. Death is quite imminent;
4. Personal knowledge
Declarant does not have to be dead, he just has to be unavailable, and he could have died for some other reason.
Circumstances surrounding the minutes before the death is the way that the declarant realizes that he is going to die.
Admissible in all civil cases
Criminal Trials:
D must be charged with a homicide
The declaration may be offered not only in a trial of the homicide of the declarant but also the homicide of someone else
Rule 803(b)(3). STATEMENT AGAINST INTEREST: A statement which was at the time of its making so far contrary to the
declarants pecuniary or propriety interest, or so far tended to subject the declarant to civil or criminal liability, or to render
invalid a claim by the declarant against another, that a reasonable person in the delcarants position would not have made the
statement unless he believed it to be true.
A statement tending to expose the declarant to criminal liability and offered to exculpate the accused is not admissible
unless corroborating circumstances clearly indicate the trustworthiness of the statement.
NOTE: Statement is made by a witness, and not a party, and where the declarant is now unavailable
(A) Declarants statement that implicates him and a third party is NOT admissible against the 3 rd party in a criminal prosecution where
the statement was made to a law enforcement officer.
o The weed is mine, but the coke is my roommates. If A told this to the police, this is not admissible in a
prosecution against As roommate. Also, if A took more responsibility, it would not matter. So, if A claimed the coke
was his, but the weed was the roommates, thats still not admissible against the roommate.
(B) If Declarants statement implicates him and a 3rd party to someone who is NOT a law enforcement officer, the statement is
admissible with corroboration.

39

o
o

Reasoning: to prevent a criminal D from getting one of his friends to claim that someone else told them that the
committed the crime
Whats considered corroboration? Two ways:
(1) Corroboration that identifies the Defendant as actually committing the crime, either through
circumstantial or direct evidence.
(2) Criteria for credibility assessment. Factors include whether declarant has a motive to lie, character of
the declarant, who heard the statement, spontaneity of the statement, timing of the declaration, relationship
between declarant and D, etc.

Rule 804(b)(4). STATEMENT OF PERSONAL OR FAMILY HISTORY.


(A) Statement by a subsequently unavailable declarant regarding the declarants own birth, adoption, marriage,
divorce, legitimacy, ancestry or relationship by blood, adoption, or marriage, or other similar facts of personal or
family history is admissible even if the declarant had no way of acquiring such personal knowledge; OR
(B) A statement concerning the foregoing matters, and death also, of another person, if the declarant was related to
the other by blood, adoption, or marriage or was so intimately associated with the others family as to be likely to
have accurate information concerning the matter declared.
Reasoning: Family members are people related by blood, marriage, adoption, or any one so intimately associated with the persons
family that the information would be accurate.
NOTE: It does not matter when the statement was made, so even if the statement was made after the controversy that the case arose
out of it is still admissible
Rule 804(b)(6). FORFEITURE BY WRONGDOING: A statement offered against a party that has engaged or acquiesced in
wrongdoing that was intended to, and did, procure the unavailability of the declarant as a witness
A clear hearsay statement that is not listed under the not hearsay rules or the hearsay exceptions, but the declarant is now
unavailable because the D procured the unavailability of the witness by killing, threatening, paying, or getting one of his
friends to do it, and it was intended to make the witness unavailable to testify.
Burden of Proof if the government can establish that it is more probably true than not true (Rule 104(b)) that you did
something, does not have to be illegal, to make the witness unavailable (get rid of witness), you have forfeited the hearsay
objection and confrontation clause objection and the statement is admissible even though it is hearsay
Applies to Civil Cases and Criminal Cases
Rule 805. HEARSAY WITHIN HEARSAY: same as multiple level hearsay (Rule 805) Hearsay included within hearsay is NOT
excluded under the hearsay rule if each part of the combined statements conforms with an exception to the hearsay rule provided in
these rules.
Evens must be related and connected in order to be admissible together.
Examples:
1) Business record coming within the hearsay exception under Rule 803(6), which includes within it information supplied by an
informant not himself under a duty to provide such information
a. If the informations statement itself qualifies as a hearsay exception, for example, in an excited utterance, Rule
803(2), the record containing it is admissible provided among others that the person recording the excited utterance
was under a business duty to do so
b. If either the original statement or statement within which the second level statement appears is admissible as NOT
hearsay under Rule (d), provided that the remaining statement is so defined or qualifies as a hearsay exception, the
two statements are admissible
2) Murder of B. Defense is that B committed suicide. Evidence consists of W (a cop) to testify that he took A to the morgue to
identify Bs body. Upon being shown the body, A became hysterical and said B had told A that B intended to commit suicide.
a. Bs statement is admissible under Rule 803(3), then existing mental or emotional condition;
b. As statement admissible under Rule 803(2), excited utterance recalling the content of a statement, which may
have been made weeks or even months before the exciting occurrence, and that A was under no business duty, or
other duty, to remember such statement at all.
i. NOTE: Graham believes that in a situation like this, where there are multiple level hearsay statements, not
involving records of regularly conducted business activities (Rule 803(6)), or another hearsay exception,
which guarantees trustworthiness concerning accuracy of reporting, should resort to the residual hearsay
exception of Rule 807 applied to the two statements taken as a whole instead

40

Rule 806. ATTACKING AND SUPPORTING CREDIBILITY OF DECLARANT: The credibility of a declarant of a hearsay
statement or of a statement defined as NOT hearsay under either Rule 801(d)(2), (C), (D), or (E) may be attacked by any evidence,
which would be admissible for that purpose if the declarant had testified as a witness
If one side uses a hearsay statement, the opposing party can call the declarant as a witness and treat him as if he was on cross
examination, and thus he would be considered hostile in law and therefore be asked leading questions.
Thus, a declarants bias, interest, coercion, or corruption, his prior conviction of a crime (Rule 609), evidence of
character and conduct bearing on truthfulness (Rule 608), or his inconsistent statements (Rule 613) may be shown as
bearing on truthfulness.
o Similarly, if the declarants credibility has been attacked, it may be rehabilitated to the same extent as if he were a
witness.
Evidence of an inconsistent statement or conduct of the declarant is
o (a) not subject to any requirement that the witness be afforded an opportunity to deny or explain (Rule 613(b) does
not apply) and
o (b) can be introduced without reference to whether the statement was made prior to or after the statement admitted
into evidence or whether the prior statement admitted into evidence was made at a prior hearing or deposition
Example: W-Witness to a car accident claims D ran red light. Ds investigator interviews her and she says she didnt see anything. W
is deposed and tells P D ran red light, D does not ask about interview with investigator. Then W is unavailable at trial. P admits the
former testimony of the W from the deposition, and then D can admit the statement that W made to investigator to attack the
credibility of W as a prior inconsistent statement
Rule 807. RESIDUAL HEARSAY EXCEPTION. A statement not specifically covered by Rule 803 or 804, but having
equivalent circumstantial guarantees of trustworthiness, is not excluded by the hearsay rule, if the court determines that
(A)
Statement is offered as evidence of a material fact (i.e. very significant to the case);
(B)
Statement is more probative on the point for which it is offered than any other evidence, which the
proponent can procure through reasonable efforts (i.e. if cumulative evidence, inadmissible); and
(C)
The general purpose of these rules and interests of justice will best be served by admission of the
statement into evidence. (e.g. non-operative clause)
However, a statement may not be admitted under this exception unless the proponent of it makes known to the adverse party
sufficiently in advance of the trial or hearing to provide the adverse party with a fair opportunity to prepare to meet it, the
proponents intention to offer the statement and the particulars of it, including the name and address of the declarant.
Notice to adverse side gives it the opportunity to either (a) object or (b) meet/deal with the statement if it gets admitted.
Five Express Requirements, all of which must be determined by the court to have been satisfied
1. Equivalent Trustworthiness: most significant statement possess circumstantial guarantees of trustworthiness equivalent to
that of statements admitted under the specific exceptions, and the court focuses upon the circumstances that surround the
making of the statement and that render the declarant particularly worthy of belief.
a. Criteria = evaluation of the credibility of the in-court witness; whether the statements were made under oath;
assurance of personal knowledge of the declarant; practical availability of the declarant at trial for meaningful cross;
ad hoc assessment of reliability based upon the totality of the surrounding circumstances
b. Trustworthiness = declarants partiality, i.e. interest, bias, corruption, or coercion; the presence or absence of time to
fabricate; suggestiveness brought on by the use of leading questions, and whether the declarant has ever recanted or
reaffirmed the statement.
2. Necessity: must be necessary in the sense of being more probative on the point for which offered than any other evidence,
which the proponent may reasonably procure.
3. Material Fact: not only must the fact the statement is offered to prove be relevant, but must be proved to be of substantial
importance
4. Satisfaction of purpose of rules: little practical importance in determining admissibility
5. Notice: may be dispensed with when the need for the hearsay statement arises on the eve of trial or in the course of the trial,
if no prejudice to the opponent is apparent. Way avoid prejudice is to grant a continuance
NOTE: Exception is used in exceptional circumstances, and in reality rarely happens except for child sex abuse cases.
o Florida allows the statute to apply to kids, but not disabled and elderly due to policy reasons
o In child abuse cases, where there are other child abuse exceptions, and the kid is competent to talk and made the
statements to his mother, who appears to be reliable, and the kid is using langugage consistent with his age, and
using language to describe something which he would not otherwise know about, the statement is seemingly reliable
and is thus admissible

41

On the other hand, when there is a 12 year old girl and states something that she would be aware of i.e.
some man grabbed her boobs, may be admitted, but doesnt have the totality of the factors. In sex crimes,
the fact that he has done this before is admissible and goes to particularized of reliability.

42

THE CONFRONTATION CLAUSE


Requires a full and fair opportunity for cross-examination
Requires face to face confrontation with defendant and witnesses (exceptions in child molestation cases)
Only applies to out of court statements that are considered hearsay
Availability of declarant is irrelevant Declarant must testify in court, under oath, subject to cross
To determine whether statement is a violation of the Confrontation Clause
1. Is the statement testimonial or not? If NO, then not violation. If YES
2. Is the statement an accusatory statement? If NOT, then not violation. If YES
3. Is the statement defined as NOT hearsay or would no be admissible under Residual Hearsay Exception? If YES, then not a
violation. If NO, then the statement is inadmissible under the Confrontation Clause
Testimonial v. Non-Testimonial Statements:
A) Any statement that is testimonial any statement made as a result of a governmental examination or interrogation that is an
accusatory statement is governed by Crawford, and is inadmissible
1. Accusatory Statement: look to the purpose/intent of the government official for which the statement is to be used for
Accusatory statements are used to gather evidence for a criminal prosecution. (not alleviate an emergency)
2. 3 situations when statement is considered testimonial
1. Ex parte, in court testimony, or its functional equivalent
Material statements made in affidavits, custodial examinations, prior testimony, where D was unable to
cross, or similar pretrial statements that the declarant would reasonably expect to be used prosecutorialy.
2. Extrajudicial statements contained in formalized testimonial materials such as affidavits, depositions, prior
testimony, or confessions
3. Statements that were made under circumstances, which would lead an objective witness to reasonably believe that
the statement would be available for use at a later trial
B) Any statement that is non-testimonial IS admissible and DOES NOT invoke the Confrontation Clause.
Situations when statement is considered non-testimonial:
1. When statement is defined as NOT hearsay,
2. When statement meets the requirements of a hearsay exception
Examples:
1) If declarant calls cops and says Bob shot me, after the incident, this statement is objectively accusatorial, since it was
given to the police, whose primary purpose is to investigate and gather evidence, and therefore this statement is inadmissible
as a violation of the Confrontation Clause.
2) If Bob calls the cops and says, I shot Dave, it is not considered testimonial because it is an admission of a party opponent,
and thus defined as NOT hearsay. Therefore, statement is not a violation of Confrontation Clause, and therefore admissible.

43

REQUIREMENT OF AUTHENTICATION OR IDENTIFICATION


Rule 901. REQUIREMENT OF AUTHENTICATION OR IDENTIFICATION
(a) GENERAL PROVISION. The requirement of authentication or identification as a condition precedent to admissibility is
satisfied by evidence sufficient to support a finding that the matter in question is what its proponent claims must be enough
evidence, taken in the light most favorable to the proponent, to support a rational jury finding that the letter is genuine or the
photograph is accurate.
Authentication of things and identification of people represent a special aspect of relevancy
Proof of authenticity or identification may be by either direct or circumstantial evidence.
The laying of the foundation must be conducted, to the extent practicable, so as to prevent inadmissible evidence from being
suggested to the jury by any means, Rule 103(c)
Compliance with this rule does not guarantee that the evidence offered is admissible, for the offered evidence may still be
excluded because of some other bar to admission
(b) ILLUSTRATIONS. By way of illustration only, and not by way of limitation, the following are examples of authentication
or identification conforming with the requirements of this rule
(b)(1). AUTHENTICATION BY TESTIMONY OF WITNESS WITH KNOWLEDGE. The witness authenticates the item of
person based on personal knowledge of unique characteristics and testifies about that personal knowledge
A. Uniqueness: Personal knowledge acquired from any of the 5 senses may form the basis for the testimony of the witness.
May testify, for example, that the person who made a particular statement is the P, that the writing bears his signature, that he
observed someone write the letter, or that the exhibit is a piece of the bottle found on the floor immediately after it exploded.
Also the police can put identifying marks on the item, such as putting it in a bag with a label on it
o Examples: You ask the witness, what is this? Witness replies, this is the bat Harry hit me with, it is my bat, I
recognize it because it has this mark on it. Or the witness testifies that the person before him is Harry because,
Ive seen him before, and I can recognize him.
B. Chain of Custody: sometimes evidence does not have a unique characteristic or for some reason the evidence needs to be shown to
be in substantially the same condition as it was when it was found, you need the testimony by each individual having possession of the
item together with testimony by each individual that the object remained in substantially the same condition while they possessed it.
Missing Chain Link: it is not necessary to have every person testify, in most cases it will be enough to have the arresting
officer and the lab technician without all of the middle men, such as the evidence locker custodian. (unless the missing link is
in the middle, then need all of them to testify)
Change in Condition: if the condition of the item has changed this does not mean it will not be admissible. The change in
condition will need to be explained by the proponent and then the trier of fact will determine the items authenticity and
weight.
Missing Person: If a person is missing, evidence is still admissible if there is a reasonable probability that it was not altered,
and the fact that the person is missing would go to the weight the jury gives it.
Example: cap takes the 12 baggies of coke, and puts them into an evidence bag and seals it. This first cop can give it to a second cop,
who gives it to a custodian to put it in a safe. Eventually, the evidence bag gets to the forensic department technician opens the bag
and does the testing on the substance (or bullet striations). Then he puts it back in the bag, seals it, and puts it back in the sage.
Since the gap in custody is in the middle, need the first cop, the arresting cop, and the technician
(b)(2). NON-EXPERT OPINION AS TO THE GENUINENESS OF HANDWRITING. A witness who is not an expert can give
his opinion authenticating a writing or signature if he is sufficiently familiar with the handwriting of the alleged writer,
providing the familiarity was not acquired for purposes of the litigation.
This means that the witness cannot be shown handwriting samples in anticipation of testifying must be based on the
witnesss own prior perception of the handwriting say through correspondence or watching the person write.
1. Comparison: On direct, cannot ask the witness to compare the handwriting of others; BUT, on cross can show the
witness several samples and ask him to pick out which handwriting is the one of X
2. Time: what if you only knew the person for a few hours and observed his handwriting, or you saw his handwriting
30 years ago, is the rule satisfied?
YES, testimony would be admissible because viewing the evidence most favorable to the proponent a
reasonable juror would believe it is what it is purported to be. However, jury can determine the weight of
credibility to be given to the evidence

44

(b)(3). COMPARISON BY TRIER OR EXPERT WITNESS. The trier of fact and/or an expert witness may base an opinion as
to the authenticity upon a comparison between the questioned piece of evidence and an exemplar as long as the authenticity of
the exemplar was sufficiently established.
Ballistics, DNA, Handwriting analysis, Fingerprints, all fall under this rule.
1. Cross Exam: Expert may be required to pick a genuine item from a group of false samples, but must have both the
actual item connected with the offense and an exemplar.
2. Exemplar: Item you tie into a particular event that you use for comparison purposes, this is what you compare the
item that is actually connected to the offense
This often requires a chain of custody analysis to authenticate
(b)(4). DISTINCTIVE CHARACTERISTICS AND THE LIKE: An item is authenticated through its appearance, contents,
substance, internal patterns, or other distinctive characteristics, taken in conjunction with circumstances.
Reply Doctrine:
1. Once a letter, email, phone call, telegram, etc is shown to have been made or sent, then the letter, email, phone
call, telegram, etc. in reply is authenticated
2. Once a letter, email, phone call, telegram, etc. has bee referred to in a communication as to a previous
communication, authenticated as to both conversations
Example: A letter that has information that only the purported sender would know is sufficient to authenticate the letter as
from the sender under this rule.
(b)(5). VOICE IDENTIFICATION: identification is adequate if the authenticating witness has sufficient familiarity with the
speakers voice.
A witness may testify in the form of an opinion identifying a voice, whether heard firsthand, or through mechanical or
electronic transmission or recording, based upon hearing the voice at any time under circumstances connecting it with the
alleged speaker.
Can acquire the familiarity for purposes of litigation and can acquire knowledge after the incident (unlike handwriting)
A. Self Incrimination: D may be compelled to speak before a witness without violating his 5th amendment privilege even if asked to
use words purported to have been uttered by the person who committed the crime of which he is charged. He may also be
compelled to speak into a recorder so the witness may compare this with what he heard on another occasion to determine if the
voices are the same.
B. Sound Recording: are only to be played in open court in the presence of the jury and judge and are only authenticate if a proper
foundation is laid, including identification of the speakers. A person who heard and participated in the conversation may
authenticate it through listening to the recording, however, sometimes such a witness is not available so you must lay a
foundation:
1. Capability of the recording device
2. Competency of the operator
3. Proper operation of the recording device
4. Preservation of the recording with no changes or deletions
5. Identification of the speakers
C. Inaudible, incomprehensible, incomplete, or questionable may be excluded under Rule 403
D. Transcripts: accurate transcripts of the sound recording may be admitted to assist the jury in listening to a tape recording, however,
the tape recording is the evidence and the recording controls in case of discrepancy
(b)(6). TELEPHONE CONVERSATIONS. Communications by telephone do not authenticate themselves. A mere assertion by
the speaker as to his identity, being hearsay, cannot be taken as a sufficient showing of his identity. However, the
aforementioned authentication techniques for voice, including the reply doctrine, may be used.
A. Outgoing Calls: calls made by the authenticating witness, may be authenticated by showing W made a call to the number
assigned by the phone company to a particular person and the circumstances show that the person who talked on the other end
was in fact the person the caller was trying to reach.
Call to business: Call was made to the phone number listed for the business and the conversation was about business
generally transacted on the phone
B. Incoming Calls: self-authentication alone is not enough, you need to use either the voice recognition, reply doctrine, or caller ID.
C. Emails: not sufficient to identify the particular individual

45

(b)(7). PUBLIC RECORDS OR REPORTS: evidence that a writing authorized by law to be recorded or filed in a public
office, or a purported public record, report, statement, or data compilation, in any form, is from the public office where items
of this nature are kept is sufficient to support a finding that the writing or record is what is purports to be.
Evidence is sufficient to support a finding that the item is from the public office where such items are kept, can be
testimony from a person from the office or other person with personal knowledge
Can get the original by subpoena
Not limited to the records that are available for public inspection
Items that are filed by those outside the government, but filed with the government. I.e. recorded deed
Cane be an admission of a party-opponent, but have to prove that it was sent by the person and have to authenticate it by
signature.
Not automatically admissible for its truth.
Hearsay Exception: Under 803(9), recorded or filed writings related to vital statistics made to a public office pursuant to
requirements, record is admissible. Under 803(14), records of documents affecting an interest in property recorded in a
public office pursuant to statute authorizing the recording of such documents in such public office
Certified Copies under other Rules: Can get in a duplicate under Rule 1003; self-authenticate certain public records and
certified copies under 902(1),(2), and (4); Under 1005 certified copies admissible with testimony of a person that he has
compared the copy with the original and it is the same
(b)(8). ANCIENT DOCUMENTS OR DATA COMPILATION. A document or data compilation, in any form, is authenticated
by evidence the exhibit
(A) is in such condition as to create no suspicion concerning its authenticity;
(B) Was in a place where, if authentic, would likely be, and
(C) Has been in existence for 20 years or more at the time it is offered
If the document has been recorded, the recording is evidence of age
If it is authenticated and over 20 years old on its face, it can come in for its truthself-authentication
Examples:
1.

Miami Herald Business Section (self-authenticating that it is the actual document), but brought into court to prove
that the weather was actually what it has been claimed to be on that date. Yes, this would be admissible since people
rely on the newspaper for weather.

2.

Hearsay Problem: Miami Herald Story about someone telling a reporter something. No, the report is not admissible,
and hearsay (no exception) as to the story. If we are reading the words, then will have to address the hearsay
problem for the truth of the matter asserted.

(b)(9). PROCESS OR SYSTEM: the results of a process or system are authenticated when you show that
(1) Process or system produces an accurate result when properly operated and correctly employed; and
(2) Process or system was in fact properly operated; and
(3) System or process was in fact so employed and operated.
In most cases the results are admitted without authentication of the process or system, but if available must be made available
to the opposing party in advance of the trial.
Examples: evidence describing process of creating x-rays, photographs, tape recordings, videotapes, computer generated
records, radar detectors, or scientific surveys
Computer Records: Do not have to show the accuracy of the computer records, just introduce a record that comes from the
ordinary business; assumed that computer works correctly. Opposing side has burden to make an argument that it is
unreliable.
(b)(10). METHODS PROVIDED BY STATUTE OR RULE: Any method of authentication or identification provided by Act of
Congress or other rules prescribed by the SCOTUS pursuant to statutory authority is sufficient, but are not intended to be
superseded by the Federal Evidence Rules.
RULE 902. SELF AUTHENTICATION. Extrinsic evidence of authenticity as a condition precedent to admissibility is not
required with respect to the following 12 subsections.
Do not need any more evidence to authenticate it than the mere item itself. BUT, the rules against hearsay and the Original
Writing Rule still apply and must be satisfied.
In order for the item to be self-authenticating it must be presented into evidence, witness cannot just testify about it Must
be brought into court.

46

Read the words on the page to establish the fact that it is a document, but what about the content? Hearsay, but under
803(8)(a) or 803(8)(b) may be admissible.

902(1). DOMESTIC PUBLIC DOCUMENT UNDER SEAL. A document bearing a seal purporting to be that of the US, or any
state, district, commonwealth, or territory, or of a political sub-division, department, officer, or agent thereof is admissible
provided there is a signature on the document purporting to be an attestation or execution.
Neither the seal or the signature have to be authenticated.
Notarized documents do not fall under this rule.
Rarely used
902(2). DOMESTIC PUBLIC DOCUMENTS NOT UNDER SEAL: Public documents of any entity provided in 902(1), which
is signed, but not bearing a seal, is self-authenticating if a public officer with a seal and having official duties in the same
district, subdivision, or office as the signer certifies under his seal that the signer has the official capacity and the signature is
of that person with the capacity.
902(3). FOREIGN PUBLIC DOCUMENTS. For a foreign public document to be certified, must show (1) Document has been
executed or attested to by an official authorized by the laws of that country to do so, and (2) Must be accompanied by a final
certification as to the genuineness of the signature and official position of the signer (document certified by custodian)
If reasonable time has been given to all parties to investigate the authenticity and accuracy of the official documents, court
may deem it authentic without final certification.
Rule is in regards to a different country (not a state)
Designed to meet the practical problems in obtaining properly authenticated public documents from abroad by abolishing all
unnecessary procedural requirement. (follows the c/l concept)
902(4). CERTIFIED COPIES OF PUBLIC RECORDS: a copy of a public document provided with a seal is self-authenticating
if the custodian certifies the correctness of the document.
Requirements:
(1) Document purports to be a copy of an official record;
(2) Certificate is attached to the copy;
(3) Certificate says that the signatory is a public custodian of official records;
(4) Certificate says that the document is a true and accurate copy of the original record; and
(5) Certificate bears a presumptively authentic signature and/or seal

This rule applies only to official domestic records or reports, not to documents of public office.
Also admissible under 1005, but hearsay as to content
Question of who has access to these records? Need the capacity to certify by a person who possesses a seal issued by the
State and specifically authorized for these purposes

902(5). OFFICIAL PUBLICATION. Books, pamphlets, or other publications purporting to be issued by public authority are
self-authenticating.
Does not confer admissibility upon the contents of all official publications; merely provides a means whereby their
authenticity may be established for purposes of admissibility
902(6). NEWSPAPERS AND PERIODICALS: Printed materials purporting to be newspapers or periodicals are selfauthenticating.
Still must bring into court
Only authenticates what it is, but not the content contained therein, and no hearsay exception for the content of the
newspaper/periodicals, so likely content will be inadmissible
902(7). TRADE INSPECTIONS AND THE LIKE: Inscriptions, signs, tags, or labels purporting to have been affixed in the
course of business and indicating ownership, control, or origin are self-authenticating.
Although can offer this evidence to the jury, no guarantee that the jury will believe its truth.
If unable to bring the item in, such as a bus, must make an argument as to why it cannot be brought to court
902(8). ACKNOWLEDGED DOCUMENTS: Documents accompanied by a certificate of acknowledgement executed in the
manner provided by law by a notary public or other officer authorized by law to take acknowledgements are selfauthenticating.
There doesnt necessarily have to be a notary seal its up to state law whether or not its required.

47

Document should include that the person executing the document has
o 1. Come before a public official or notary authorized to take an acknowledgement;
o 2. That his identity was known to the said official or notary or he produced sufficient documents of identification,
and
o 3. That he stated to the official or notary that he executed the document of his own free will
Contrast to a Jurat: does not establish the identity of the person being sworn, and thus does not come within this Rule, and not
self-authenticating.
o Still notarized, but where the notary has made no statement as to whom the person is that he purports to be, i.e. when
selling a car, sign name then go through motor vehicles. Sworn to me by someone claimed to be John Smith.
o Florida: Can have an acknowledgment in front of a notary (1008) person came before me and upon her free will
signed the document, and is either personally known to me or has produced he following document (drivers
license) to prove his/her identification.

902(9). COMMERCIAL PAPER AND RELATED DOCUMENTS: Commercial paper, signatures thereon, and documents
relating thereto, to the extent provided by general commercial law are self-authenticating.
A formal certificate of protest (such as a postmark showing that the document was refused) is admissible.
902(10). PRESUMPTION UNDER ACTS OF CONGRESS: Any signature, document, or other matter declared by Act of
Congress to be presumptively or prima facie genuine or authentic are self-authenticating.
Under 902(11) and (12), all business records, foreign or domestic, can be authenticated by the custodian of the records. All the
custodian has to do is sign the declaration noted in the textbook has to be signed and notarized to authenticate, and must be a threat
of penalty of perjury (which must be noted in the declaration).
NOTE: You still need a hearsay exception, such as the business records exception.
As long as all the requirements are met, you dont need the custodian to testify. The written declaration is enough.
902(11). CERTIFIED DOMESTIC RECORDS OR REGULARLY CONDUCTED ACTIVITY: The original or a duplicate of a
domestic record of regularly conducted activity, that would be admissible under Rule 803(6), if accompanied by a written
declaration of its custodian or other qualified person, in a manner complying with any Act of Congress or rule prescribed by
the SCOTUS, certifying that the record was
(A) made at or near the time of the occurrence of the matters set forth by, or from information transmitted by, a person
with knowledge of those matters;
(B) kept in the court of the regularly conducted activity; and
(C) was made by the regularly conducted activity as a regular practice.
A party intending to offer a record under this Rule, must provide written notice of that intention to all adverse parties, and
must make the record and declaration available for inspection sufficiently in advance of their offer into evidence to provide an
adverse party with a fair opportunity to challenge.
902(12). CERTIFIED FOREIGN RECORDS OF REGULARLY CONDUCTED ACTIVITY: Certified same requirements as
902(11) only applies to foreign records.
RULE 903. SUBSCRIBING WITNESSS TESTIMONY UNNECESSARY: If a person is a subscribing witness, which means
that he is attesting put his name on the document for the purpose of making a statement that the document was executed by
the purported maker (a witness to a signature), he doesnt have to testify to authenticate unless the jurisdictions laws require
it.
THE ORIGINAL WRITING (BEST EVIDENCE) RULE
Original Writing Rule (Best Evidence Rule): Requires production of an original of a document, photograph, recording, etc.
when the contents are at issue.
If theres no good faith dispute of the existence or content of the document, admission of a duplicate isnt clearly
erroneous, but merely harmless error;
Thought better to retain the requirement rather than it being secondary evidence
Whenever you provide an original, satisfy this rule, whether it is required or not.
An original or show that none of the originals are available, then can show something such as a draft.
When you copy an original, say in a Xerox machine, then have a duplicate, and not an original (will be able to admit it under
a separate rule). But, can press print unlimited amount of times and will be considered an original.

48

Example: (a) the dispute is over a clause in the K if there is a requirement in the K and as part of the case, produce an original of the
document you are claiming is in dispute. (b) Letter as a form of an admission (c) recordings on a cell phone; emails; text messages;
etc.
Rule 1001. DEFINITIONS:
(1) WRITINGS AND RECORDINGS. Letters, words, or numbers written down by handwriting, typewriting, printing,
photostating, photographing, magnetic impulse, mechanical or electronic recording, or other form of data compilation.
(emails, computer storage, letters, Ks, etc.)
(2) PHOTOGRAPHS. Include still photographs, X-ray films, video tapes, and motion pictures
(3) ORIGINAL.
Writing or record = the writing or record itself or any counterpart intended to have the same effect by a person
executing or issuing it.
Photograph includes the negative or any print therefrom.
i. If you show a slide or a print from the negative, admissible under the Rule
ii. In contrast, with digital photos, because they are located in a computer, the photograph is actually data
stored in a computer, and thus follow below conditions
If data is stored in a computer or similar device, any printout or other output readable by sight, shown to reflect the
data accurately is an original
i. Any printout, which would be on a screen, is an original. Take the data, put it on a disk, and then display it
on a computer to a jury, this projection would constitute an original. Thus, if you print out a hard copy, or
show it on a screen, it is an original
Intent of Parties Controls: For writings/recordings the intent of the parties controls as to whether the document is an original or a
copy. Think of carbon copies of a form K (such as a car rental). The customer gets a copy, the bank gets a copy, and the merchant gets
a copy. There are 3 originals here. Thats the intent of the parties. Something is a duplicate if it is not intended by the parties to have
the same effect as the original.
Copies are equivalent as originals if intended by the parties. Signed document can be Xeroxed say 26 times, and still be
considered an original if the people are given the same thing, and it was intended by the parties as being 26 originals (or
could copy it 27 times and through out the ink copy).
Can also be executed in part via fax machines, except for real estate documents (deeds: need real original ink document in
order for it to be recorded).
Rule 1002. REQUIREMENT OF ORIGINAL. To prove the content of a writing, recording, or photograph, the original
writing, or photograph, is required, except as otherwise provided by Rules or by Acts of Congress
Thus, for the rule to apply, the contents of the writing, photograph, or recording must be sought to be proved, or in dispute. If
the content is not in dispute, then the original is NOT required.
When the Original Writing Rule may apply
A. Event in question is solely in the form of a writing, recording, or photograph. When this is the event, the Original Evidence
Rule is applicable; OR
a. When dispute over written K, content will always be in question
b. Photograph of celebrity was printed, and dispute over defamation
B. Where there is a memoralization of a testified-to event, and trying to prove the matter through the memoralization.
Example: D gives an oral confession to Cop that is taped and typed by a stenographer. Then, the typed confession is signed by the D.
The Cop can testify about the oral confession without admitting the recorded or typed confession. However, if he wants to testify
about the contents of the taped or recorded confession, they must be admitted as evidence and authenticated. This is because the oral
confession is an event that is independent of the recording or writing, and therefore the original writing rule does not apply. BUT, once
one side wants to introduce the recording or writing, then the best evidence of this original recording or writing, and then the Rule
would apply.
Three confessions
o 1. Orally to the cop (no memoralization);
o 2. Orally, in the presence of the officer and court reporter (memoralization typed by the court reporter).
o 3. Typed Confession (Original Writing Rule is applicable, and this written document must be admitted in court or
proved to be unavailable. Cannot show a duplicate, need an original.)

49

Rodney King Beating: Original Writing Rule does not come into play because of the availability of live witnesses, but since the issue
was memoralized in a video, can use the original video to prove the issue.
What if someone wants to testify to the content of the recording? Cannot testify as to the original of the recording unless
lawyer admits it into evidence, unless the recording is unavailable and account for the unavailability.
1002.2. FORMER TESTIMONY AND DEPOSITIONS. Prior testimony, despite being written down or recorded, can be
attested to by any witness who heard it. Proof of the prior testimony can be shown without the writing or recording because it is
independent of the memoralization of an oral statement
Methods to prove the prior testimony or deposition without the Original
1) By testimony of a firsthand observer, provided the court is satisfied he can remember and purport all that was said even if he cant
remember the exact words.
2) By testimony of a firsthand observer who has refreshed his recollection through the use of a memorialization, like the transcript
3) By the official stenographers notes or transcript which falls under the public record hearsay exception
4) By the notes of an observer if they meet the standards of recorded recollection hearsay exception
1002.3. DOCUMENT SPEAKS FOR ITSELF: An objection the opposing party may make when the original writing is being
offered into evidence, and in the case of a document the witness begins to read it to the jury.
If the opposing party thinks the jury may place undue emphasis on the evidence by being read by the witness, likely to object.
o Example: When a cop reads the confession of a D.
Since it is an admission of a party opponent, would be able to be admissible into evidence under hearsay
exemption, by cops testimony
Here, confession admissible in different way; the jury is able to read it by themselves as evidence
1002.4. CROSS ON WRITING: Cross-Examination on Writing: On occasion, a party will make an objection to a witness
being crossed as to a writing or recording, after they have already admitted that it was theirs. However, this is not a proper
objection because counsel (who is taking cross) will want to test the memory of the witness and not question about the contents of the
writing or recording.
1002.5. PHOTOGRAPHS. Photos, x-rays, videos, and motion pictures are subject to the Original Writing Rule when their
contents are at issue.
This means the contents of the photos must have independent probative value (demonstrative), and not merely be illustrative
of a witnesss testimony.
o Cases where this rule will apply: copyright; defamation; invasion of privacy; video of a bank robber.
o Note: often does not apply, because substantively the photograph will be admitted due to the requirement of
authentication when admitted through other rules.
1002.6. CHATTELS BEARING INSCRIPTION. Technically the Rule applies to chattels bearing inscriptions such as the
McDonalds arches or the name inscribed on the side of a bus, but for practical reasons the court accept secondary evidence of
its contents.
Rule 1003. ADMISSIBILITY OF DUPLICATES. A duplicate is admissible to the same extent as an original, unless a genuine
question is raised about the authenticity of the original or it would be unfair to admit the duplicate instead of the original.
Genuine questions as to the accuracy of the duplicate would also call for the original.
If copy an original as part of the ordinary course of business, will not be considered an original of the K, but can be
admissible under the Business Record rule
Rule 1004. ADMISSIBILITY OF OTHER EVIDENCE OF CONTENTS: Production of the original will be excused and use of
secondary evidence allowed if all of the originals are:
1) lost of destroyed;
2) not obtainable;
3) in the possession of an opponent;
4) relate to a collateral matter (if the matter is collateral, not important to the case, you can just have someone testify)
NOTE: There are no degrees of secondary evidence; thus, once production excused, and evidence is admissible. But if you produce
crappy secondary evidence your adversary will produce something better, so the adversary system forces the production of the best
degree of secondary evidence.

50

Rule 1005. PUBLIC RECORDS. When you have a certified copy of the original then you have satisfied the Original Writing
Rule and you do not have to produce the original or show the original is unavailable. Rule also satisfied with and testimony of
custodian that it is a true and accurate copy.
Rule 1006. SUMMARIES: Requires the use of an expert witness, and governs how to conveniently present to the jury what
otherwise would be inconvenient.
Example: A ton of boxes of files of documents not admitted into evidence, but were relied upon and are admissible. Instead of
admitted every document, can present a summary to the jury.
Requirements
1) The documents used to create the summary must be made available to the opposing party in advance of trial
2) The chart or summary is inadmissible if it relies on any documents that are inadmissible
3) The char or summary is admissible, but the report of the P or D prepares to create the chart or summary is not admissible because it
is hearsay and prepared in anticipation of litigation.
Rule 1007. TESTIMONY OR WRITTEN ADMISSION OF A PARTY: The contents of a writing, recording, or photograph
may be proved without accounting for the non-production of the original through the testimony or deposition of the party
against whom it was offered or by his written admission.
Rule 1008. FUNCTIONS OF COURT AND JURY: If an issue is raised regarding the following, the factual determinations
concerning the merits of the controversy are to be made by the trier of fact while those concerning merely the administration
of the Rule is to be decided solely by the court.
Jury
(a) whether the asserted writing ever existed; or
(b) whether another writing, recording, or photograph is the Original (Battle of Originals): or
- when one party says, I have the original and the other party says no, I have the original the jury decides which is the
original
(c) Whether other evidence of contents correctly reflects the contents
Judge
(a) What is an original; and
(b) whether the original is unavailable
Paul Newmans The Verdict: D says he has the original and it says 9. P says he has a copy of the original that says 1, and testimony
from the nurse who helped create the document.
The court decides whether there is sufficient evidence to show that the Ps copy could have been an original, thereby allowing
the secondary evidence of the copy and the testimony of the nurse in.
The jury will then decide if the original was altered or not.

51

OPINION TESTIMONY OF LAY & EXPERT WITNESSES


Rule 701. OPINION TESTIMONY BY LAY WITNESS: If the witness is not testifying as an expert, the witnesss testimony in
the form of opinions or inferences is limited to those opinions or inferences which are
(a) Rationally based on the perception of the witness,
(b) Helpful to a clear understanding of the witnesss testimony of the determination of a fact in issue, and
(c) Not based on scientific, technical, or other specialized knowledge within the scope of Rule 702.

Provides for the admissibility of opinions or inferences of a witness not testifying as an expert
Assumption that everyone is a competent witness except in diversity cases where the rule of decision is otherwise
Excluded from disclosure requirement
Opinions that ordinary people can make based off of common experiences
Absolute certainty is not required. Statements such as I believe are admissible.
o Anything based on their perception (personal knowledge), thats helpful to the understanding of their testimony or
a fact at issue
o Has to be something the average person would be able to assess
o Can be an accurate total impression, although witness is unable to account for all of the details
Examples: description of a person (hair, weight, eye color, hair color, height, etc.); describing a person as running to a getaway car;
skipping; a comparison for what something smelled like or looked like
SKILLED LAY OBSERVER: Lay witness can also be a skilled lay observer needs to have the ordinary ID capacity of the average
person PLUS specific experience that makes him uniquely able to make a comparison
Examples:
A. Tile layer has a unique experience of being familiar with laying tile, so he can testify to comparisons about tile due to
his past experience
B. Friend of the D who heard him speak on numerous occasions that can ID the Ds voice. Its not that the friend has a
unique ability to recognize voices, just has previous experience that puts him in a special position to recognize the Ds
voice.
C. Lay witness can testify to gunshot noise or speed of a fleeing vehicle, just have to lay foundation that the witness has the
necessary basis for comparison.
Rule 702. TESTIMONY BY EXPERTS: Testimony providing scientific, technical, or other specialized knowledge, in the form
of an opinion or otherwise, is admissible only if
1. Knowledge is based upon sufficient facts, data, or opinions;
2. Sufficient assurances present that the explanative theory being employed to create the knowledge produces a correct result to
warrant jury acceptance;
3. Explanative theory has been applied in accordance with proper prodedures;
4. Witness is qualified as an expert by knowledge, skill, experience, training or education to provide such knowledge; AND
5. Knowledge will assist the trier of fact to understand the evidence or to determine a fact at issue

Experts can either give opinions or just scientific information and let the jury draw their own opinions
Experts dont have to have formal degrees, just sufficient knowledge
You can pay an expert for his time, but not on contingency basis. NOT paying for his testimony.
Can us an expert as long as it will assist the jury, even if fact considered common knowledge

Disclosure Requirement: Required to disclose as a witness who will give testimony regarding scientific, technical, or other specialized
knowledge that will be revealed at trial, which eliminates the element of surprise. If fail to disclose, the particular witness is precluded
from testifying about the scientific, technical, or other specialized knowledge, as well as any other witness during trial !! Lay
witnesses were excluded from this rule
Qualification of Expert Witness An expert must be shown by the party calling him to possess scientific, technical, or other
specialized knowledge.
A. Judge determines qualification in his own discretion under Rule 104(a), and may require a voir dire examination by opposing
counsel into the experts qualifications to express a particular opinion.
Where the witness is unqualified, he may be excused before presenting the opinion to the jury.
Where witness is qualified, preferable that the court not advise the jury as to the voir dire exam

52

B. An expert may be qualified as an expert by virtue of any one factor, or upon a combination
(1) Knowledge;
(2) Skill;
(3) Experience;
(4) Training; OR
(5) Education
NOTE: specific degrees, certificates of training or membership in a professional organization are not required. But, obviously, a
certain education will be required if the knowledge necessary is something like medical info that only an MD would know (such as
orthopedic surgeon). The more complicated the area, the more formal training will be necessary.
Rule 703. BASES OF OPINION TESTIMONY BY EXPERTS: In addition to the scientific, technical, or other specialized
knowledge forming the basis of the witnesss expertise under 702, an expert may base his opinions or inferences on
(1) Firsthand observation of facts, data, or opinions perceived by him before trial;
(2) Facts, data, or opinions presented at trial as by a familiar hypothetical question or by having the expert attend the
trial and hear the testimony establishing the facts, data, and opinion relied on, and
(3) Facts, data, or opinions presented to the expert outside of the court other than by his own direct perception

An experts basis of knowledge is NOT hearsay.


o If a doctor actually looks up something in a book, thats hearsay, but if he just knew it based on his education, then
thats not treated as hearsay (even though he must have gotten from a book or from a teacher at some point).
Expert can rely on the opinion of someone else to form his opinion, i.e. a doctor can rely on the opinion of the radiologist to
determine that a patient has a broken leg.

A. Use of Hypos eliminated: Under c/l, if you didnt admit in evidence every fact used by the expert in forming his opinion, you
had to ask a hypo. Now, you dont have to use a hypo you can admit an experts opinion even if you havent admitted into
evidence every piece of data used by that expert in the forming of his opinion, which allows the omission of the discussion of
certain factors not helpful to the case
B. Do not have to lay the basis for the experts opinion at all; just have to show that the expert is qualified.
o BUT, Judge can require foundation would have to determine that the probative value in helping the jury evaluate
the experts opinion outweighs the prejudicial effect of admitting otherwise be hearsay
C. On cross, OC isnt limited to challenging the experts opinions on the facts of the case; can challenge the experts expertise.
This is a back door way to admit evidence; open the door to a proponents rebuttal with information that was reasonably
relied upon and thus serves as an equivalent to a hearsay exception
Substantive Evidence Facts, data, or opinions reasonably relied upon by an expert witness are NOT by virtue thereof substantive
evidence; reasonably relied on facts, data, or opinions constitute substantive evidence only if otherwise admitted into evidence.
Trial court must balance the probative value of the facts, data, or opinions in assisting the jury to evaluate the experts opinion
or inference on the one hand, against the risk of prejudice resulting from the jurys potential misuse.
o Considerations: (a) proponents wish to take the sting out of the adverse partys anticipated attack, and thereby
prevent the jury from drawing an unfair negative inference; and (b) probable effectiveness of a limiting instruction.
If the otherwise inadmissible facts, data, or opinions are disclosed, the judge must give a limiting instruction upon request informing
the jury that the underlying facts, data, or opinions are to be considered solely in evaluating the expert witnesss basis for his opinion
and ARE NOT to be used as substantive evidence
RULE 705. DISCLOSURE OF FACTS OR DATA UNDERLYING EXPERT OPINION: The expert may testify in terms of
opinion or inference and give reasons therefore without first testifying to the underlying facts or data, unless the court requires
otherwise. The expert may in any event be required to disclose the underlying facts or data on cross.
Two requirements to satisfy the court that the objects the expert is using to form his opinion are valid:
1. Must be of a type customarily relied upon by experts in the field; AND
2. Must have sufficient trustworthiness to make reliance reasonable.
A. In order for an expert to form an opinion based on facts/data no admitted into evidence, it must be the type of facts reasonably
relied upon by experts in that field.

53

Reasonable: Opinion of one doctor relied upon by another in the field


NOT Reasonable: Accident reconstruction expert who uses the statements of bystanders at the scene if forming his
opinion [hearsay statement; doesnt meet the hearsay exceptions]

B. Reliance is reasonably only if the facts, data, or opinions possess trustworthiness identical or equivalent to that possessed by
hearsay statements admissible pursuant to any hearsay exception are apparently contemplated.
This is the restrictive approach in comparison to the liberal approach, which finds customary reliance by experts in
the field alone sufficient.
Arson problem =
C. BUT, the facts that the expert relied on cannot be told to the jury; avoid having and end-run around the hearsay rules. Only time
the basis of the opinion (i.e. method of testing) is introduced is under cross.
o If the proponent wants it admitted, he has to find a hearsay exception that applies.
o Exception is where the judge requires the basis of the opinion, where the probative value outweighs the prejudicial
effect. Here, the jury would be able to say analyze the complete method of basis.
o NOTE: NO State follows this provision
Example: Doctor sends patient to a special bone density clinic to see if the car accident actually caused the broken arm or if a fragile
bone disease caused it. If the clinic sends the doctor a report, thats a business record and its in. If the clinic just calls the doctor and
tells him what they found, then the doctor can testify that, in his opinion, the accident caused the broken arm, but cannot say that this
is based on the result of the bone density test UNLESS you have the person who performed that test come in and testify OR can
comply with the authentication rule by requesting a copy by the custodian.
RULE 704. OPINION ON ULTIMATE ISSUE: Testimony in the form of an opinion or inference by lay or expert is admissible
as to an ultimate issue. Cannot render opinion as to questions that are matters of law for the court, advise the jury to decide
the question in a particular way, to offer a speculative opinion upon an unsubstantial basis, or to testify as to whether another
witness is telling the truth.
Exceptions
1. Cannot tell the jury who should win (who is right)
a. Cannot say whether P or D is right or wrong or whether D is innocent or guilty
2. Cannot say who is or is not truthful
a. Slight exception is child abuse commendation syndrome where an expert says not that the kid is actually telling the
truth, but that his behavior is consistent with a person who has been abused.
Rule 704(b). Expert cannot testify on the ultimate issue in a criminal case regarding whether or not a D possessed the mental
state, which is an element of the crime of an affirmative defense. Such ultimate issues are matters for the trier of fact alone.
BUT, an expert CAN testify to ultimate issues in criminal cases on anything else that constitutes an element of the crime or
affirmative defense, as long as its not about the mental state of the D.
o Cannot ask shrink if the D was able to understand the consequences of his actions, but CAN ask if D had a mental
condition and what his symptoms were.
o Some courts also say you cannot ask an expert to testify on the ultimate issue of intent to distribute in a drug
possession case or knowingly evading taxes in a tax evasion case.
o To get around this, you ask the expert if a hypothetical person who has X amount of cocaine would have the intent to
distribute them (focus is on the amount of the drugs, not the actual mental state of the D).
Laying the Foundation: before the expert can testify on the ultimate issue, you have to lay the foundation for it, which means you ask
about each element of the issue and then you ask about the experts conclusion.
Example: where an expert is testifying on testamentary capacity in a will case, you ask the expert about all the elements of
testamentary capacity and then you ask if the person had it or not.
RULE 706. COURT APPOINTED EXPERTS: Court appointed experts are not instead of experts offered by the parties, but
in addition to the parties own experts.
(A) APPOINTMENT: The court can appoint its own expert either on its own motion or on the motion of any party
The parties then nominate experts and the court makes a selection (can also pick its own expert) or parties can agree to any
expert witness.

54

Each party then gets to depose the expert and each can cross examine the expert (even the party that calls him as a witness).
Cannot be appointed by the court unless the witness consents to act.
A witness so appointed shall advise the parties of the witnesss finding, if any; the witnesss deposition may be taken by any
part; and the witness may be called to testify by the court or any party. The witness shall be subject to cross by each party,
including a party calling the witness
(B) COMPENSATION: Expert witnesses so appointed are entitled to reasonable compensation in whatever sum the court may allow.
The compensation thus fixed is payable from funds which may be provided by law in criminal cases and civil actions and
proceedings involving just compensation
In other civil actions, the compensation shall be paid by the parties in such proportion and at such time as the court directs,
and thereafter charged in like manner as other costs.
(C) DISCLOSURE OF APPOINTMENT: In the exercise of its discretion, the court may authorize disclosure to the jury of the fact
that the court appointed the expert witness.
You can chose whether or not to disclose to the jury that there is a court appointed expert
Drawbacks to Court Appointed Expert
Fear that jury will believe court-appointed expert thinking he is a more neutral party
Never know if expert is really neutral or not
Note: A lot of courts appoint experts in order to aid in their own understanding of the substantive issues, but wont testify to jury.
RULE 702(2): GATEKEEPING: In order for an expert to testify as to an opinion, the court first needs to determine that the
testimony is the product of reliable principles and methods.
FRYE TEST Was it generally accepted in the field where it belongs?
The 2 requirements for scientific and other expert testimony:
1. Reliability: Evidence must
a. Constitute scientific knowledge, meaning scientifically valid and derived by the scientific method;
b. Be good science; AND
c. Rest on a reliable foundation
2. Relevancy: Must be significantly tied to the facts of the case. So, even if the evidence is reliable, it must fit into what the
proponent wants to use it for.
Judges job is NOT to determine whether the result of the tests were reliable, just whether the method used is reliable. The judge is not
looking at the conclusions reached, just how the expert got to those conclusions. If the experts opinion is then admitted, it is up to the
jury to decide if the test was actually accurate.
(A) Even if a method is not generally accepted in the field, it can still be admitted in the courts discretion, and thus new or novel
techniques have a better chance of getting admitted.
(B) This reasoning was extended to cases outside the scientific arena. It now applies to any testimony that involves specialized
knowledge.
Example: Tire expert testifying in a products liability case where the expert will testify that the tire malfunctioned despite the
tire being old and worn out. The court will apply the same analysis as scientific tests trial court will consider the same
factors in determining the reliability of the expert testimony.
(C) Does not apply to all criminal or all civil cases, and when it is used only for new and novel black box evidence.
Only deal with New or Novel Issues. This essentially eliminates all medical testimony from review
CANNOT be pure opinion, has to be Black Box: a test which the jury will value heavily because such evidence looks like
scientific evidence, i.e. because it is a machine (a black box)
Note: This test eliminates the state court judges role from being overly involved: eliminates discretion to instances where it is more
likely to be bullshit; cost elimination
Factors to determine whether evidence is scientific knowledge, under reliability factor under Daubert
NOTE: Not a bar to admissibility; still under courts discretion, even if none of some, may still be admitted.
a. Whether the theory has been or can be reliably tested.
b. Whether the theory has been subject to peer review and publication
c. Rate of Error
d. Standards controlling the techniques operation, i.e. a professional organization that has standards for how these
tests must be done

55

e.
f.

Whether the test is generally accepted


Whether the technique was developed independently of the litigation

Practical Problem is that this is not that helpful outside of scientific cases
KUMHO TEST applies to ALL scientific, technical, specialized fields
Daubert is a flexible, practical approach to determine whether the jury should hear the testimony. Factors are relevant to this
determination, but the factors are not controlling.
Always ask: Was the theory used in the industry outside the litigation? If it is used all the time in the professional world, will
be less weary because it is sufficiently reliable in and of itself. Court is not going to question the professionals expertise.
o NOTE: Doesnt have to be generally accepted, but then based on other things that can be shown to be an extension
of established theories because you will have to prove the reliability.
Opposing side will object to the general acceptability or credibility, before the trial and after discovery, so cannot add new
experts. Thus, if the motion is granted, the Plaintiffs case is over wont be able to have any expert testify
Example: Tire expert testifying in a products liability case where the expert will testify that the tire malfunctioned despite the tire
being old and worn out. The court will apply the same analysis as scientific tests trial court will consider the same factors in
determining the reliability of the expert testimony.

56

HABIT AND ROUTINE PRACTICE


Character: general disposition and distinguished from
Habit and Routine Practice: Consistently and invariably narrow things that someone repeatedly does.
RULE 406. HABIT; ROUTINE PRACTICE: A persons habit or routine practice is admissible to prove that his conduct on a given
day conformed to his normal habit.
A. Evidence must indicate a specific, semi-automatic response to a specific situation.
B. The routine practice of a business is the same as a habit admissible to prove that the practice was followed on the occasion
in question
For example, can be used to describe how a letter is mailed
C. Evidence of habit doesnt have to be corroborated. Even if theres a contrary eyewitness, habit is still admissible
Silence as to the method of proof permitted
Example: If theres a car accident and one party is dead, you can bring in evidence that the deceased always stops at
stop signs where the D in the case claims that the deceased ran the light. This is even true if D had a neutral
eyewitness that says the deceased ran the stop sign
D. Evidence of character is not the same character evidence is not admissible to prove that a person acted a certain way on a
given occasion (cant say Mike is a peaceful guy so Im sure he didnt hit Joe on the head) except in some limited instances
Evidence of a persons tendency toward drunkenness, peacefulness, and observation of religion is NOT evidence of
habit; its character evidence
Evidence of habit must be a narrower issue and must be repeated extremely regularly to make it a habit.
Going to synagogue is not a habit because it consists of too many questions and variables; but, sitting in the same
place once you arrive is a habit its a voluntary, volitional thing
Example of habit evidence: Doctor giving patients the same info on drug interaction is admissible to prove that his conduct on a given
incident conformed to his usual standard (that he gave a particular patient the same information)
EVIDENCE OF CHARACTER
RULE 404(a). CHARACTER EVIDENCE NOT ADMISSIBLE TO PROVE CONDUCT; OTHER CRIMES, WRONGS, OR
ACTS
(a) Evidence of a persons character or a trait of character is NOT admissible for the purpose of proving action in conformity
therewith on a particular occasion, EXCEPT:
(1) Character of the Accused. Evidence of a pertinent trait of character offered by an accused, or by the prosecution to rebut
the same, or if evidence of a trait of character of the alleged victim of the crime is offered by an accused and admitted under
Rule 404(a)(2), evidence of the same trait of character of the accused
Used to prove conformity; criminal cases
(2) Character of Alleged Victim. Evidence of a pertinent trait of character of the alleged victim of the crime offered by an
accused, or by the prosecution to rebut the same, or evidence of a character trait of peacefulness of the alleged victim offered
by the prosecution in a homicide case to rebut evidence that the alleged victim was the first aggressor;
(3) Character of the Witness. Evidence of the character of the witness, as provided in Rules 607, 608, and 609.
Used to impair/enhance a witnesss credibility by showing the witnesss character for truthfulness, but only applies
to witnesses, defined as those actually testifying at trial
EXCEPTION = Where character is an essential element in the case (claim, charge, or defense) under the substantive law, then
character evidence is always relevant and admissible
Here, it can be a civil case such as negligent entrustment negligently lending a car to an unsafe driver
o Have to show drivers character of being a bad driver
Other than when its an element of the case, character evidence is admissible in 3 situations only:
1) Character of the accused: evidence of the character of the D is not admissible by the prosecution in their case-in-chief. D is
allowed to raise the character issue regardless of whether or not he testifies, but it has to relate to his character on the date in question
(date of the crime). Once D chooses to introduce his character (by calling a character witness), the prosecution gets to bring in its own
character witness to rebut
This is conformity character evidence. If D testifies, this is also impeachment of a witness.
2) Character of the victim: Evidence of the character of a victim can also not be admitted by the prosecution in their case-in-chief. D
is allowed to attack the character of the victim and then the prosecution can rebut it. Again, the victim does not have to be a witness in
the case. If its a homicide case and the D is claiming self-defense, the prosecution is permitted to offer evidence of the victims

57

character for peacefulness. Once D offers evidence that the victims character was bad, prosecution gets to introduce evidence that the
victims character was good and that the Ds character was bad.
This is conformity character evidence. If the victim also testifies, also impeachment of a witness.
NOTE: There is one restraint on this for sex offense cases see Rule 412
3) Character of a witness: Credibility of a witness can be attacked, subject to Impeachment Rules, and addresses the witnesss
character for truthfulness
This is impeaching a witness only
RULE 404(b). OTHER CRIMES, WRONGS, OR ACTS. Not admissible to prove the character of a person in order to show
action in conformity therewith. It may, however, be admissible for other purposes, such as proof of motive, opportunity, intent,
preparation, plan, knowledge, identity, or absence of mistake or accident, provided that upon request by the accused, the
prosecution in a criminal case provides reasonable notice of the general nature of any such evidence.
Likely to be a limiting instruction to the jury stating that the evidence is not admissible to prove that the D acted in
accordance with his past actions.
Bad man inference still likely to be a problem the more similar the past wrong, act, or crime, the increased likelihood
Doesnt have to be a crime to fit in this rule and even if the behavior is criminal, D doesnt have to have been convicted.
Prosecution just needs a good faith basis that D did it
o No Good Faith Basis: If D was vindicated someone else confessed to the crime or D was brought to trial and found
not guilty, then there is no good faith basis, even if the D had been arrested.
o Example: If D rented a car and then used it in robbing a bank, that is evidence of preparation of the crime, and thus
evidence of renting the car (the past act) is admissible
The other crime or act has to be separate from the crime being charged, and cant be an element of the crime D is on trial for
o Inadmissible: If D stole a security guards gun and used it to rob the bank, evidence of stealing the gun cant be
admitted as an other crime to prove Ds intent because you would need to show the theft of the gun to explain the
actual crime being charged
o Admissible: If D bought the gun illegally on the street and then used it in the bank robbery, thats evidence of plan or
preparation and its a distinct act, so its in
Judge doesnt have to decide whether or not the D committed the past crime in order to admit it just asks if a reasonable
juror could find it more probably true than not (Rule 403; 50% plus one)
o If yes, its in, regardless of the bad man inference, which favors the prosecution
Situations where this Rule is commonly used:
(A) intent in drug cases: Prior convictions for possession or possession w/ intent to distribute is always admissible to prove
present crime of possession w/ intent to distribute (even if its a completely different drug).
(B) Repeat Conduct: Can be used to show lack of mistake, that something wasnt accidental.
a. Example: Woman who had 3 husbands die in bathtub deaths of past husbands can be brought in to show an
absence of mistake, that it was intentional murder.
Has to be relevant for something other than for character of conformity in order to be introduced
Can be used in civil or criminal, but usually in criminal
You must bring witness with personal knowledge; police report not enough
In order to introduce evidence for truth, it must be something that is so unique that it provides identification for something
RULE 405. METHODS OF PROVING CHARACTER AT TIME OF EVENT.
(b) REPUTATION OR OPINION: In all cases in which evidence of character or a trait of character of a person is admissible,
proof may be made by testimony as to reputation or by testimony in the form of an opinion. On cross, inquiry is allowable into
relevant specific instances of conduct.
(c) SPECIFIC INSTANCES OF CONDUCT: In cases in which character or a trait of character of a person is an essential
element of a charge, claim, or defense, proof may also be made of specific instances of that persons conduct.
To pull one misshapen stone out of the grotesque structure is more likely simply to upset its present balance between adverse
interests than to establish a rational edifice.

Prosecution in rebuttal after D calls a witness, can now call witnesses in regard to reputation of the witness used by D, and
can now ask about specific instances of conduct.
When asking about specific instances of conduct, must have a good faith basis but have to accept the answer given.
If the D starts the ballgame by introducing character evidence of the victim, then the prosecution can introduce evidence as to
the D

58

Where character is an element an issue within the pleading (defamation; negligent entrustment; chastity)
1. Specific instances of that conduct
2. Reputation evidence: a persons nature or general disposition, not reputation in the community
3. Opinion evidence of character (in some jurisdictions) Are you aware?
Where character is being used to impeach a witness (affecting truthfulness)
1. Reputation
2. Specific instances of conduct (under the majority rule, it can be anything, but in minority, has to be conduct that resulted in criminal
convictions) Did you hear about x?
Note: Good character evidence is not admissible UNLESS credibility is attacked by proof of bad character
Where character is being used to prove conformity
1. Reputation or opinion only, not specific instances of conduct
2. Must be a witness w/ knowledge about the Ds character as of the date in question
If character is NOT an element, then would be circumstantial evidence
RULE 412. SEX OFFENSE CASES; RELEVANCE OF ALLEGED VICTIMS PAST SEXUAL BEHAVIOR OR ALLEGED
SEXUAL PREDISPOSITION:
(a) EVIDENCE GENERALLY INADMISSIBLE: Evidence offered to prove that any alleged victim engaged in other sexual
behavior; and to prove any alleged victims sexual disposition is not admissible in any civil or criminal proceeding involving
alleged sexual misconduct EXCEPT
(1) CRIMINAL CASES: the following is admissible, if otherwise inadmissible
(A) Evidence of specific instances of sexual behavior by the alleged victim offered to prove that a person other than
the accused was the source of semen, injury, or other physical evidence
(B) Evidence of specific instances of past sexual behavior offered to prove consent (must be past sexual conduct
with the accused, not someone else)
(C) Evidence where if it were excluded, it would violate Ds Constitutional rights
Illustration of Usage: where the same victim previously claimed to have been raped and it was proved to be
false or where the victim has a motive to fabricate (such as where shes a teenager whose parents catch her
in the act).
NOTE: Evidence of the way the victim was dressed on a given day is not considered character evidence
and its not prohibited from being admitted
(2) CIVIL CASES: In order for the victims past sexual conduct to be admissible = conduct must be otherwise
admissible and its probative value substantially outweighs the danger of harm to any victim and of unfair prejudice to
any party.
Evidence of an alleged victims reputation is admissible only if it has been placed in controversy by the alleged
victim
(b) Procedure to determine admissibility:
(1) A party intending to offer evidence must:
(A) Need written motion 14 days before trial describing evidence and purpose
(b) Serve motion and notify victim
(2) Before admitting, court must conduct a camera hearing where victim and parties are entitled to be present and
heard. The motion, related papers and the record of the hearing must be sealed and remain under seal unless the court orders
otherwise
RULE 413. EVIDENCE OF SIMILAR CRIMES IN SEXUAL ASSAULT AND CHILD MOLESTATION CASES: Evidence of
past sexual battery or child molestation (doesnt have to have been convicted of the crime) is admissible to prove character for
conformity (not just identity or plan, etc.).

In a sexual battery case, you can admit Ds commission of another offense(s) of sexual assault, and may be
considered for its bearing on any matter to which it is relevant, but prior molestations are inadmissible
In a molestation case, you can admit prior molestations but not prior sexual batteries

Procedure: Government has to disclose the evidence to the D 15 days before the trial

59

CROSS, IMPEACHMENT, AND REFUTATION


RULE 611. CROSS MODE AND ORDER OF INTERROGATION AND PRESENTATION:
(A) Control by Court Court controls mode and order of interrogating witnesses and presenting evidence so that its effective for the
ascertainment of the truth, it avoids needless time wasting, and it protects witnesses from harassment or undue embarrassment.
(B) Scope of Cross limited to subject matter of direct and matters affecting credibility. Court has discretion to allow further
examination, as if on direct.
(C) Leading Questions not used on direct except as necessary to develop witnesss testimony. Ordinarily allowed on cross.
Exception where typically allowed on direct: hostile witness; adverse party; witness ID with adverse party.
RULE 104(d). TESTIMONY BY ACCUSED: Accused can testify on a preliminary matter without being subject to cross on
other issues in the case
Example: 4th amdendment Suppression Hearing D can testify that he didnt consent to the police searching his home and the
prosecution cannot cross on anything other than the 4th amendment suppression issue
Scope and Extent
1) Scope is what youre allowed to talk about
2) Extent means how much you can talk about it

D has a right in a criminal case to cross witnesses, only to the extent that questioning is effective
o If P calls a witness, he can ask the witness only questions helpful to his side and then the D can only cross on the
subjects brought up on direct. So, if the witness has something thats beneficial to the D, D has to call the witness
himself during his case-in-chief

EXCEPTIONS: when leading questions are NOT allowed on cross


A. When the court, in its discretion, allows questioning which exceed the scope on cross, and in essence the witness is now your
witness, and the questions need to meet the requirements of direct
B. When a cross is done, after the other side was allowed to use leading questions on direct when questioning an adverse party
(or someone associated with an adverse party).
Example: P calls the Ds wife as a witness. P can use leading questions on direct. When D conducts cross, he cant use leading
questions on his own clients wife
NOTE: In order to be identified with the adverse party, a person cant just be an occurrence witness who is testifying for the opposing
side; he has to be particularly identified with the D such as the Ds wife, child, sometimes employee, where the employee is still
employed by the D. But, typically an employee is not automatically deemed to be identified with the adverse party when its his
employer
IMPEACHMENT
RULE 607. WHO MAY IMPEACH: The credibility of a witness may be attacked by any party, including the party calling the
witness.
Both contradiction and sincerity are part of 607 prior convictions admissible under exceptions of character evidence go to sincerity
risk more likely to lie on the witness stand
Impeachment = attacks credibility:
A. Forgetful, made a mistake attacking personal knowledge based on an inconsistent statement or
B. that witness is lying
NOTE: even if the person is considered a good witness, can prove that she is wrong or forgetful, not that she is lying based on the
capacity and actuality of a witnesss perception, his ability to record and remember sense impressions, his ability to comprehend
questions and narrate, and mental capacity.
Discovery: adverse party is entitled to know the names and addresses of everyone who is appearing as a witness in a case. But, you
get the address of where they lived at the time of the incident, not the present address.
Collateral v. Non-collateral: Question of whether after the question is asked and an answer given, can additional evidence be
admitted to refute the answer. If its non-collateral, the extrinsic evidence gets in and when its collateral, it does not.

60

(A) Non-collateral = extrinsic evidence is admissible; when it is an important part of the case
(B) Collateral = extrinsic evidence is not admissible; when it is not an important part of the case. Have to accept the answer as is,
and thus the jury has to decide the truth without additional evidence
Categories of Impeachment:
1. CONTRADICTION: Introduction of evidence that contradicts the testimony of the witness on direct. This has two
functions: (1) it proves the fact being brought up, and (2) it impeaches the witnesss truthfulness.
Substantively admissible vs. admissible only for impeachment
o SUBSTANTIVELY ADMISSIBLE: A hearsay statement is substantively admissible if it is a hearsay exception
same rules apply.
For instance, if you have a witness on the stand and you impeach the witness with a prior inconsistent
statement, thats a hearsay exception, so the statement is substantively admissible
o ADMISSIBLE FOR IMPEACHMENT ONLY: does not fit within hearsay exception
If you impeach the witness with a statement he made to a police officer two hours after the event occurred
(no longer an excited utterance), its still hearsay, so its not substantively admissible. But, it is admissible
for impeachment purposes
When is it collateral? A prior statement is going to be non-collateral when it is relevant in the lawsuit for something other
than impeachment establishes fact of consequence
Example 1: Witness to a car accident says she stopped for an egg McMuffin and saw D run the red light. Whether or not she actually
had an egg Mcmuffin is not relevant in the lawsuit, except for impeachment. So, if she said in a deposition that she went to Wendys,
you can use that to impeach her, but you cannot substantively admit the evidence cannot bring in the clerk from Wendys who saw
her there. You can ask the question but you have to accept the answer
Example 2: If the witness said at a deposition that D had a green light, that is non-collateral. You can impeach with it and you can
also bring in extrinsic proof. So, if the witness on the stand denies having said at the deposition that D had the green light, youre not
only allowed, but youre required to produce evidence of the inconsistent statement. (Sometimes, you have the option, but with
inconsistent statements, you are required to bring in the extrinsic evidence)
2. CREDIBILITY: Matters bearing on witnesss credibility other than merely through contradiction are non-collateral and
may be contradicted by other evidence. This includes:
1. Untrustworthy partiality, i.e., bias, interest, corruption, coercion;
2. Alcohol or drug use;
3. Deficient mental capacity; AND
4. Want of capacity, opportunity, or absence of actual acquisition and retention of personal knowledge.
Drug and alcohol use:
(A) Drug or alcohol use at the time of the incident can be used for impeachment (even if legal drug); credibility, not that the
witness is more likely to lie
(B) Evidence that the witness is an alcoholic is not sufficient
(C) Evidence that a witness is a drug addict is only enough if you have an expert testifying that the drug use has some kind of
on-going effect (like an LSD addict who sees things).
Entirety of Testimony: Even though a matter might not be otherwise relevant in a lawsuit, if it challenges the entirety of what a
witness says, its non-collateral.
Example: Perception of a witness to a car accident if she thought she was on Road A and she was actually on Road B, that
changes her perspective. On certain roads, it may be impossible for the witness to have been where she said she was (say she
was traveling on a one-way heading South but that road only goes East). So even though the street she was traveling on is
really not relevant in the lawsuit, it becomes relevant because if the witness perspective changes, that might change how she
interpreted the accident.
Witness Protection Program: Have to provide old information (name and address), but dont have to give alias information
Good Faith Basis: YOU HAVE TO HAVE A GOOD FAITH BASIS OF THE UNDERLYING TRUTHFULNESS OF QUESTIONS
ON CROSS. This is true only where asking fact-based questions. This does NOT apply to where cross-examiner is asking if
something is or is not true. Your glasses were being repaired at the time of the accident, werent they? requires good faith basis
while were you wearing your glasses at the time of the accident does not

61

MODES OF IMPEACHMENT
A. Prior inconsistent statements
B. Contradiction by other evidence
C. Untrustworthy partiality
D. Conviction of a crime
E. Prior acts of misconduct
F. Character of witness for untruthfulness and truthfulness
G. Religious beliefs or opinions
RULE 613. PRIOR STATEMENTS OF WITNESSES
(a) EXAMINING WITNESS CONCERNING PRIOR STATEMENT: When using a prior statement to impeach a witness,
whether its written or oral statement or any other memorialization, there is no requirement to disclose to the witness nor be
shown, but has to be shown or disclose to opposing counsel if it is requested.
(b) EXTRINSIC EVIDENCE OF PRIOR INCONSISTENT STATEMENT OF WITNESS: Extrinsic evidence of a prior
inconsistent statement is not admissible, unless the witness has an opportunity to explain or deny the statement and the
opposing party has the opportunity to interrogate the witness about it; or if the interests of justice require otherwise. This does
not apply to admissions of a party opponent.
PRIOR INCONSISTENT STATEMENTS came about after the event, but are different from what they testified to. Not
admissible substantively (not to prove the truth of the matter asserted), but admissible to attack a prior inconsistent statement; good
faith basis; and must be regarding a non-collateral matter (relevant for other than credibility). Thus, because it is non-collateral,
MUST bring in evidence to prove the inconsistency (unless witness admits to inconsistency)
Note: Questions given at deposition enables you to impeach on cross and not saying something that is relevant is also
grounds for impeachment
o Example: witness never said in deposition how fast the D was driving, and in court he testifies that D was going 70
mph
When questioning someone on cross regarding a memorialization or writing, do NOT have to show the witness the writing
before asking the question
Cannot contradict on collateral matter, unless witness volunteers information that is not otherwise part of the case
Prior inconsistent statements are non-collateral, and thus on cross
A. Can confront the witness as to his previous statement so that the jury can assess their credibility, truthfulness, and perception.
B. Can also provide jury with a contradiction
C. Admissible to impeach for the fact said, unless meets a hearsay exception (then comes in substantively)
D. Admissible to question testimony
NOTE: Everything is non-collateral except for 405(a), 608(b)(1) and (2).
Foundation on Cross When laying down a foundation, must show
1. Time
2. Place
3. Persons
4. Presence/circumstances
5. Content
Example: Traffic accident. Witness simply says that the light turned red, and the bus went through the light and crashed. On cross,
need to establish the time, place, persons present, circumstances, and content, thereby focusing the witnesss attention in order to give
him an opportunity to respond that the statement was made.
Ask, Do you remember that X made a statement to you regarding the accident (alone, lunch, on date, with X. etc.). Do
you recall discussing that you saw an accident 4 days prior? At this time, Didnt you say (quote the exact statement that
your witness will say on the stand). [if transcript or writing, read it into the record, if its a statement, close to exact as you
can.]
Witness wont want to say, I said it, so will usually say if it is in there I must have said it, which equals a YES, and thus
impeachment has been completed
Remember HAVE TO call the other person if the witness denies saying/writing anything have to call X to impeach the negative
answer

62

If the witness on the stand admits to the prior statement, you dont have to prove it dont have to call X
o If the witness admits it, some courts say you can bring in the extrinsic evidence, some say you cant. Its a Rule 403
issue
If the witness does not admit it, you MUST introduce extrinsic evidence if its non-collateral. If it is collateral, you cannot
introduce extrinsic evidence.

UNTRUSTWORTHY PARTIALITY: Admissible to impeach under Rule 607 matters, which may reasonably be expected to
color the testimony of a witness or cause him to testify falsely, such as bias, interest, corruption, and coercion. [Admissible
because factors go to sincerity risk; bias and interest broad.]
(1) Interest if you have interest in the outcome of a case, it affects your credibility, directly and indirectly
(2) Bias you can have a bias against a person, in favor of a group, any other interest than money
(3) Corruption you get paid for testimony
(4) Coercion physical threat
NOTE: Wide latitude is afforded to the D during cross for the purpose of establishing partiality
RULE 609. IMPEACHMENT BY EVIDENCE OF CONVICTION OF A CRIME:
(a) General Rule. For the purpose of attacking the credibility of a witness,
(1) Evidence that a witness other than an accused has been convicted of a crime shall be admitted, subject to Rule 403,
if the crime was punishable by death or imprisonment in excess of one year under the law under which the witness was
convicted, and evidence that an accused has been convicted of such a crime shall be admitted if the court determines that the
probative value of admitting this evidence outweighs its prejudicial effect to the accused; and
(2) Evidence that any witness has been convicted of a crime shall be admitted if it involved dishonesty or false
statement, regardless of the punishment
1. Deceit, untruthfulness, or falsification an active misrepresentation. Needs to be a lie of some sort, such as lying
on tax return, false police report, or perjury; NOT theft
2. Dishonesty or false statement has to be an actual element of the offense, so lying to a victim in order to rape her,
is not a crime of dishonesty or false statement

A prior conviction is part of character for truthfulness, and thus if you have disobeyed a law of society and been found guilty,
this is evidence of a bad character and in turn that you may have lied on the witness stand more likely to lie because of
possible consequences such as jail time
Evidence of the prior conviction is admissible only to impeach the witnesss character for truthfulness and not as evidence
bearing on the witnesss character for being law abiding
Under Rule 609(a)(2), have to indicate that something is false in order to impeach on truthfulness have to lie by hand or
mouth, part of grotesque structure
o Cannot include stealing, robbery, or narcotics have nothing to do with lying
Under Rule 609(a)(1), has to be something that is a felony (punishable by death or imprisonment more than a year) +
balancing test
o Balancing Test of 403
Criminal Ds probative value has got to outweigh the unfair prejudice. Probative value is on character for
truthfulness on a criminal D that is testifying and has the greatest interest in the case to begin with
Everyone else falls under 403 the probative value has to be substantially outweighed by the prejudice
o 403 Factors = factors that are considered in determining whether the probative value of admitting the evidence upon
the credibility of the criminal defendant outweighs its prejudicial effect:
The nature of the prior crime;
The length of the defendants criminal record;
Defendants age and circumstances;
The likelihood that the defendant would not testify;
The nearness or remoteness of the prior crime;
Defendants subsequent career;
Whether the prior crime was similar to the one charged;
The centrality of the issue of credibility; and/or
The need for defendants testimony
ANTICIPATORY DISCLOSURE: Forfeiture for Right to Appeal: Date & Nature of the conviction is disclosed if the
prosecution has won the motion in limine, and chose to disclose the prior conviction on Direct, rather than waiting for
prosecution to admit prior conviction on Cross.

63

Under Lucy, in order to appeal a ruling where the judge lets in a prior conviction of a D, D has to testify and be impeached with the
prior conviction on Cross.
If the judge rules on a motion in limine that the prior conviction is in, and so you dont put the D on the stand, then the issue
is not preserved for appeal
Also, if the D reveals the prior conviction during their case-in-chief, before the prosecution has the ability to impeach the D
with the prior conviction, then also not preserved for appeal; considered a waiver of the right to appeal
When do you have to make an objection during the trial after there was a ruling denying the motion in limine on admissibility of a
prior conviction? (for prior convictions, under 1 year, can make a motion to suppress)
Judge doesnt have to rule on the motion until it gets to trial
If he does, the question is whether or not its a definitive ruling you have to ask the judge if its definitive or not.
o If it is definitive, you do not have to object again at trial in order to preserve the right to appeal
o If it is not definitive, you do have to object again at trial in order to preserve the issue for appeal
Florida: Eliminates balancing test ANY previous felony (punishable by 1 year or more) OR found guilty for crime that involves
dishonesty or false statement can be used to impeach:
Eliminates discrimination; reliability
Method by which it is to be introduced will be different than federal system: no date, no nature, no amount of sentence.
o If admit prior convictions on direct, while on cross, CANNOT question the D on anything relevant to the
convictions (cannot repeat questions or expand on them, i.e. cannot ask what the crime was)
Structure balance
(b) TIME LIMIT. Evidence of a conviction under this rule is not admissible if a period of more than 10 years has elapsed since
the date of the conviction OR of the release of the witness from the confinement imposed for that conviction, whichever is the
later date, unless the court determines, in the interests of justice, that the probative value of the conviction supported by
specific facts and circumstances substantially outweighs its prejudicial effect. However, evidence of a conviction more than 10
years old as calculated herein, is not admissible UNLESS the proponent gives to the adverse party sufficient advance written
notice of intent to use such evidence to provide the adverse party with a fair opportunity to contest the use of such evidence

if the conviction is over 10 years, the probative value has to substantially outweigh the prejudice
o one of the most important things is you have to give notice. Then it applies to everything
Often, the federal courts allow convictions that are more than 10 years old if there are convictions (felonies/misdemeanors)
that are less than 10 years old
It is 10 years as of either the date the trial begins or the date the witness testifies (debated)
Covers all crimes (felonies; dishonesty or false statement crimes)
Same rule for both Ds and witnesses

(c) Effect of Pardon, Annulment, or Certificate of Rehabilitation. Evidence of conviction is not admissible under this rule if
(1) Conviction was the subject of a pardon, annulment, certificate of rehabilitation, or other equivalent where the person
convicted was found rehabilitated, and that person has not been convicted of a subsequent crime which was punishable by
death or imprisonment in excess of one year, OR
(2) Conviction has been the subject of a pardon, annulment, or other equivalent procedure based on a finding of innocence
HAS to be based on rehabilitation OR innocence. Getting out of jail (say for time served) or getting civil rights restores is
NOT sufficient.
If released on probation can still use your prior conviction
(d) Juvenile Adjudications. Evidence of juvenile adjudications is generally not admissible under this rule. The court may, however,
in a criminal case allow evidence of a juvenile adjudication of a witness other than the accused if conviction of the offense would be
admissible to attack the credibility of an adult and the court is satisfied that admission in evidence is necessary for a fair
determination of the issue of guilt or innocence

Cannot use prior conviction to impeach because in juvenile system and character still forming
Exception: Davis case very narrow: if evidence is also relevant for impeachment for untrustworthy impartiality (bias),
then can be used to impeach in the discretion of the court
o Relevant for something other than character for truthfulness

64

(e) Pendency of Appeal. The pendency of an appeal there from does NOT render evidence of a conviction inadmissible. Evidence of
the pendency of an appeal is admissible
NOTE: If plea nolo contender, still admissible, even if received restoration rights
What information about the prior conviction gets in? Allowed to admit whatever appears on the judgment of the conviction
(a) Date
(b) Court
(c) Nature of offense
(d) Sentence
Rule 608 & 609 character of truthfulness of witness AT TRIAL, when you testify, evaluation of credibility, amount
of truthfulness is question (sincerity risk) NOTE Rule 608 and 609 Evidence has to be regarding the time of
the testimony, NOT the time of the event. So, youre trying to show the witness has a character for untruthfulness at
the time of the testimony.
Versus
Rule 414 AND 404 (A and B), character for pertinent of the crime, AT THE TIME OF EVENT, offered to prove that
he actually committed the offense either through character or non-character acts.
RULE 608. EVIDENCE OF CHARACTER AND CONDUCT OF WITNESS
(a) OPINION AND REPUTATION EVIDENCE OF CHARACTER. The credibility of a witness may be attacked or supported
by evidence in the form of opinion or reputation, but subject to the following limitations:
(1) The evidence may refer only to character for truthfulness or untruthfulness, and
(2) Evidence of truthful character is admissible only after the character of the witness for truthfulness has been
attacked by opinion or reputation evidence or otherwise
Refutation. Rebuttal of Character Testimony. You cant say that a party has a character for truthfulness or a character for
peacefulness, etc. until the opposing party has already attacked that partys character.
On cross of a character witness, can ask about specific instances of conduct of the principal witness after he gives his
opinion, if the court allows it.
How do you attack? A character witness in the form of reputation, opinion, or otherwise attack
Or otherwise
Used because it is undefined and subject to interpretation by court
Can attack narration, recordation, sincerity, or personal knowledge (usually personal knowledge)
Contradiction is not a direct attack
Note: most of the time, opinion and reputation witnesses wont be called because of risk on cross
Prior convictions
o Under Rule 608, if the attack on the witnesss character is by means of a prior conviction, then this is a strong attack
on character for truthfulness. Corruption and coercion are strong attacks also, dont happen often, but Bias or
interest are not sufficient attacks due to its being too broad
o Under Rule 609, prior convictions are the biggest attacks have to be admitted
What can you ask?
1) You can ask about instances of coercion and intimidation but NOT bias.
2) Can ask questions regarding the basis of the testimony
3) Cannot ask the witness on cross whether his testimony would change if he knew the accused committed the crime hes charged
with. BUT, can ask questions on cross as to the character of the D and can question him regarding Ds truthfulness, in regards to
previous crimes, acts, etc. because goes to the basis of the witness. Thus, reputation or opinion witnesses will not be called.
There has to be an attack on the witnesss character for truthfulness before you can refute it
Good character for truthfulness is only admissible if there has been a character witness called by the opposing side during
rebuttal (cannot call witness during case-in-chief, since there hasnt been an attack as of yet)
Thus, if the D does not testify, his character for truthfulness can never be attacked since he wasnt a witness !

65

Character in community is based on where a person hangs out, where he works, or where he lives, etc. (no longer limited to
JUST where he lives)
You typically cannot ask the character witness why he has that opinion (specific instances of conduct that the opinion is
based upon), UNLESS the court gives you discretion to do o and the specific instances have to concern the witnesss
character for truthfulness

(b) SPECIFIC INSTANCES OF CONDUCT. Specific instances of conduct (crime, wrong, or act) of a witness, for the purpose
of attacking or supporting a witness, other than conviction of a crime as provided in Rule 609 (acts of dishonesty or false
statement), may not be proved by extrinsic evidence.
(1) They may, in the discretion of the court, if probative of truthfulness or untruthfulness, be inquired into on cross of the
witness OR
(2) Concerning the character for truthfulness or untruthfulness of another witness as to which character the witness being
cross-examined has testified.
NO arbitrary 10-year limitation: Can be a crime thats over 10 years old court may allow questions to impeach, but not
extrinsic evidence
o Thus, if the witness is the D, and he says he didnt do it, you cannot prove he did
o If the court feels that the past act/event is too attenuated to the current trial
No limit on the number of past instances, its just up to the court to decide when theres too many; often there will be no
limit
Evidence of Character and Conduct of Witness on STAND probative of truthfulness: you can attack a witnesss character by
using opinion or reputation based on an act of dishonesty or false statement
Subject to these limitations:
1) May refer only to character for truthfulness or untruthfulness cannot ask questions besides the specific act/event ask about the
underlying act, not the indictment or conviction
2) Evidence of a truthful character is admissible only after the witnesss character for truthfulness has already been attacked
3) MUST accept the answer due to its collateral nature, and thus cannot bring extrinsic evidence
4) MUST have a good faith basis
Prior inconsistent statements indicating an inconsistency of belief can go to personal knowledge or credibility. How do we
know at the end of the cross, using prior inconsistent statements a VIGOROUS attack in order to allow D to immediately call
another witness during case-in-chief? In cross, if using leading questions and prior inconsistent statements, must use
CLEARLY SUGGESTED THAT THE WITNESS LIEDthen court will allow.
RULE 610. RELIGIOUS BELIEFS OR OPINIONS: You cannot use religious beliefs or affiliations to attack the credibility (or
truthfulness) of the witness, except if it is relevant to establish a bias or interest.
Any probative value is outweighed by any unfair prejudice to the jury.
Never considered a privilege always be inadmissible
But, religious affiliations will show up if they prove the witness has an interest in the case (exception)
Example: if the Scientologists are on trial and Tom Cruise is a witness, you can use his belief in Scientology to show bias or interest.
Not belief, but because they are a member of the church, and thus bias regarding testimony can be questioned
RULE 607. WHO MAY IMPEACH: Any party can attack the credibility of a witness, even the party who called the witness.
If the party calls the witness and the witness gives testimony that is surprising and damaging to your case, you can attack you
own witnesss credibility
To use a prior inconsistent statement to impeach a witness, the witness has to actually testify to an inconsistency, cannot just
get up there and say nothing
Voucher Rule is eliminated (when you called a witness, vouching for your belief in his credibility)
Could also argue admissibility under Rule 403: Mere Subterfuge Primary Rule if anything relevant to the case, then you can
impeach as to an inconsistent statement used to impeach. Applies to the concept that if there is no other purpose, say something
positive. This favors the government significantly. Balance of testimony that is not otherwise admissible and the perceived reliability.
Cant explain to witness can summarize a redirect based on non-leading questions.
1) Witness testimony came as surprise assume that the witness was going to testify consistently

66

2) Affirmatively Damaged: positive assistance/testimony to the D (say witness testifies that the person who he saw looked
completely different to the D; thus, this helps D). Appearance to competence does not qualify

67

RELEVANT EVIDENCE AND SOCIAL POLICY

Relevant admissible evidence is being refused based on other reasons than Rule 403 concerns. Examples are the various
privileges. The value to society outweighs letting the evidence in.

RULE 407. SUBSEQUENT REMEDIAL MEASURES: evidence of remedial measures that would have made the injury or
harm less likely to occur is not admissible to prove negligence, culpable conduct, a product defect, or design defect. This does
not exclude the admission of subsequent remedial measures for other purposes such as ownership or control if that issue is in
controversy.

When, after an injury or harm allegedly caused by an event, measures are taken that, if taken previously, would have made
the injury or harm less likely to occur, evidence of the subsequent measures is not admissible to prove the allegation, but can
be used to prove ownership, control, feasibility of precautionary measures if controverted, or impeachment.
Public Policy concerns dictate that subsequent remedial measures should be inadmissible in order to promote the taking of
safety precautions.
Does not include when the government tells you to do something (such as a recall); have to do it on your own; go out and
change something so that the injury or harm is less likely to occur.
o Matters that are excluded: (a) subsequent removal of a dangerous condition; (b) termination of an employee
involved; (c) later changes in rules of procedure of entity; (d) warnings, notices, instructions attached to
subsequently produced products; (e) manufacturers agreement to accept or return unsold products

Impeachment: you can use subsequent remedial measures to impeach a witness if it is being used to contest claims by the D that they
were in fact the safest product on the market
D is not normally going to say that its action was proper, i.e. safe, not defective, etc. on its case-in-chief, and once
evidence of safe or not defective is introduced impeachment by reference to subsequent remedial measure as inconsistent
conduct seems appropriate on cross.
o Under this Rule, cross on this critical issue cannot be precluded
o 403 may permit exclusion if the court determines that the danger of the jury considering evidence on the issue of
negligence or culpable conduct substantially outweighs the probative value for the evidence of the impeachment.
But, many decisions still let evidence in.
o BUT, when the testimony introduced by a D goes no further than to maintain that nothing improper occurred,
impeachment with a subsequent remedial measure will be prohibited.
Despite the limiting instruction (that the remedial measure is to be used for a purpose other than to establish negligence of
culpable conduct, i.e. impeachment), effectiveness is extremely suspect, and thus policy considerations are greatly
compromised
A. Inadmissible to Impeach: Witness-D goes no further than to maintain that nothing improper occurred; if on cross, a witness
simply says the situation would not in fact have been safer if X had been done, the witness could not then be asked why X
was in fact later done as the impeachment of credibility arises solely by means of the prohibited inference. safe, or not
defective. [can say what is necessary to defend]
B. Admissible to Impeach: Witness goes beyond what is necessary, and states on direct that this conduce was the safest, most
reasonable, or best designed product possible. A similar answer on cross or direct of an adverse witness should also trigger
the right to impeach, as would unresponsive answers. we have tested 7 things and this is the best. [any statement with
hyperbole.]
Ownership or Control If act, and dont want that subsequent measure admitted for impeachment purposes, do not dispute
ownership, control, or feasibility
Example: The threshold between Macys and the mall, but Macys still responsible for that area. Someone gets shoe caught in the
threshold and falls down
If Macys claim is that the threshold is not in their control, but did something such as putting a plate; changing maintenance
companies, etc. inadmissible to prove culpability, but admissible to prove ownership or control
Feasibility If you fix something and dont want it to be admitted, then do not dispute ownership, feasibility, or control
If claim that subsequent measures were not possible/feasible, and then go and do something, then admissible to prove
feasibility of the measure due to the fact that evidence is to controvert your claim (impeachment)
Fixing includes
o Changing the condition itself;

68

o
o

Changing the policy that controls;


Redesigning the product

NOTE: If you conduct a test to see if there is unreasonable risk, then can be admissible against you if it shows something wrong with
the product. This is defect in the way the rule was written test does not make the accident less likely to happen
Example: Tire that is made to go 90 miles an hour, for the purpose of allowing cops to go on high speed chases. Cops buy the tires,
and the only time the tire fails is in high-speed chases, which was the entire purpose of the tire. The large tire entities will have to fix
this anyways; thus, they will argue that this rule is not needed. But, rule still applies Here, if the manufacturer fixes the tire on its
own accord, evidence of fixing it will be admissible for impeachment purposes
RULE 409. PAYMENT OF MEDICAL AND SIMILAR EXPENSES: Evidence of offers or completed offers to pay medical or
hospital expenses from an injury isnt admissible to prove liability for the injury.
Encourages the person to get back to where he originally was without admitting guilty & furthers the policy of encouraging
assistance to an injured party, if assistance might be withheld if there was a risk of its use as an admission in a subsequent
trial
BUT, note that the only thing that is protected is the furnishing of payment. If anything else is stated regarding the guilt,
then admissible as a party opponent
o Limited scope does not cover company statements
Rule 408. COMPROMISE AND OFFERS TO COMPROMISE: Evidence of (1) furnishing or offering or promising to furnish,
or (2) accepting or offering to accept, a valuable consideration (money) in compromising or attempting to compromise a claim
which was disputed as to either validity or amount, is NOT admissible to prove liability for or invalidity of the claim or its
amount

Evidence of conduct or statements made in settlement negotiations is likewise not admissible.


o Does not require the exclusion of any evidence otherwise discoverable merely because it is presented in the course
of compromise negotiations
o Does not require exclusion when the evidence is offered for another purpose, such as proving bias or prejudice of a
witness, negating a contention of undue delay, or proving an effort to obstruct a criminal investigation or prosecution
Thus allowed to show a bias or prejudice based on having an economic interest in settlement. Ordinarily,
not allowed, unless you have a real interest (Mary Carter)

Example: If you want to settle a claim you have to talk about numbers and content, so if you are willing to pay 100k to settle the
claim the jury might take it as an admission of guilt. Therefore, offers to settle cannot be used to prove liability as long as the party is
contesting damages or liability.
A. Inadmissible: If the party says, I hit you, but I dont agree with your damage estimate or I didnt hit you, but your damage
estimate is correct (though it is arguable that they are admissions of party opponents)
B. Admissible: If the party offers to pay an admitted amount or to pay the entire cost of repair or replacement
NOTE: Dont need to be with an attorney, but if there is no dispute as to fault then admissible as an admission by a party
opponent
408(a)(2) Amendment to Rule: could be admitted in a criminal case substantively, and/or impeachment purposes because not being
used to show guilt? No, not generally admitted, but DOJ has interest in other suits such as SEC and antitrust any government
enforcement issues. So, government brings consecutive civil and criminal cases statements made in conjunction with
compromises in civil case can be used in a criminal case where the claim was made to a government entity.
Even if statement is inadmissible/protected in the civil case, still admissible in a criminal case to keep you from testifying
inconsistently to the statements made cannot maintain two different statements.
ONLY THE STATEMENT(S) YOU MAKE THAT ARE INCONSISTENT if statement you make in criminal case is
inconsistent with that in the civil case, then the statement made in civil case is substantively admissible to impeach.
If statement in criminal case is collected through Miranda, then inadmissible
Example: car accident hit by DUI driver and have civil suit for damages, but criminal case arises out of the same accident. If civil
suit is settled, can it (and/or statements) be used in a criminal case? NO, not generally admitted (not settling with government entity?)
(b) Exceptions. The court may admit this evidence for another purpose, such as proving a witnesss bias or prejudice, negating
contention of undue delay, or proving an effort to obstruct a criminal investigation or prosecution.

69

RULE 410. INADMISSIBILITY OF PLEAS, PLEA DISCUSSIONS, AND RELATED STATEMENTS: Except as otherwise
provided in this Rule, evidence of the following is not, in any civil or criminal proceeding, admissible against the D who made
the plea or was a participant in the plea discussions:
(1) a plea of guilty which was later withdrawn;
(2) a plea of nolo contendere;
(3) a plea that is not accepted by the Judge; OR
(4) any statement made in the course of plea discussions with an attorney with prosecuting authority which does not result in a
plea of guilty or which results in a plea of guilty later withdrawn
Note: The ONLY time statements are admissible against D is when plea of guilty has been entered and not removed

It is not considered an admission of a party opponent and cannot be used to prove their liability or guilt in either a civil or
criminal case
Plea discussions that dont lead to a plea are inadmissible, but if the D brings in part of the plea discussion and in fairness
another part of the discussion should be brought in for fairness then it will.
o Not Plea Bargaining = What can you do for me, doesnt contain the elements of negotiation
o Plea Bargaining quid pro quo = Ill plead this if you give me that what concession will you give me for a guilty
pleagot to actually talk with lawyer in plea bargaining terms!!!
The statements made in conjunction with the plea-bargaining have to be made to the prosecutor in order to be excluded; if
they are made to the cops they are admissible !

IF D exhibited an ACTUAL EXPECTATION for something in return for a guilty plea and that expectation IS REASONABLE
under the circumstances, the statements D makes are not admissible against you in a subsequent case or for impeachment
purposes if the plea bargain does not go through
Problem comes when there is no lawyer, and D is talking to prosecution
What happens if D talks to cop? Ill talk to you, and plead guilty to armed robbery, in exchange for a guarantee that my
sister who drove the car wont be prosecuted. Here, the D in fact exhibited an actual expectation, but that expectation was
NOT reasonable, because defendants are deemed to know that cops cannot make a promise regarding prosecution. Thus, the
statements can be used against the D.
o If the cop says he is authorized to speak, then different situation federal never authorized, have to negotiate with
an assistant state attorney but some states hold that if cops say they are authorized, then wouldnt be fair to D, so
statements may be inadmissible.
Two Times when Admissible
1) proceeding wherein another statement made in course of same plea or plea discussion has been introduced and the statement ought
to in fairness be considered together
2) Criminal Proceeding for perjury or false statement if statement was made by D under oath on record in the presence of counsel
WAIVER: Waiver in the federal system is constitutional, and can be proposed by the Government. If D waives his rights,
statements can be admitted against the D or can be used as an admission of a party opponent.
Impeachment Restrictions + Substantive Restrictions (admission of a party opponent)
Example: Government says that in order for plea bargain, D has to give someone up (telling personal knowledge regarding them), and
has to agree to waive his Rule 410 rights.
Statement: If at any time, you testify to any inconsistencies, even in your own case, we will be able to use any statement to
impeach you as to any inconsistent statement. If you change your mind at any point, we will be able to impeach you. And we
can substantively admit in our case-in-chief as an admission of a party opponent.
o Jury will obviously use testimony substantively, even with a limiting instruction
Treated as a K, not a right, so able to waive both rights
Justification = plea-bargaining important component of criminal justice system and benefits all of those concerned. Not
enough judges, juries, jails designed for the Governments benefit !
State Court Usually waiver concept is not used, since in state courts, Ds are prosecuted for what they were arrested for not part
of a larger conspiracy
RULE 411. LIABILITY INSURANCE: The presence or absence of liability insurance is not admissible to show liability,
however it is admissible for other reasons such as proof of agency, ownership, control, bias, or prejudice of a witness

70

Insurance is relevant in the form of character evidence under 401? Jury will use this evidence improperly vased on the deep pockets
theory, and thus Rule 411 eliminates the Balancing Test of 401, and dictates a bright line rule that insurance is not admissible when
offered to prove whether the person was liable
Example: if involved in auto accident, what is the relevance of insurance in determining whether someone is negligent? (a)
Cautious, careful person more likely to drive carefully b/c concern of carefulness OR (b) person buys insurance in order to
drive more liberally since he knows that he will not be personally liable insurance will pay damages, not him
EXCEPTION: insurance is admissible when used to show another purpose agency, ownership, or control.
Proof of Agency: used to establish if the person is an employee of a company or an independent contractor.
If the newspaper insures the newspaper deliverys car, then the proof of insurance can be used to show the delivery man is an
employee of the newspaper.
In car accident, when arguing that the driver is not an employee (or an agent), but an IC, the presence of you on the insurance
can be used to show that you are responsible for the car, not admissible to show liability
Bias or Prejudice:
(1) Insurance investigators need someone to testify to a non-collateral inconsistent statement. On cross, can ask whom he works for,
whether the P has insurance, how much, etc, because of the possibility of bias insurance company may be biased b/c they will be
liable for the amount
How is it avoided? When the insurance company may be liable for a lot of money, will hire an independent investigator.
Then, on cross, cannot question on bias concerns because the independent investigator is not an agent of the insurance
company, he is an IC difference is that he is not directly employed by the company.
(2) Jurors: may have some connection with an insurance company, which has some connection with the case. Thus, would have an
interest in the outcome (based on interest of monetary concerns/ownership/insurance rates, etc).
Accepted that any case where insurance/monetary damage is involved, jurors automatically assume that there is financial
liability/financial responsibility, and thus assume that insurance is involved, so general rule regarding whether the juror has
insurance is not permitted
But, if in fact a juror is an insurance agent or owner, then insurance issue is involved: Counsel has an opportunity to bring it
up when asking about employment questions during voir dire

71

JUDICIAL NOTICE
RULE 201. JUDICIAL NOTICE OF ADJUDICATIVE FACTS: This rule governs only judicial notice of adjudicative
facts. (who; where; what)
If the fact is not beyond reasonable dispute, a judge can take judicial notice of something.
Matters that would be difficult to prove and are time consuming to prove
A party can still have evidence presented for a fact that is not in dispute if they feel they have more compelling evidence
Not governed by evidence rules governed by PROCEDURAL RULES
Required to take judicial notice in the US as a part of statutory, constitutional, and judicial rules within the jurisdiction of
the US only Only applies to domestic law
A judicially noticed fact must be one not subject to reasonable dispute in that it is either
(1) Generally known within the territorial jurisdiction of the trial court, AND
Varies by region (i.e. buoys being capable of their own movement can be judicially noticed in New Orleans, not in
Oklahoma)
o Example: It is generally known within Miami that a brand new Mercedes is worth more than $10,000 in
grand theft case; may not be generally known in some bum fuck city.
(2) Capable of accurate and ready determination by resort to sources whose accuracy cannot reasonably be questioned
Cannot be based on political facts (i.e. Israelis torture of prisoners) can be reasonably disputed by a trier of fact
Has to be a source that cannot reasonably be questioned
Finding the law that applies: capable of accurate and ready determination by resort to sources whose accuracy cannot reasonably be
questioned
In federal cases, facts found in court decisions and Federal Registrar since they are easily discovered.
When you cannot find the applicable codes, for example Dispute about a driveway in Coral Gables, but trial in Illinois
applicable municipal codes as to driveways in Coral Gables are not readily available.
o Where are you going to find them? To the extent that codes, laws, statutes, etc are now becoming more readily
available on the internet, court may take judicial notice still discretionary, but judges are leaning to the rule of
judicial notice where available on Internet
When can judicial notice happen?
1. Court may take judicial notice, whether requested or not, on its own motion
2. Court shall take judicial notice if requested by a party and supplied with the necessary information. OC must be given an
opportunity to be heard on the motion either before or after judicial notice was taken
3. Judicial notice may be taken at any stage of the proceeding, including on appeal
4. Doesnt matter who the trier of fact is can be a judge or jury
Civil v. Criminal Instructions Jury will be instructed during the trial, when the information is offered, as to what to do with the
facts that were taken on judicial notice. NOT in jury instruction
A. CIVIL = Court shall instruct the jury to accept as conclusive any fact judicially noticed.
OC can object and reserve the error for appeal, but the court will instruct the jury to take the facts as true.
Example: I will instruct you to find that the newspaper is in general circulation..
B. CRIMINAL = Court shall instruct jury that it may, but is not required to, accept as conclusive any fact judicially noticed.
In a criminal case, where the issue is an element of the crime, cannot receive judicial notice because the government has the
burden of proving each and every element of a case beyond a reasonable doubt
Thus, the jury instruction above, in theory, is not conclusive, although jury usually takes the instructions to mean that that
element has been met.
Examples:
1. Whether prison is within the jurisdictional district of the trial fact not subject to reasonable dispute, and thus can be taken
by judge alone, upon request, or on appeal, without actual proof
2. Error in process and party fails to prove the fact Criminal case. Person stole car in the showroom, happens to be $35,000.
Charged with felony because of value over $10,000. Ordinarily, the governments witness, owner of the car, and I didnt
give, lend, borrow it, he basically stole it. Then counsel would ask how much he paid for it lay witness can testify to his
own propertys value. Here, would testify that he paid over 10,000 the day before it was stolen. What happens if he doesnt
say that the car was worth over 10,000? OC will request a motion to dismiss based on the fact that all of the elements of the

72

3.

charge were not met the value of the car. Prosecution can request to call one witness to establish the value, but also ask the
court to take judicial notice of the fact that this particular car is worth more than 10,000
Criminal cases even if there is no dispute say about the excess about the money. NOT allowed to advise the jury to take
that as true, because that would remove an element that is to be determined by the jury. In the concept of a jury trial, the jury
is entitled to decide whether or not the ultimate fact has been proven so the judge cannot tell the jury what to find (removes
an element from the case for jury deliberation).
Court would state, you may or may not be required to find that the car was over 10,000. Assisting the jury;
basically, there is enough there, but not required to find the element.
Now, in theory, can offer evidence as to either the amount is over or under

Domestic Law Application


A. As between states taking judicial notice of federal law, Supremacy Clause applies
B. As between states taking judicial notice of other states law, Full Faith and Due Credit Clause
C. When Federal court has to take judicial notice of state law, say in a diversity case Federal Statute requires that the federal
court take judicial notice of the state law
FOREIGN LAW: Legislative fact NOT required to take judicial notice of anywhere outside US.
Can take judicial notice as a matter of discretion, but not required !
How does foreign law get introduced? Every judge will attempt to not admit another countrys law and instead use the law of
the US.
When trying to prove to the court what the other countrys law is, have to get a WHORE witness having other reasonable
explanation i.e. a lawyer who is an expert in that field, and who will testify as to the applicable law of the foreign country.
Other side will get another WHORE. Judge has to decide which one of the experts is correct
Both federal rule of civil and criminal procedure dictated that the evaluation is to be made through testimony of an expert
witness.
o But, as to US law, Judge himself is ultimately responsible for determining law no testimony needed!

73

PRIVILEGES
RULE 501. GENERAL RULE: Common law rules, as may be interpreted by courts, in light of reason or experience on
privileges apply, except where the Constitution, congressional statute, or SC rules require otherwise.
However, in civil actions and proceedings, with respect to an element of a claim or defense as to which State law
supplies the rule of decision, the privilege of a witness, person, government, state or political subdivision thereof shall
be determined in accordance with state law (when in federal court)
Three situations where state evidence rule will take over federal rules in federal court
1) Privileges
2) Competency
3) Presumption
Standard 502. REQUIRED REPORTS PRIVILEGED BY STATUTE (go to legislature or argue in court on merits)
A person, corporation, association, or other organization or entity, either public or private, making a return or report required
by law to be made has a privilege to refuse to disclose and to prevent any other person from disclosing the return or report, if
the law requiring it to be made so provides
o All entities involved hold the power to refuse to disclose and to prevent any other person from disclosing return or
report
NO privilege exists under this rule in actions involving perjury, false statements, fraud in the return or report, or other failure
to comply with the law in question
Thus, if there is a c/l or statutory privilege, then it exits. This is a dual system
o Example: tax returns, public assistance reports, unemployment claims, public health records
A. LAWYER-CLIENT PRIVILEGE: Confidential communications privilege covers both communications from client to
lawyer, but also communications from lawyer to client.
Requirements:
1. Must be intended to be confidential; and
2. Must be seeking legal advice (not just business advice)
a. Who is a lawyer? Anyone who is authorized or reasonably believed to practice law is under privilege
Can be part of the lawyers team paralegal, associates, experts retained to assist in preparing for litigation to the extent that
they are not going to testify.
Just cant be someone thats intended to be called to testify basis of experts opinion is subject to cross or through
discovery.
Can be someone holding himself out to be a lawyer who actually isnt (say someone who was disbarred, but is working in
an investigatory manner). Privilege still applies.
Client can have representatives as well but has to be stated in confidence, limited number of people; information given in
confidence for legal advice with no extra people present
b. Who can seek legal advice for a business entity
A. Control Group test: limited to information that came from those that had the capacity to decide something once the legal
advice was given: the control group officers, managers, directors decision-makers. If a member of this group spoke to an
employee who spoke to the lawyer, thats not privileged
B. Subject-matter test: Argument that the control group in reality was not the place where the information prevalent to the
case was received. Now, under c/l, attorney can talk to or gather information from employees as long as its within the
process of giving legal advice and its covered under the privilege. This is the current rule.
c. Joint Defense Problem: where numerous Ds all get together with their respective lawyers to discuss the case. Not a waiver of the
privilege. Its considered a common defense or joint interest.
Example: Company B is being sued along with Company A and Company C involved in government prosecution and all are Ds.
These Ds may have interest in giving consistent (same) defense and may coordinate witness testimony or who is going to take control.
Say all lawyers are present to discuss legal question. Problem is that there are clients present, which you do not represent.
o Are these protected communications? Does privilege extend to this common meeting?

74

If C makes a deal, and now it is in Cs interest to disclose, can C testify (disclose) as to information of A & B, that
was stated during the common meeting? NO, B can stop both A and C from disclosing anything that happened at the
common meeting b/c privilege is held by B!

There is no privilege under this rule


1. Furtherence of Crime or Fraud: if the services of the lawyer were sought or obtained to enable or aid anyone to commit or
plan to commit what the client knew or reasonably should have known to be a crime or fraud; or
2. CLAIMANTS THROUGH SAME DECEASED CLIENT: As to a communication relevant to an issue between parties
who claim through the same deceased client, regardless of whether the claims are by testate or intestate succession or by inter
vivos transaction.
When the person is dead, and people are claiming an issue, say as to what is in the will, there is no privilege.
Reasoning: only one entity, and one legal relationship
3. Breach of duty by lawyer or client
4. Documents attested by lawyer
5. JOINT CLIENTS: communication relevant to a matter of common interest between 2 clients if it was made by one of them
to a lawyer retained or consulted in common, when offered in an action between any of the clients, but still privileged against
3rd parties
EXAMPLE: Divorce. Dont want to file themselves and go to a consultant pro se kind of thing, dont want to spend
a lot of money. Go together and go to the same attorney because they dont think there will be a problem, then
realize that there is a problem so one goes out and gets a different lawyer. Any statements made are NOT protected
because they consulted a lawyer in common protected against 3rd aprties but not protected as between them !
Justification: at the time, couple was one client, thus discussions amongst one couple is not privileged
What about non-natural persons? Lawyers in NY have developed entities that are different; each of those legally recognized entities is
separate, and has a right to attorney client privilege, i.e. government entities, etc.
Waiver component:
o You have to be sure that when a statement is made, all those that are present are under this privilege. If the statement is made
before a person that is not covered by this privilege, there is no privilege.
o If you are unaware of their existence, then the situation is different
o If you render legal advice with the intent to communicate it to others, then there is no privilege.
o Must be for legal advice ONLY
B. PSYCHOTHERAPIST-PATIENT: Patient is a person who consults or is examined by a psychotherapist. A psychotherapist is a
person licensed to treat mental or emotional conditions, including drug addiction, or a person licensed as a psychologist under state
law.
A communication is confidential if its not intended to be disclosed to a third party other than those present to further the
interest of the patient in the consultation, examination, or interview or a person reasonably necessary for the transmission of
the communication, or persons who are participating in the diagnosis and treatment under the direction of the
psychotherapist, including members of the patients family.
Must be made for the purpose of diagnosis or treatment of his mental or emotional condition
May be claimed by the patient, his guardian or conservator, or by the personal representative of a deceased patient.
Psychotherapist may claim the privilege, but only on behalf of the patient his authority to do so is presumed in the absence
of evidence to the contrary.
Exceptions
A. No privilege in proceedings to hospitalize the patient for mental illness
B. No privilege where judge orders exam of mental or emotional condition (unless judge orders privilege)
C. No privilege where D relies on his mental condition as an element of the offense (such as not guilty by reason of insanity)
NO ACCOUNTANT AND PHYSICIAN (APART FROM A SHRINK) PRIVILEGE IN FEDERAL SYSTEM! Not felt necessary to
aid in their condition
C. HUSBAND-WIFE: Two types of husband-wife privilege: Testimonial and communicational
Both spouses are the holders of the privilege
Applies to both criminal and civil proceedings
Anything that you observe has to be during the marriage, NOT before. Have to be legally married.
o What about common law marriage? If recognized in the jurisdiction, then privileged.

75

Cannot be a sham marriage still legally married, but havent been living together, and for practical purposes no
married. Thus, survives divorce

Confidential communication privilege:


(1) No 3rd parties can be present.
(2) Spouses hold the privilege and neither can disclose w/o consent of the other
(3) There must be a currently existing valid marriage. Must be recognized by the state
(4) Anything disclosed before you were married is not covered under the privilege.
(5) Common law marriage can be okay but it has to be recognized by the state and many states dont recognize common law
marriage
(6) The privilege has to be claimed when the marriage is still in existence and it cant be a shammed marriage
Exceptions:
1. No privilege where one spouse is charged with a crime against the other or a child or both;
Reasoning: Both participate in crime and against child molestation
2. No privilege as to matters which occurred prior to the marriage; and
3. No privilege regarding immoral purpose crimes, such as importing an alien for prositution
Testimonial privilege: If validly married, have a right to not testify against the other one.
If a spouse wants to testify at a criminal trial against the D-spouse, the accused cannot prevent his spouse from testifying if
she chooses he cant keep her off the stand, but she can choose not to testify.
Reasoning: Spouse should be able to decide if the marriage is worth saving.
Scope of the privilege is broader than confidential information she has a testimonial privilege as to communications and
observations = anything observed or communicated during the existence of a valid marriage (includes where a spouse
witnesses the commission of the crime) regardless of whether or not it was disclosed in confidence
Difference between confidential communicational and testimonial privilege
A. Confidential communications: Only to communications; survives divorce and applies to both (right to communicate)
B. Testimonial: Applies to both observations and communications; each spouse is the holder of the privilege and only
during the existing marriage (right to keep the marriage)
D. CLERGYMEN: Absolute Confidential Communications Privilege
Clergyman is a priest, minister, rabbi, etc. or a person believed to be so by the person consulting him.
Communication is confidential if made privately and not intended for further disclosure except to other persons present in
furtherance of the purpose of the communication
Privilege can be claimed by the person, his guardian, or by his personal representative if deceased. Clergymen can claim
privilege on behalf of the person.
E. WAIVER OF PRIVILEGE BY VOLUNTARY DISCLOSURE
Once you voluntarily disclose the privileged information, then privilege is waived, and it is admissible in any court for any
reason
But, if the person made a good faith effort to protect a privilege but it was disclosed inadvertently, courts typically allow the
privilege to be upheld
F. SELECTIVE DISCLOSURE: some places allow the privileged information to be admissible in the case at hand, but seal the
information so as to not be used in subsequent lawsuits. This is controversial and only allowed in few states
Absolute privilege = confidential communications spousal; clergyman
vs.
Qualified Privilege: (a) Informers: cant use the informers again; not always qualified; (b) police surveillance privilege protects
sources and ability to function in the future; (c) Journalistic; (d) trade secrets qualified; not based on confidential communications.
(e) Also 5th amdnt not based on confidential communication; (f) Peer Review tenured university positions and doctors (medical peer
review)

76

BURDENS OF PROOF
CIVIL BURDEN OF PROOF BELONGS TO P.
Burden of Production: Person bringing the action will have the burden of proof in almost every case. After P rests, D moves to
dismiss for failure to make out a prima facie case failure to meet burden of production
Burden of production is the minimum amount of evidence needed in order for the fact finder to decide that the P met the
burden of persuasion
Preponderance of the evidence = more probably true than not standard; more than the opposite
Viewed most favorably on the side of the non-moving party the P. Taking the probative value to the highest extent, can a
reasonable juror possibly find for the P?
o If yes, then the jury is entitled to hear the case. P has satisfied the burden of production has introduced prima facie
evidence on every element of the case (if there are 4 elements, you need to satisfy the burden of production for each
of those elements)
Burden of Persuasion: The burden is 50+% = Only time we have an actual numerical value
When burden of persuasion is clear and convincing evidence about 70-75% certain
o Undue influence cases, because its impugning the character of the individual
Fraud, missing people, civil commitment, denaturalization, etc
When taking kids away from parents and become wards of the state must be proven by clear and
convincing evidence as a federal standard. Higher burden to get something to happen
Burdens
A. More probably true than not 50+%
B. Clear and convincing (firm conviction; highly probable) clearly erroneous 70-75%
C. Beyond reasonable doubt not defined in jury instructions and lawyers cannot argue SC accepted definition, doubt which
would make you hesitate in deciding a matter in your everyday life Graham: 90-95%
Shifting Burdens: Typically, once the P meets the burden of production, it only means that he gets to go to the jury, not that the
burden shifts to the D. Shifting of burdens of production and burdens of persuasion come about in limited cases
Presumptions: Created based on probability (person driving car for company is probably working, not off on a private endeavor, if
something is mailed, theres a presumption that it got there) and social policy. Idea is that a presumed fact exists unless the D rebuts
it.
A presumption is not evidence; its a rule of law
Requires a certain fact to be accepted if another facts exists
If the D doesnt rebut, then its a directed verdict for the P on that fact jury instructed to find the presumed fact against the
D.
Presumption based on POLICY: Presumption of probability if havent heard from the guy in 7 years, presumed dead
Theories on how to rebut a presumption:
1) Thayer bursting bubble theory: Bursts the bubble of presumption: opponent of the presumption has to come forward and set
forth evidence that indicates that the presumption is wrong (D would have to show that he didnt get the letter). This evidence would
be viewed in the light most favorably to the D. Then, the presumption bursts it goes away. All youre left with is the natural
inference no direction to the jury. This shifts the burden of production only (not persuasion).
2) Morgan Method: Shifting the burden of persuasion and production: presumed fact is taken as true unless and until the
opponent establishes to the satisfaction of the jury that the fact did not exist.
Burden shifting usually comes from statutes, but Thayer is the default. In civil cases, when the state law provides the rule of
decision, then the state law with respect to presumptions is applicable
Example: Mailing a letter. P claims that something was mailed, and D person says they didnt get it. Law states that there is a
presumption of receipt if the P can prove more probably true than not true that in fact the letter was mailed presumed that when you
mail a letter it will be received because otherwise it will be sent back to the sender. Thus, the receiver must prove (rebut) they didnt
get it, and if he is unable to do so, Judge will instruct the jury if you find that the letter was mailed, you must find that it was
received
1) Thayer Bubble Position: prove more probably true than not that the letter was mailed, then the jury MUST find the letter
was received

77

2) Morgan Method: shifting of the burden of persuasion if letter mailed, then received, until the guy proves that he didnt
receive rule.
PRESUMPTIONS IS A RULE OF LAW. INFERENCE IS A DEDUCTION
CRIMINAL
Production = state has to come forward with evidence to establish the burden of production
Persuasion = in criminal case, the burden of persuasion is beyond a reasonable doubt
When viewed most favorably to the State, evidence has to be enough that a reasonable juror would find beyond a reasonable
doubt that the D is guilty.
There are no numbers defined in beyond a reasonable doubt. Most people say its about 90-95% likeliness. It is not 100%
certainty
State has burden of persuasion in each element of the case has to prove each.
o At the conclusion of the states case-in-chief, D will make motion of a directed verdict for acquittal, arguing that
theres an absence of prima facie evidence on one element
Burden of Presumption = can never shift the burden from the State to the D. Jury can never be instructed to find a fact in a
criminal case when its against the D. So you can never have a criminal prosecution against the D
Instructive Criminal Inference (may or may not be required to find) This is not presumption! Merely an inference to
the jury in the form of an instruction
o Example: If find televisions stolen, may or may not infer that the D knew they were stolen [inference that arises]
Burden is typically entirely on the State: EXCEPTIONS
1. True Affirmative Defense D who wishes to assert insanity has the burden of pleading, production, and persuasion to
establish that hes insane (have to give notice of intent to plead insanity, have to produce evidence, and have to establish
beyond a reasonable doubt that they are in fact crazy). This burden is placed on the D by the statute. Its not a criminal
presumption
2. So-Called Affirmative Defense entrapment, duress D is required to introduce some or any evidence (more than 0, a lot
less than a burden of production). Then, the State has the burden of proving the absence of duress or the absence of
entrapment beyond a reasonable doubt
ALIBI D has to give notice, but not a defense, just an argument negating State obligation to prove identity. The state has to prove
identity of the criminal (D), and Ds alibi is used to counter that So, D doesnt have to prove anything. (Self-defense is like alibi
burden is entirely on the State)

78

Vous aimerez peut-être aussi